Chapter: Head and Neck Anatomy; Topic: Muscles of the Neck; Subtopic: Sternocleidomastoid Muscle
Keyword Definitions:
• Sternocleidomastoid: A paired muscle in the neck that helps in rotation and flexion of the head.
• Accessory Nerve (XI): Provides motor supply to sternocleidomastoid and trapezius muscles.
• Mastoid Process: A conical projection from the temporal bone, providing attachment for sternocleidomastoid.
• Neck Movements: Include flexion, extension, and rotation of the head via cervical muscles.
Lead Question – 2014
False about sternocleidomastoid?
a) Arises from sternum and clavicle
b) Insertes on mastoid process
c) Motor supply by spinal accessory nerve
d) Tilt the head on opposite side
Explanation:
The sternocleidomastoid arises from the manubrium of sternum and medial clavicle, and inserts on the mastoid process. It is supplied by the spinal accessory nerve (motor) and C2–C3 (proprioception). It tilts the head to the same side and rotates to the opposite side. Hence, option (d) is false.
1) Which muscle forms the posterior triangle’s anterior boundary?
a) Trapezius
b) Sternocleidomastoid
c) Platysma
d) Omohyoid
Explanation:
The posterior triangle of the neck is bounded anteriorly by the posterior border of the sternocleidomastoid, posteriorly by the anterior border of the trapezius, and inferiorly by the middle third of the clavicle. The sternocleidomastoid thus forms the anterior boundary. Answer: (b).
2) Which nerve injury causes weakness in turning the head to the opposite side?
a) Hypoglossal nerve
b) Spinal accessory nerve
c) Facial nerve
d) Vagus nerve
Explanation:
The spinal accessory nerve supplies the sternocleidomastoid and trapezius. Injury to this nerve causes weakness in turning the head to the opposite side (due to SCM paralysis) and difficulty elevating the shoulder. Hence, (b) is correct.
3) Which of the following structures is deep to the sternocleidomastoid?
a) Internal jugular vein
b) External jugular vein
c) Platysma
d) Transverse cervical vein
Explanation:
The internal jugular vein lies deep to the sternocleidomastoid, while the external jugular vein runs superficially over it. This relationship is important during venipuncture and neck surgery. Answer: (a).
4) A patient with torticollis has which muscle affected?
a) Trapezius
b) Sternocleidomastoid
c) Platysma
d) Omohyoid
Explanation:
Congenital torticollis results from fibrosis or injury to the sternocleidomastoid, leading to tilting of the head toward the affected side and rotation to the opposite side. The primary involved muscle is sternocleidomastoid. Answer: (b).
5) Which artery crosses the sternocleidomastoid superficially?
a) Superior thyroid artery
b) Occipital artery
c) Transverse cervical artery
d) Suprascapular artery
Explanation:
The occipital artery, a branch of the external carotid artery, crosses the sternocleidomastoid superficially near its upper part before reaching the posterior scalp. This is an important surgical landmark. Answer: (b).
6) Clinical Case: A 35-year-old man develops neck stiffness and pain after whiplash injury. Which muscle is most likely strained?
a) Splenius capitis
b) Sternocleidomastoid
c) Trapezius
d) Levator scapulae
Explanation:
Whiplash injury causes overstretching of anterior neck muscles, mainly the sternocleidomastoid, due to rapid hyperextension and flexion. The patient presents with pain on head rotation and tenderness along the muscle belly. Hence, (b) is correct.
7) Clinical Case: A surgeon performing lymph node biopsy accidentally damages a nerve under the SCM. What symptom develops?
a) Difficulty swallowing
b) Shoulder droop
c) Loss of facial expression
d) Loss of tongue movement
Explanation:
The spinal accessory nerve runs deep to the sternocleidomastoid before supplying the trapezius. Injury causes shoulder droop due to trapezius weakness. Therefore, shoulder droop (b) occurs post-surgery.
8) Clinical Case: An infant presents with head tilted to one side since birth. Which is the most probable cause?
a) Fracture of clavicle
b) Injury to sternocleidomastoid during delivery
c) Facial nerve palsy
d) Atlantoaxial dislocation
Explanation:
Birth injury to the sternocleidomastoid causes fibrosis and shortening, leading to congenital torticollis with the head tilted toward the affected side. Hence, (b) is correct.
9) Clinical Case: A man complains of inability to rotate head to the left. Which side of SCM is paralyzed?
a) Left
b) Right
c) Both
d) None
Explanation:
Each sternocleidomastoid rotates the face to the opposite side. Therefore, inability to turn head to the left indicates right SCM paralysis. Hence, (b) is correct.
10) Clinical Case: Swelling beneath the sternocleidomastoid along the carotid sheath indicates infection in which space?
a) Retropharyngeal space
b) Parapharyngeal space
c) Pretracheal space
d) Carotid space
Explanation:
The carotid sheath lies deep to the sternocleidomastoid and contains the carotid artery, internal jugular vein, and vagus nerve. Infection in this space appears as swelling beneath the SCM. Hence, (d) is correct.
Chapter: Head and Neck Anatomy; Topic: Pharynx; Subtopic: Killian’s Dehiscence and Pharyngoesophageal Junction
Keyword Definitions:
Killian’s dehiscence: A weak area between the thyropharyngeus and cricopharyngeus parts of the inferior constrictor muscle of the pharynx.
Cricopharyngeus muscle: The lower part of the inferior constrictor muscle, functioning as the upper esophageal sphincter.
Zenker’s diverticulum: A pharyngoesophageal pouch that protrudes through Killian’s dehiscence.
Pharyngoesophageal junction: The region between the pharynx and esophagus, significant for swallowing mechanics and pathology.
Lead Question (2014):
Killian's dehiscence is seen in?
a) Oropharynx
b) Nasopharynx
c) Cricopharynx
d) Vocal cords
Explanation:
Killian’s dehiscence is a triangular gap located between the oblique fibers of the thyropharyngeus and the transverse fibers of the cricopharyngeus, parts of the inferior constrictor muscle. It is a potential site of herniation leading to Zenker’s diverticulum. This weak area is anatomically part of the cricopharyngeal region. Answer: Cricopharynx. It is clinically important in dysphagia and diverticular formation.
1) Killian’s dehiscence lies between which muscle fibers?
a) Thyropharyngeus and cricopharyngeus
b) Stylopharyngeus and cricopharyngeus
c) Palatopharyngeus and stylopharyngeus
d) Superior and middle constrictor muscles
Explanation:
The Killian’s dehiscence is a weak gap between the oblique fibers of the thyropharyngeus and the transverse fibers of the cricopharyngeus muscle. Both belong to the inferior constrictor group. Answer: Thyropharyngeus and cricopharyngeus. This anatomical weakness predisposes individuals to Zenker’s diverticulum, which manifests as dysphagia and regurgitation of undigested food.
2) Zenker’s diverticulum arises through?
a) Laimer’s triangle
b) Killian’s dehiscence
c) Triangle of Petit
d) Pirogov’s triangle
Explanation:
Zenker’s diverticulum is a pulsion diverticulum that arises through Killian’s dehiscence, the weak area between the thyropharyngeus and cricopharyngeus parts of the inferior constrictor. It commonly presents with dysphagia, halitosis, and regurgitation. Answer: Killian’s dehiscence. It typically occurs in elderly patients and may require surgical correction through diverticulectomy.
3) The upper esophageal sphincter is formed by which muscle?
a) Thyropharyngeus
b) Cricopharyngeus
c) Stylopharyngeus
d) Palatopharyngeus
Explanation:
The upper esophageal sphincter (UES) is primarily formed by the cricopharyngeus muscle, the lower part of the inferior constrictor. It prevents regurgitation of esophageal contents and air entry during respiration. Answer: Cricopharyngeus. Dysfunction of this muscle may lead to Zenker’s diverticulum and swallowing difficulties in elderly patients.
4) Laimer’s triangle lies:
a) Above the cricopharyngeus
b) Below the cricopharyngeus
c) Between the middle and inferior constrictors
d) Between palatopharyngeus and stylopharyngeus
Explanation:
Laimer’s triangle is located just below the cricopharyngeus muscle. It represents another weak area through which rare esophageal diverticula may form. Answer: Below the cricopharyngeus. This triangle lies between longitudinal muscle fibers and the cricopharyngeus, but herniation here is much less common than through Killian’s dehiscence.
5) A patient with regurgitation of undigested food and gurgling in the neck is likely to have?
a) Achalasia cardia
b) Zenker’s diverticulum
c) Laryngocele
d) Cricopharyngeal spasm
Explanation:
Zenker’s diverticulum causes regurgitation of undigested food, gurgling sounds, halitosis, and possible aspiration. It arises through Killian’s dehiscence due to cricopharyngeal dysfunction. Answer: Zenker’s diverticulum. Barium swallow shows a posterior pouch at the pharyngoesophageal junction, and treatment is surgical myotomy or diverticulectomy.
6) Killian’s dehiscence is related to which part of the pharynx?
a) Nasopharynx
b) Oropharynx
c) Laryngopharynx
d) Hypopharynx
Explanation:
Killian’s dehiscence lies in the posterior wall of the laryngopharynx (hypopharynx), between the thyropharyngeus and cricopharyngeus parts of the inferior constrictor. Answer: Laryngopharynx. This area is clinically relevant because of its association with pharyngoesophageal diverticula formation, which may cause dysphagia and aspiration symptoms in elderly individuals.
7) Which nerve supplies the cricopharyngeus muscle?
a) Glossopharyngeal nerve
b) Pharyngeal plexus
c) Recurrent laryngeal nerve
d) Superior laryngeal nerve
Explanation:
The cricopharyngeus muscle receives its motor supply mainly from the recurrent laryngeal nerve (branch of the vagus), while the rest of the inferior constrictor is supplied by the pharyngeal plexus. Answer: Recurrent laryngeal nerve. Damage to this nerve can cause dysfunction of the upper esophageal sphincter, leading to swallowing difficulties and aspiration risk.
8) Which imaging technique best demonstrates Zenker’s diverticulum?
a) CT neck
b) Barium swallow
c) MRI
d) Ultrasound
Explanation:
A barium swallow radiograph is the diagnostic tool of choice for Zenker’s diverticulum. It demonstrates a posterior outpouching at the pharyngoesophageal junction, arising through Killian’s dehiscence. Answer: Barium swallow. CT or MRI may be used for complications, but barium swallow remains the most specific and sensitive method for identifying the diverticular sac.
9) Killian-Jamieson diverticulum differs from Zenker’s by arising:
a) Above cricopharyngeus
b) Below cricopharyngeus
c) Through Laimer’s triangle
d) In the thoracic esophagus
Explanation:
Killian-Jamieson diverticulum arises below the cricopharyngeus, unlike Zenker’s which occurs above it. It projects anterolaterally, whereas Zenker’s projects posteriorly. Answer: Below cricopharyngeus. The difference in anatomical site and direction of protrusion helps radiologists distinguish these diverticula during barium studies and avoid surgical misidentification.
10) In Zenker’s diverticulum, the most likely symptom due to compression of surrounding structures is:
a) Hoarseness
b) Dysphagia
c) Stridor
d) Dysphonia
Explanation:
Dysphagia is the most characteristic symptom of Zenker’s diverticulum. The pouch compresses the esophagus and retains food, causing regurgitation, halitosis, and aspiration. Answer: Dysphagia. Large diverticula may also cause neck swelling and gurgling. Surgical cricopharyngeal myotomy often resolves symptoms by eliminating the pressure gradient responsible for the outpouching.
Chapter: Anatomy; Topic: Anterior Abdominal Wall; Subtopic: Muscles and Ligamentous Structures of Abdominal Wall
Keyword Definitions:
• External Oblique: The outermost flat muscle of the anterior abdominal wall, involved in trunk rotation and flexion.
• Lacunar Ligament: Crescent-shaped ligament formed by fibers of the external oblique tendon attaching to the pubic bone.
• Pectineal Ligament: Ligament along the pectineal line of the pubic bone, contributing to the inguinal canal structure.
• Conjoint Tendon: Tendinous fusion of internal oblique and transversus abdominis, not external oblique.
• Inguinal Ligament: Formed by the lower border of external oblique aponeurosis from ASIS to pubic tubercle.
Lead Question – 2014
External oblique forms all except?
a) Lacunar ligament
b) Pectineal ligament
c) Conjoint tendon
d) Inguinal ligament
Explanation:
The conjoint tendon is not formed by the external oblique; it is formed by the fusion of the internal oblique and transversus abdominis aponeuroses. External oblique contributes to the inguinal ligament, lacunar ligament, and pectineal ligament. Understanding the origins of these structures is critical for hernia surgery and abdominal wall reconstruction. Hence, the answer is conjoint tendon.
1. Which ligament is formed by fibers of the external oblique attaching to pubic bone?
a) Lacunar ligament
b) Pectineal ligament
c) Conjoint tendon
d) Inguinal ligament
2. The inguinal ligament extends from:
a) Pubic symphysis to ASIS
b) ASIS to pubic tubercle
c) Pubic tubercle to iliac crest
d) Iliac crest to ASIS
3. Conjoint tendon is formed by which muscles?
a) External oblique only
b) Internal oblique and transversus abdominis
c) Rectus abdominis and external oblique
d) Transversus abdominis only
4. Pectineal ligament is associated with which part of abdominal wall?
a) Iliac crest
b) Pectineal line of pubic bone
c) ASIS
d) Pubic symphysis
5. Which structure forms the roof of the superficial inguinal ring?
a) External oblique aponeurosis
b) Internal oblique
c) Conjoint tendon
d) Lacunar ligament
6. A patient has direct inguinal hernia. Which ligament is most relevant surgically?
a) Lacunar ligament
b) Pectineal ligament
c) Conjoint tendon
d) Inguinal ligament
7. Which abdominal wall muscle contributes to the inguinal canal formation?
a) External oblique
b) Transversus abdominis
c) Rectus abdominis
d) Pyramidalis
8. Lacunar ligament is clinically important in:
a) Femoral hernia repair
b) Umbilical hernia repair
c) Diaphragmatic hernia
d) Hiatal hernia
9. In repair of a direct inguinal hernia, which structure is reinforced?
a) Conjoint tendon
b) External oblique
c) Inguinal ligament
d) Lacunar ligament
10. Which aponeurosis forms the superficial inguinal ring?
a) External oblique
b) Internal oblique
c) Transversus abdominis
d) Conjoint tendon
11. During abdominal wall reconstruction, avoiding damage to which tendon is crucial for conjoint action?
a) Conjoint tendon
b) Lacunar ligament
c) Inguinal ligament
d) Pectineal ligament
Explanation:
The external oblique contributes to the inguinal ligament, lacunar ligament, and pectineal ligament but not the conjoint tendon, which arises from the internal oblique and transversus abdominis aponeuroses. Proper knowledge of these anatomical relationships is essential for hernia surgery, abdominal wall reconstruction, and understanding the mechanics of the inguinal canal. Therefore, the correct answer is conjoint tendon.
Topic: Inguinal Canal and Its Contents; Subtopic: Nerves Associated with the Inguinal Canal
Keyword Definitions:
Inguinal Canal: An oblique passage in the lower anterior abdominal wall transmitting the spermatic cord in males and round ligament in females.
Deep Inguinal Ring: The internal opening of the inguinal canal located above the midpoint of the inguinal ligament.
Genitofemoral Nerve: A branch of the lumbar plexus (L1-L2) dividing into genital and femoral branches.
Ilioinguinal Nerve: A branch of L1 nerve that traverses part of the canal but not through the deep ring.
Pudendal Nerve: Supplies perineum, not related to the inguinal canal.
Lead Question – 2014
Nerve entering the inguinal canal through deep inguinal ring?
a) Ilioinguinal nerve
b) Pudendal nerve
c) Genital branch of genitofemoral nerve
d) Superior rectal nerve
Explanation: The genital branch of the genitofemoral nerve enters the inguinal canal through the deep inguinal ring and runs within the spermatic cord in males, supplying the cremaster muscle and scrotal skin. In females, it travels with the round ligament. The ilioinguinal nerve, however, enters the canal through the superficial ring only. Hence, the correct answer is c) Genital branch of genitofemoral nerve.
1. The genital branch of the genitofemoral nerve arises from:
a) L1-L2
b) T12-L1
c) L2-L3
d) L3-L4
Explanation: The genitofemoral nerve originates from the lumbar plexus with roots L1-L2. It divides into the genital and femoral branches. The genital branch enters the inguinal canal, while the femoral branch supplies skin over the femoral triangle. Thus, the correct answer is a) L1-L2.
2. The ilioinguinal nerve enters the inguinal canal through:
a) Deep inguinal ring
b) Superficial inguinal ring
c) Posterior wall
d) Femoral ring
Explanation: The ilioinguinal nerve passes through part of the inguinal canal but enters it by piercing the internal oblique muscle, not through the deep inguinal ring. It exits via the superficial ring to supply skin of the upper medial thigh and external genitalia. Hence, the answer is b) Superficial inguinal ring.
3. The nerve responsible for the cremasteric reflex is:
a) Femoral branch of genitofemoral nerve
b) Genital branch of genitofemoral nerve
c) Iliohypogastric nerve
d) Pudendal nerve
Explanation: The cremasteric reflex involves contraction of the cremaster muscle when the inner thigh is stroked. The afferent limb is supplied by the femoral branch of genitofemoral and ilioinguinal nerves, and the efferent limb by the genital branch of the genitofemoral nerve. Therefore, the correct answer is b) Genital branch of genitofemoral nerve.
4. Which nerve does not pass through the inguinal canal?
a) Ilioinguinal nerve
b) Genital branch of genitofemoral nerve
c) Pudendal nerve
d) None
Explanation: The pudendal nerve does not traverse the inguinal canal; it passes through the greater sciatic foramen, Alcock’s canal, and supplies the perineum. The genital branch of genitofemoral and ilioinguinal nerves, however, are associated with the inguinal canal. Hence, the correct answer is c) Pudendal nerve.
5. The contents of the spermatic cord include all except:
a) Vas deferens
b) Pampiniform plexus
c) Genital branch of genitofemoral nerve
d) Iliohypogastric nerve
Explanation: The iliohypogastric nerve does not enter the inguinal canal or spermatic cord. The spermatic cord carries vas deferens, testicular vessels, cremasteric artery, pampiniform plexus, and genital branch of the genitofemoral nerve. Therefore, the correct answer is d) Iliohypogastric nerve.
6. (Clinical) Injury to the genital branch of the genitofemoral nerve during hernia repair may cause:
a) Loss of cremasteric reflex
b) Loss of sensation in the scrotum
c) Weakness of abdominal muscles
d) Urinary retention
Explanation: The genital branch supplies the cremaster muscle and scrotal skin. Injury during hernia repair may abolish the cremasteric reflex and cause localized numbness. Other functions remain intact. Thus, the correct answer is a) Loss of cremasteric reflex.
7. (Clinical) During laparoscopic hernia repair, identifying which nerve helps avoid postoperative neuralgia?
a) Genital branch of genitofemoral nerve
b) Pudendal nerve
c) Obturator nerve
d) Lateral cutaneous nerve of thigh
Explanation: The genital branch of the genitofemoral nerve runs close to the internal spermatic vessels. During laparoscopic hernia repair, avoiding its injury prevents chronic groin pain or sensory loss. Hence, the correct answer is a) Genital branch of genitofemoral nerve.
8. (Clinical) A 35-year-old male presents with pain and loss of sensation in the anterior scrotum after hernia surgery. The most likely injured nerve is:
a) Iliohypogastric
b) Ilioinguinal
c) Genital branch of genitofemoral
d) Pudendal
Explanation: The genital branch of the genitofemoral nerve supplies the cremaster and skin of the anterior scrotum. Its injury leads to scrotal numbness and absent cremasteric reflex. The ilioinguinal nerve supplies root of penis and upper medial thigh. Thus, the answer is c) Genital branch of genitofemoral.
9. (Clinical) In females, the genital branch of the genitofemoral nerve supplies:
a) Round ligament of uterus
b) Labia majora
c) Uterus
d) Perineum
Explanation: In females, the genital branch accompanies the round ligament within the inguinal canal to the labia majora, providing sensory innervation. It corresponds to the scrotal supply in males. Hence, the correct answer is b) Labia majora.
10. (Clinical) During varicocele surgery, which nerve is most at risk due to its proximity to spermatic vessels?
a) Femoral branch of genitofemoral nerve
b) Genital branch of genitofemoral nerve
c) Iliohypogastric nerve
d) Obturator nerve
Explanation: The genital branch of the genitofemoral nerve runs with the spermatic cord structures and is close to the testicular vessels. It can be damaged during varicocele or hernia surgeries, leading to scrotal anesthesia. Thus, the answer is b) Genital branch of genitofemoral nerve.
Topic: Pelvic Spaces and Fascia
Subtopic: Cave of Retzius
Keyword Definitions:
Cave of Retzius: Also known as the retropubic space, it lies between the pubic symphysis and urinary bladder.
Pelvic Fascia: Connective tissue covering pelvic organs, providing support and passage for vessels.
Urinary Bladder: A hollow muscular organ for urine storage, located in the lesser pelvis.
Rectovesical Space: The peritoneal space between bladder and rectum in males.
Pubic Symphysis: Midline cartilaginous joint uniting the left and right pubic bones.
Lead Question – 2014
Where is the Cave of Retzius present?
a) Between urinary bladder and rectum
b) Between urinary bladder and cervix
c) In front of the bladder
d) Between the cervix and the rectum
Explanation: The Cave of Retzius, or retropubic space, is located anterior to the urinary bladder and posterior to the pubic symphysis, not in contact with the peritoneum. It contains loose connective tissue and the venous plexus, aiding in surgical access to the bladder. Hence, the correct answer is c) In front of the bladder.
1. The retropubic space of Retzius is bounded anteriorly by:
a) Rectum
b) Pubic symphysis
c) Urethra
d) Peritoneum
Explanation: The retropubic space (Cave of Retzius) lies between the pubic symphysis anteriorly and the urinary bladder posteriorly. This space is clinically important in bladder surgery and pelvic trauma. It contains the retropubic venous plexus. The correct answer is b) Pubic symphysis.
2. The space of Retzius is filled with:
a) Loose areolar tissue
b) Fat
c) Fibrous tissue
d) Muscular tissue
Explanation: The space of Retzius is a potential space filled with loose areolar connective tissue, allowing bladder distension and movement. It acts as a cushion and supports surrounding structures. Therefore, the correct answer is a) Loose areolar tissue.
3. The clinical importance of the space of Retzius is:
a) Site for hernia repair
b) Approach for retropubic prostatectomy
c) Passage of fallopian tubes
d) Site for peritoneal dialysis
Explanation: The space of Retzius provides a surgical approach to the prostate and bladder without breaching the peritoneum. Retropubic prostatectomy and bladder neck procedures utilize this route safely. Hence, the correct answer is b) Approach for retropubic prostatectomy.
4. Which structure is located posterior to the Cave of Retzius?
a) Pubic symphysis
b) Rectum
c) Urinary bladder
d) Peritoneum
Explanation: Posterior to the Cave of Retzius lies the urinary bladder, separated by fascia and connective tissue. The space serves as a buffer during bladder filling and emptying. Thus, the correct answer is c) Urinary bladder.
5. In females, the space of Retzius lies between:
a) Pubic symphysis and uterus
b) Pubic symphysis and urinary bladder
c) Urinary bladder and cervix
d) Rectum and vagina
Explanation: In females, the space of Retzius occupies the area between the pubic symphysis and the urinary bladder, similar to males. It helps in gynecological surgeries like Burch colposuspension. Therefore, the answer is b) Pubic symphysis and urinary bladder.
6. (Clinical) A 60-year-old male undergoing prostatectomy develops venous bleeding from the retropubic area. The source is likely from:
a) External iliac vein
b) Retropubic venous plexus
c) Superior vesical artery
d) Inferior epigastric vein
Explanation: During retropubic surgeries, injury to the retropubic venous plexus within the Cave of Retzius can cause severe bleeding. This plexus drains the prostate and bladder. Therefore, the correct answer is b) Retropubic venous plexus.
7. (Clinical) During bladder surgery, why is the retropubic space entered carefully?
a) It contains the ureters
b) It contains a venous plexus prone to bleeding
c) It contains peritoneum
d) It contains nerves to bladder wall
Explanation: The retropubic space holds a dense venous plexus (Santorini’s plexus), which, if injured, can lead to major bleeding. This makes careful dissection essential during urological procedures. Hence, the correct answer is b) It contains a venous plexus prone to bleeding.
8. (Clinical) A cystocele repair often involves dissection through which space?
a) Rectovaginal space
b) Space of Retzius
c) Ischiorectal fossa
d) Vesicouterine pouch
Explanation: During cystocele repair, the surgeon enters the space of Retzius to reposition the bladder and reinforce pelvic fascia. It provides access to the anterior vaginal wall without entering the peritoneal cavity. Hence, the answer is b) Space of Retzius.
9. (Clinical) A patient with pelvic trauma has urine extravasation in front of the bladder but not into the peritoneum. The space involved is:
a) Retropubic space (Cave of Retzius)
b) Rectovesical pouch
c) Ischiorectal fossa
d) Paravesical space
Explanation: Extraperitoneal urine extravasation occurs in the retropubic space (Cave of Retzius) after bladder rupture anteriorly. This distinguishes it from intraperitoneal rupture involving peritoneal spaces. Hence, the correct answer is a) Retropubic space (Cave of Retzius).
10. (Clinical) During laparoscopic surgery, which landmark helps identify the space of Retzius?
a) Cooper’s ligament
b) Ischial spine
c) Arcus tendineus fascia pelvis
d) Round ligament
Explanation: Cooper’s ligament (pectineal ligament) marks the lateral boundary of the space of Retzius during laparoscopic or open pelvic surgeries. It guides dissection in hernia repairs and bladder surgeries. Hence, the correct answer is a) Cooper’s ligament.
Topic: Anterior Abdominal Wall
Subtopic: Superficial Fascia and Its Extensions
Keyword Definitions:
• Scarpa’s fascia: Deep membranous layer of superficial fascia in the lower anterior abdominal wall.
• Camper’s fascia: Fatty layer superficial to Scarpa’s fascia.
• Buck’s fascia: Deep fascia of penis continuous with Scarpa’s fascia.
• Suspensory ligament of penis: Fibrous structure supporting penis, formed partly by Scarpa’s fascia.
Lead Question - 2014
True about Scarpa’s fascia?
a) Deep fascia of anterior abdominal wall
b) Also called Buck’s fascia
c) Attached to Iliotibial tract
d) Forms suspensory ligament of penis
Explanation:
The correct answer is d) Forms suspensory ligament of penis. Scarpa’s fascia is the deep membranous layer of superficial fascia in the lower anterior abdominal wall. It continues into the perineum as Colles’ fascia and contributes to the formation of the suspensory ligament of penis or clitoris. It limits fluid extravasation below the inguinal ligament.
1. The membranous layer of the superficial fascia of the anterior abdominal wall is known as:
a) Camper’s fascia
b) Scarpa’s fascia
c) Colles’ fascia
d) Dartos fascia
Explanation:
The correct answer is b) Scarpa’s fascia. It lies deep to Camper’s fascia and is most prominent below the umbilicus. It is a tough fibrous layer that is clinically important in preventing urine extravasation from penetrating into the thigh due to its attachment to the fascia lata just below the inguinal ligament.
2. Scarpa’s fascia is continuous with which fascia in the perineum?
a) Buck’s fascia
b) Colles’ fascia
c) Gallaudet’s fascia
d) Dartos fascia
Explanation:
The correct answer is b) Colles’ fascia. Scarpa’s fascia extends into the perineum and becomes continuous with Colles’ fascia (the superficial perineal fascia). This continuity allows fluid or urine from a ruptured urethra to track from the perineum to the anterior abdominal wall but not into the thigh due to fascial attachments.
3. Clinical case: A male patient with rupture of spongy urethra shows swelling over the lower abdomen and scrotum but not the thigh. This is due to attachment of which fascia?
a) Camper’s fascia
b) Dartos fascia
c) Scarpa’s fascia
d) Deep fascia of thigh
Explanation:
The correct answer is c) Scarpa’s fascia. It is attached to the fascia lata (deep fascia of thigh) just below the inguinal ligament, preventing urine from spreading into the thigh. Instead, urine collects in the scrotum, penis, and lower abdominal wall, forming a characteristic clinical pattern of extravasation.
4. Which structure lies deep to Scarpa’s fascia?
a) External oblique muscle
b) Deep fascia
c) Camper’s fascia
d) Skin
Explanation:
The correct answer is a) External oblique muscle. Scarpa’s fascia lies superficial to the external oblique aponeurosis in the lower abdominal wall. It serves as a plane separating the superficial fat (Camper’s fascia) from the muscular layer, important during surgical dissections and in the spread of fluid in trauma or infections.
5. Clinical case: A man sustains a pelvic fracture causing rupture of the membranous urethra. Urine is seen collecting deep to Colles’ fascia. Which layer of anterior abdominal wall fascia is continuous with this space?
a) Camper’s fascia
b) Scarpa’s fascia
c) Deep fascia
d) Dartos fascia
Explanation:
The correct answer is b) Scarpa’s fascia. This fascia forms a continuous plane with Colles’ fascia in the perineum. Urine from urethral injury may spread along this plane to involve the scrotum, penis, and lower anterior abdominal wall, highlighting its clinical importance in pelvic trauma.
6. Camper’s fascia is:
a) Fatty layer
b) Fibrous layer
c) Deep fascial layer
d) Muscular layer
Explanation:
The correct answer is a) Fatty layer. Camper’s fascia forms the superficial fatty layer of the anterior abdominal wall fascia. It is well developed in the lower abdomen and continues into the thigh and perineum. It acts as insulation and a site of fat storage, especially prominent in obese individuals.
7. Scarpa’s fascia is attached to:
a) Iliotibial tract
b) Fascia lata
c) Rectus sheath
d) Linea alba
Explanation:
The correct answer is b) Fascia lata. Scarpa’s fascia is attached to the fascia lata below the inguinal ligament, preventing downward spread of infections or fluid collections from the anterior abdominal wall to the thigh. This attachment has great clinical importance in localizing urinary extravasation in pelvic injuries.
8. Clinical case: Following catheterization injury, a patient develops swelling confined to the scrotum and lower abdomen. The urine is limited by which structure?
a) Deep perineal fascia
b) Camper’s fascia
c) Scarpa’s fascia
d) Dartos fascia
Explanation:
The correct answer is c) Scarpa’s fascia. It restricts the spread of urine beyond the lower abdominal wall due to its firm attachment to the fascia lata. Understanding this fascial continuity helps surgeons predict the spread of fluid collections in perineal and urethral injuries effectively.
9. The fascia forming the superficial perineal pouch roof is derived from:
a) Dartos fascia
b) Camper’s fascia
c) Scarpa’s fascia
d) Deep fascia
Explanation:
The correct answer is c) Scarpa’s fascia. The membranous layer of the superficial fascia continues as Colles’ fascia, which forms the roof of the superficial perineal pouch. This anatomical continuity allows infections or urine to spread along these fascial planes from the perineum to the lower abdomen.
10. Scarpa’s fascia contributes to the formation of:
a) Dartos muscle
b) Suspensory ligament of penis
c) Deep perineal fascia
d) Buck’s fascia
Explanation:
The correct answer is b) Suspensory ligament of penis. The membranous Scarpa’s fascia contributes to the formation of the suspensory ligament of penis or clitoris. This ligament helps anchor the root of the penis to the pubic symphysis, maintaining its position and stability during erection and movement.
Topic: Retroperitoneal Structures
Subtopic: Anatomy of Retroperitoneal Organs
Keyword Definitions:
Retroperitoneal structures: Organs located behind the peritoneum but in front of the posterior abdominal wall, such as kidneys, ureters, pancreas (except tail), duodenum (except first part), and adrenal glands.
Peritoneum: A serous membrane lining the abdominal cavity and covering abdominal organs.
Primary retroperitoneal organs: Organs that develop and remain behind the peritoneum, like kidneys and ureters.
Secondary retroperitoneal organs: Organs initially intraperitoneal but later become retroperitoneal, like duodenum and pancreas.
Lead Question (2014): Which of the following is a retroperitoneal structure?
a) Ileum
b) Jejunum
c) Ureter
d) Appendix
Explanation: The ureter is a primary retroperitoneal structure, meaning it develops and remains behind the peritoneum throughout life. Ileum, jejunum, and appendix are intraperitoneal structures surrounded by peritoneum and suspended by mesentery. Answer: c) Ureter
1. Which of the following organs is secondarily retroperitoneal?
a) Pancreas
b) Kidney
c) Ureter
d) Adrenal gland
Explanation: The pancreas is a secondarily retroperitoneal organ because it initially develops intraperitoneally but becomes fixed to the posterior abdominal wall as the peritoneum fuses during development. Answer: a) Pancreas
2. Which of the following organs is intraperitoneal?
a) Spleen
b) Kidney
c) Ureter
d) Duodenum (2nd part)
Explanation: The spleen is completely covered by peritoneum and attached by ligaments, making it an intraperitoneal organ. The others are retroperitoneal. Answer: a) Spleen
3. Which part of the duodenum is intraperitoneal?
a) First part
b) Second part
c) Third part
d) Fourth part
Explanation: The first part of the duodenum (ampulla) is intraperitoneal and mobile, while the rest (2nd, 3rd, and 4th parts) are retroperitoneal and fixed. Answer: a) First part
4. A patient undergoing surgery for a right renal mass has the peritoneum intact anteriorly. Which space is entered during surgery?
a) Retroperitoneal space
b) Intraperitoneal cavity
c) Subphrenic space
d) Omental bursa
Explanation: The kidney lies in the retroperitoneal space, so accessing it surgically without breaching the peritoneum keeps the dissection within the retroperitoneal space. Answer: a) Retroperitoneal space
5. The pancreas is considered retroperitoneal because -
a) It becomes fixed to posterior wall during development
b) It develops in the posterior wall
c) It is mobile within peritoneal cavity
d) It is surrounded by peritoneum completely
Explanation: The pancreas starts intraperitoneally but becomes secondarily retroperitoneal when the dorsal mesentery fuses with the posterior abdominal wall. Answer: a) It becomes fixed to posterior wall during development
6. A stab injury to the right lumbar region injures a retroperitoneal organ. Which structure is likely affected?
a) Kidney
b) Spleen
c) Stomach
d) Gall bladder
Explanation: The kidney lies in the retroperitoneal space of the lumbar region, making it prone to injury from posterior or lateral penetrating wounds. Answer: a) Kidney
7. Which retroperitoneal organ crosses the psoas major muscle?
a) Ureter
b) Spleen
c) Duodenum (1st part)
d) Jejunum
Explanation: The ureter descends vertically along the anterior surface of the psoas major muscle in the retroperitoneal space, from the renal pelvis to the urinary bladder. Answer: a) Ureter
8. Which statement about retroperitoneal organs is true?
a) They are partially covered by peritoneum
b) They are fully surrounded by peritoneum
c) They hang freely by mesentery
d) They lie inside the peritoneal cavity
Explanation: Retroperitoneal organs are only partially covered by peritoneum on their anterior surface, lying behind the peritoneal cavity. Answer: a) They are partially covered by peritoneum
9. A 50-year-old man with blunt trauma to the abdomen shows retroperitoneal hemorrhage on CT. Which organ is most likely injured?
a) Kidney
b) Spleen
c) Stomach
d) Small intestine
Explanation: Retroperitoneal hemorrhage typically arises from injury to retroperitoneal organs such as kidneys, pancreas, or duodenum. Answer: a) Kidney
10. Which retroperitoneal structure lies anterior to the right psoas major and crosses the pelvic brim?
a) Ureter
b) Appendix
c) Cecum
d) Sigmoid colon
Explanation: The right ureter passes anterior to the psoas major muscle, crosses the pelvic brim near the bifurcation of the common iliac vessels, and continues to the urinary bladder. Answer: a) Ureter
Topic: Lungs and Mediastinum
Subtopic: Hilum of Lungs
Keyword Definitions:
Hilum of lung: Depression on the mediastinal surface where bronchi, vessels, and nerves enter.
Azygous vein: Drains thoracic wall, arches over root of right lung into SVC.
Vagus nerve: Provides parasympathetic fibers to lungs.
SVC: Superior vena cava, drains blood from upper body to heart.
Arch of aorta: Curved portion of aorta, related to left lung hilum.
Lead Question - 2014
Not related to hilum of right lung?
a) Azygous vein
b) Vagus nerve
c) SVC
d) Arch of aorta
Explanation: The hilum of the right lung is related to the azygous vein, superior vena cava, and vagus nerve. However, the arch of the aorta is a relation of the left lung hilum, not the right. Thus, the correct answer is Arch of aorta. This distinction is important in thoracic anatomy.
Guessed Questions for NEET PG:
1) Structure most commonly arching over the root of the right lung?
a) Pulmonary trunk
b) Azygous vein
c) Right phrenic nerve
d) Right brachiocephalic vein
Explanation: The azygous vein arches over the root of the right lung before draining into the superior vena cava. This anatomical landmark is unique to the right side. The correct answer is Azygous vein. This relation is crucial during mediastinal dissections and radiological imaging for diagnosis.
2) In a patient undergoing lung surgery, which nerve must be preserved near hilum of right lung?
a) Phrenic nerve
b) Vagus nerve
c) Hypoglossal nerve
d) Sympathetic trunk
Explanation: The vagus nerve passes posterior to the hilum of the right lung and provides parasympathetic innervation. Injury to it can cause impaired bronchoconstriction and secretory reflexes. Thus, the correct answer is Vagus nerve. Preservation is critical during lung resections and mediastinal lymph node dissections.
3) Which vessel lies anterior to the right lung hilum?
a) Superior vena cava
b) Azygous vein
c) Pulmonary artery
d) Arch of aorta
Explanation: The superior vena cava lies anterior to the hilum of the right lung, draining venous blood into the right atrium. This relation helps surgeons and radiologists distinguish normal vascular landmarks. The correct answer is Superior vena cava. Its injury can cause rapid hemodynamic compromise during thoracic procedures.
4) Clinical case: A 50-year-old with carcinoma lung has enlarged lymph nodes compressing the structure arching over right hilum. Which is affected?
a) Aortic arch
b) Left subclavian artery
c) Azygous vein
d) Right pulmonary artery
Explanation: Lymphadenopathy around the right lung hilum often compresses the azygous vein, which arches over it. Compression may cause venous congestion. The correct answer is Azygous vein. Recognition of this relation helps in staging lung carcinoma and planning surgery or radiotherapy.
5) Right pulmonary artery in relation to hilum lies:
a) Superior to right bronchus
b) Inferior to right bronchus
c) Anterior to right bronchus
d) Posterior to right bronchus
Explanation: In the right lung, the pulmonary artery lies anterior to the bronchus at the hilum. This arrangement differs from the left lung, where the artery is superior to bronchus. The correct answer is Anterior to right bronchus. This anatomical difference is remembered by “RALS” (Right Anterior, Left Superior).
6) Clinical case: During mediastinoscopy, the surgeon encounters a nerve behind right lung hilum. Likely identity?
a) Phrenic nerve
b) Vagus nerve
c) Intercostal nerve
d) Accessory nerve
Explanation: The vagus nerve runs posterior to the root of the lung, unlike the phrenic nerve, which runs anterior. Identifying this during thoracic procedures prevents iatrogenic injury. The correct answer is Vagus nerve. Injury may cause autonomic dysfunction of bronchi and impaired airway regulation.
7) Which structure is not found in hilum of lungs?
a) Pulmonary artery
b) Pulmonary veins
c) Main bronchus
d) Thoracic duct
Explanation: The hilum transmits bronchi, pulmonary arteries, and pulmonary veins. Thoracic duct does not pass through the hilum, instead ascends in the posterior mediastinum. Therefore, the correct answer is Thoracic duct. Differentiating contents is key for interpreting chest radiographs and CT scans.
8) Clinical scenario: A stab wound injures a structure anterior to right hilum. Which is likely damaged?
a) Superior vena cava
b) Aortic arch
c) Azygous vein
d) Descending thoracic aorta
Explanation: Anterior to the right lung hilum lies the superior vena cava. A penetrating injury here may cause fatal venous hemorrhage. The correct answer is Superior vena cava. Identifying this anatomical relation helps trauma surgeons anticipate bleeding sources in thoracic injuries.
9) Which structure is posterior to both lung hila?
a) Azygous vein
b) Descending aorta
c) Vagus nerve
d) Pulmonary veins
Explanation: The vagus nerve is consistently posterior to the hilum of both lungs. This relation is surgically important in thoracic approaches. Thus, the correct answer is Vagus nerve. Damage may result in autonomic dysfunction including impaired bronchial tone regulation.
10) Clinical case: CT chest shows a mass compressing right hilum. Which vessel arching above hilum must be checked?
a) Aortic arch
b) Azygous vein
c) Pulmonary artery
d) Superior vena cava
Explanation: The azygous vein arches above the right lung hilum. On CT scans, compression of this vein may indicate mediastinal pathology or hilar mass. The correct answer is Azygous vein. Recognizing this landmark helps radiologists in staging lung cancers and evaluating mediastinal syndromes.
Topic: Thorax
Subtopic: Relations of Lung Hilum
Keyword Definitions:
Hilum of lung: Area on the medial surface of lung where bronchi, vessels, and nerves enter or leave.
Vagus nerve: Cranial nerve X, provides parasympathetic supply to thoracic and abdominal organs.
Azygos vein: Vein draining posterior thoracic wall, arches over right lung root.
SVC: Large vein returning venous blood from upper body to right atrium.
Arch of aorta: Major arterial curve from ascending to descending aorta, giving major branches.
Lead Question - 2014
Posterior relation of hilum of lung ?
a) Azygous vein
b) SVC
c) Vagus nerve
d) Arch of aorta
Explanation: The posterior relation of the lung hilum is the vagus nerve, which passes behind the root of both lungs. Azygos vein arches over right lung root anteriorly, and arch of aorta lies superiorly. Correct answer: c) Vagus nerve.
Guessed Questions for NEET PG:
1) Which nerve lies anterior to the lung hilum?
a) Vagus
b) Phrenic
c) Sympathetic trunk
d) Intercostal
Explanation: The phrenic nerve passes anterior to the root of the lung, while the vagus nerve lies posterior. This relationship is crucial during thoracic surgeries. Correct answer: b) Phrenic.
2) Which vessel arches over the right lung hilum?
a) Hemiazygos vein
b) Azygos vein
c) Superior vena cava
d) Internal thoracic vein
Explanation: The azygos vein arches over the root of the right lung to drain into the superior vena cava. This is a classical radiological landmark in chest imaging. Correct answer: b) Azygos vein.
3) Which structure crosses the arch of aorta anteriorly?
a) Left phrenic nerve
b) Left vagus nerve
c) Left recurrent laryngeal nerve
d) Left subclavian vein
Explanation: The left phrenic nerve crosses the arch of aorta anteriorly, while the left recurrent laryngeal nerve hooks around ligamentum arteriosum posteriorly. Correct answer: a) Left phrenic nerve.
4) A patient with left lung tumor compresses the posterior relation of hilum. Which nerve is likely affected?
a) Left phrenic
b) Left vagus
c) Sympathetic chain
d) Intercostal nerve
Explanation: Tumors at the left lung hilum posteriorly may compress the vagus nerve, leading to parasympathetic dysfunction, hoarseness, or cardiac reflex changes. Correct answer: b) Left vagus.
5) In right lung hilum, which lies superior among structures?
a) Pulmonary vein
b) Pulmonary artery
c) Main bronchus
d) Azygos vein
Explanation: At the right hilum, the bronchus lies posterior, pulmonary artery superior, and pulmonary veins anterior-inferior. This arrangement is remembered as "B-A-V" (Back to front). Correct answer: b) Pulmonary artery.
6) Left recurrent laryngeal nerve hooks around?
a) Right subclavian artery
b) Arch of aorta
c) Left pulmonary artery
d) Superior vena cava
Explanation: The left recurrent laryngeal nerve, branch of vagus, hooks around the arch of aorta near ligamentum arteriosum. Correct answer: b) Arch of aorta.
7) Which bronchus is more prone to foreign body aspiration?
a) Left main bronchus
b) Right main bronchus
c) Both equal
d) Depends on position
Explanation: The right main bronchus is shorter, wider, and more vertical, making it more prone to aspiration of foreign bodies. Correct answer: b) Right main bronchus.
8) During hilar lymph node dissection, which nerve injury can cause hoarseness of voice?
a) Phrenic
b) Sympathetic
c) Recurrent laryngeal
d) Intercostal
Explanation: Injury to recurrent laryngeal nerve during lymph node dissection causes vocal cord paralysis and hoarseness. Correct answer: c) Recurrent laryngeal.
9) In chest radiograph, hilar shadow mainly represents?
a) Bronchi
b) Pulmonary arteries
c) Lymph nodes
d) Pulmonary veins
Explanation: On radiographs, hilar shadows are predominantly formed by pulmonary arteries. Bronchi are air-filled and less visible. Correct answer: b) Pulmonary arteries.
10) A 60-year-old smoker has left hilar mass compressing anterior relation of hilum. Which nerve is affected?
a) Left vagus
b) Left phrenic
c) Left recurrent laryngeal
d) Sympathetic chain
Explanation: The anterior relation of the lung hilum is the phrenic nerve. Compression leads to diaphragmatic palsy, breathlessness, and raised hemidiaphragm. Correct answer: b) Left phrenic.
Subtopic: Thoracic Duct Formation
Keyword Definitions:
Thoracic duct: The largest lymphatic vessel in the human body draining lymph from most areas.
Cisterna chyli: Dilated sac at the lower end of thoracic duct collecting lymph from abdomen.
Subclavian vein: Vein that drains blood from upper limb into brachiocephalic vein.
Jugular vein: Vein draining blood from head and neck.
Brachiocephalic vein: Large vein formed by subclavian and internal jugular veins.
Lead Question - 2014
Thoracic duct is formed by?
a) Union of left subclavian and left internal jugular vein.
b) Union of brachiocephalic vein and internal jugular vein
c) Continuation of upper end of cisterna chyli
d) None of the above
Explanation: The thoracic duct originates as the continuation of the cisterna chyli at the level of L1-L2 vertebrae, ascending through the thorax. It drains into the venous system at the junction of the left internal jugular and left subclavian veins. Correct answer: c) Continuation of upper end of cisterna chyli.
Guessed Questions for NEET PG:
1) Length of thoracic duct is approximately?
a) 10 cm
b) 20 cm
c) 40 cm
d) 50 cm
Explanation: The thoracic duct measures about 40 cm in adults. It starts from cisterna chyli in the abdomen and ascends to the venous angle. Its long course makes it prone to injury during surgery. Correct answer: c) 40 cm.
2) Thoracic duct pierces diaphragm through?
a) Aortic hiatus
b) Caval opening
c) Esophageal hiatus
d) None
Explanation: The thoracic duct passes through the diaphragm along with the aorta at the aortic hiatus at the level of T12 vertebra. This is a key anatomical relation during abdominal and thoracic surgeries. Correct answer: a) Aortic hiatus.
3) Which vein receives terminal drainage of thoracic duct?
a) Right subclavian vein
b) Left brachiocephalic vein
c) At junction of left internal jugular and left subclavian vein
d) Superior vena cava
Explanation: The thoracic duct terminates into the venous system at the left venous angle, i.e., the junction of the left subclavian vein and left internal jugular vein. This is a key anatomical landmark. Correct answer: c) Junction of left internal jugular and subclavian vein.
4) Thoracic duct drains all except?
a) Left upper limb
b) Right thorax
c) Left abdomen
d) Left thorax
Explanation: The thoracic duct drains lymph from entire body except the right upper limb, right thorax, right side of head and neck, which are drained by the right lymphatic duct. Correct answer: b) Right thorax.
5) In a neck surgery, thoracic duct injury leads to leakage of?
a) Blood
b) Bile
c) Chyle
d) Lymphocyte-depleted fluid
Explanation: Injury to thoracic duct leads to chylous fistula, with leakage of milky chyle rich in triglycerides. This complication is common during left neck dissections near the venous angle. Correct answer: c) Chyle.
6) Cisterna chyli is located at?
a) T8-T9
b) L1-L2
c) S1-S2
d) T12
Explanation: The cisterna chyli is located anterior to the bodies of L1 and L2 vertebrae, behind the right crus of diaphragm. It acts as the reservoir for intestinal and lumbar lymph trunks. Correct answer: b) L1-L2.
7) In chylothorax, fluid accumulates in?
a) Pleural cavity
b) Peritoneal cavity
c) Pericardial cavity
d) Subarachnoid space
Explanation: Chylothorax occurs when the thoracic duct is injured, leading to leakage of chyle into the pleural cavity. It is a serious surgical complication, requiring drainage and repair. Correct answer: a) Pleural cavity.
8) Right lymphatic duct drains lymph from?
a) Right upper limb
b) Right thorax
c) Right side of head and neck
d) All of the above
Explanation: The right lymphatic duct drains lymph from right upper limb, right thorax, and right side of head and neck. It terminates into the right venous angle. Correct answer: d) All of the above.
9) In case of lymphoma, thoracic duct obstruction may cause?
a) Ascites
b) Chylothorax
c) Chylous ascites
d) Edema
Explanation: Thoracic duct obstruction due to malignancy such as lymphoma may cause chylous ascites, characterized by milky fluid in peritoneum. This is a clinical indicator of lymphatic obstruction. Correct answer: c) Chylous ascites.
10) During oesophageal carcinoma surgery, thoracic duct is at risk at level of?
a) T2
b) T4
c) T8
d) T12
Explanation: Thoracic duct runs posterior to oesophagus in thorax, closely related at T4 to T8 levels. Surgical manipulation in esophagectomy carries risk of injury. Correct answer: c) T8.
Topic: Tibia and Fibula
Subtopic: Blood Supply and Relations
Keyword Definitions:
Tibia: The larger medial bone of the leg, weight-bearing, articulates with femur and talus.
Fibula: The slender lateral bone of the leg, non-weight-bearing, provides muscle attachment.
Nutrient artery: Artery supplying marrow and cortex of long bones, usually from main neighboring artery.
Common peroneal nerve: Branch of sciatic nerve, winds around fibular neck, prone to injury.
Osteomyelitis: Infection of bone and marrow, tibia commonly involved due to poor soft tissue cover.
Lead Question - 2014
False about tibia-fibula is ?
a) Nutrient artery of tibia is from posterior tibial artery
b) Nutrient artery of fibula is from peroneal artery
c) Proximal end of tibia is related to common peroneal nerve
d) Tibia is the most common site of osteomyelitis
Explanation: The nutrient artery of the tibia is from the posterior tibial artery, while the fibula receives from the peroneal artery. The common peroneal nerve relates to the fibula, not tibia. Tibia is the most common site for osteomyelitis. Thus, the false statement is c.
Guessed Questions for NEET PG
1) Nutrient artery of fibula arises from?
a) Anterior tibial artery
b) Peroneal artery
c) Posterior tibial artery
d) Popliteal artery
Explanation: The nutrient artery of the fibula usually comes from the peroneal artery, a branch of the posterior tibial artery. Correct answer: b.
2) Which nerve winds around neck of fibula?
a) Tibial nerve
b) Common peroneal nerve
c) Sural nerve
d) Saphenous nerve
Explanation: The common peroneal nerve winds around the lateral aspect of the fibular neck, making it vulnerable to injury in fractures. Correct answer: b.
3) Most common site of osteomyelitis in long bones?
a) Tibia
b) Femur
c) Fibula
d) Radius
Explanation: The tibia is the most common site for osteomyelitis because of subcutaneous position and limited soft tissue covering. Correct answer: a.
4) A patient with fracture neck of fibula develops foot drop. Likely nerve injured?
a) Tibial nerve
b) Sural nerve
c) Common peroneal nerve
d) Saphenous nerve
Explanation: Common peroneal nerve injury at fibular neck leads to foot drop due to loss of dorsiflexion. Correct answer: c.
5) Nutrient artery of tibia arises from?
a) Popliteal artery
b) Anterior tibial artery
c) Posterior tibial artery
d) Peroneal artery
Explanation: The nutrient artery of tibia is a branch of the posterior tibial artery, supplying the shaft and marrow. Correct answer: c.
6) In open tibial fractures, complication seen commonly is?
a) Osteomyelitis
b) Osteosarcoma
c) Pseudarthrosis
d) Fat embolism
Explanation: Due to poor soft tissue cover, open tibial fractures are highly prone to chronic osteomyelitis. Correct answer: a.
7) Which part of tibia is subcutaneous and vulnerable to injury?
a) Lateral border
b) Posterior border
c) Anterior border
d) Superior surface
Explanation: The anterior border and medial surface of tibia are subcutaneous and commonly injured in trauma. Correct answer: c.
8) Proximal tibiofibular joint is what type?
a) Synovial plane
b) Syndesmosis
c) Cartilaginous
d) Fibrous
Explanation: The proximal tibiofibular joint is a synovial plane type of joint allowing slight gliding movements. Correct answer: a.
9) Commonest site of stress fractures in tibia?
a) Proximal end
b) Middle third
c) Lower third
d) Upper epiphysis
Explanation: Stress fractures of tibia are most often seen in lower third region due to repetitive strain in athletes. Correct answer: c.
10) A child presents with swelling over tibia, fever, and bone pain. Most likely diagnosis?
a) Osteosarcoma
b) Osteomyelitis
c) Fibrous dysplasia
d) Ewing’s sarcoma
Explanation: In a child, fever, localized pain, and swelling over tibia typically indicate acute osteomyelitis. Correct answer: b.
Topic: Foot Joints and Ligaments
Subtopic: Support of Talus
Keyword Definitions:
Talus: A tarsal bone of the ankle articulating with tibia, fibula, calcaneus, and navicular for weight transfer.
Spring ligament: Plantar calcaneonavicular ligament supporting head of talus and medial longitudinal arch.
Deltoid ligament: Strong medial ankle ligament preventing talar eversion, arising from medial malleolus.
Lateral collateral ligament (LCL): Group of three ligaments supporting lateral ankle joint, preventing inversion injuries.
Cervical ligament: Connects calcaneus and talus, stabilizing subtalar joint during inversion and eversion.
Lead Question - 2014
Ligament supporting the talus is ?
a) Spring ligament
b) Deltoid ligament
c) LCL
d) Cervical ligament
Explanation: The spring ligament (plantar calcaneonavicular ligament) forms a sling under the talus, supporting its head and maintaining the medial longitudinal arch of the foot. It is essential for weight transmission and stability. Thus, the correct answer is a) Spring ligament, with clinical importance in flatfoot deformity if weakened.
Guessed Questions for NEET PG
1) Which ligament prevents talar eversion?
a) Deltoid ligament
b) Spring ligament
c) LCL
d) Cervical ligament
Explanation: The deltoid ligament on the medial side prevents excessive talar eversion and stabilizes the ankle joint. Correct answer: a) Deltoid ligament.
2) Ligament most often injured in ankle inversion sprain?
a) Anterior talofibular
b) Deltoid
c) Cervical
d) Spring
Explanation: The anterior talofibular ligament, part of the lateral collateral ligament complex, is most commonly injured in inversion sprains. Correct answer: a) Anterior talofibular.
3) Ligament maintaining medial longitudinal arch?
a) Deltoid
b) Spring
c) Cervical
d) Plantar aponeurosis
Explanation: The spring ligament is key in maintaining the medial longitudinal arch by supporting the head of talus. Correct answer: b) Spring.
4) Which ligament connects calcaneus to talus in subtalar joint?
a) Spring
b) Cervical
c) Deltoid
d) LCL
Explanation: The cervical ligament lies between talus and calcaneus, stabilizing subtalar joint during inversion and eversion. Correct answer: b) Cervical.
5) Injury of spring ligament leads to?
a) High arch foot
b) Flatfoot
c) Clubfoot
d) In-toeing
Explanation: Weakening of spring ligament causes collapse of medial longitudinal arch, resulting in flatfoot deformity. Correct answer: b) Flatfoot.
6) A child presents with painful flatfoot and medial foot collapse. Likely structure involved?
a) Spring ligament
b) Deltoid ligament
c) Cervical ligament
d) LCL
Explanation: Pediatric flatfoot with medial arch collapse is most commonly due to weakened spring ligament support. Correct answer: a) Spring ligament.
7) The deltoid ligament arises from?
a) Lateral malleolus
b) Medial malleolus
c) Talus head
d) Navicular tuberosity
Explanation: The deltoid ligament originates from medial malleolus and spreads to talus, calcaneus, and navicular. Correct answer: b) Medial malleolus.
8) Which ligament supports talus during walking?
a) Cervical ligament
b) Spring ligament
c) LCL
d) Interosseous ligament
Explanation: The spring ligament forms a sling for the head of talus, supporting weight during walking. Correct answer: b) Spring ligament.
9) Ligament stabilizing lateral ankle against inversion?
a) Spring ligament
b) Lateral collateral ligament
c) Deltoid ligament
d) Cervical ligament
Explanation: Lateral collateral ligament complex stabilizes the ankle against inversion forces. Correct answer: b) Lateral collateral ligament.
10) A football player develops medial ankle swelling after forced eversion. Injured ligament?
a) Deltoid
b) Spring
c) Cervical
d) ATFL
Explanation: Forced eversion injuries stretch or tear the deltoid ligament on the medial ankle. Correct answer: a) Deltoid.
Topic: Fascia and Muscular Insertions
Subtopic: Iliotibial Tract
Keyword Definitions:
Iliotibial tract: Thickened lateral band of fascia lata extending from iliac crest to lateral condyle of tibia.
Gluteus maximus: Largest gluteal muscle inserting partly into iliotibial tract, aiding hip extension and lateral stability.
Fascia lata: Deep fascia of thigh enclosing muscles and forming iliotibial tract laterally.
Lateral tibial condyle: Outer condyle of tibia where iliotibial tract inserts, providing lateral knee stabilization.
Lead Question - 2014
True about iliotibial tract all except?
a) Receives insertion of gluteus maximus
b) Derived from fascia lata
c) Inserted on lateral tibial condyle
d) None
Explanation: The iliotibial tract is a thickened lateral band of fascia lata. It receives insertion from both gluteus maximus and tensor fasciae latae and inserts on the lateral tibial condyle (Gerdy’s tubercle). All statements are true, so the correct answer is d) None. Clinical importance lies in stabilizing the lateral knee.
Guessed Questions for NEET PG
1) Iliotibial tract inserts at:
a) Medial tibial condyle
b) Lateral tibial condyle
c) Femoral condyle
d) Patella
Explanation: The iliotibial tract inserts on the lateral tibial condyle at Gerdy’s tubercle, stabilizing the knee joint laterally. Correct answer: b) Lateral tibial condyle.
2) Muscle contributing to iliotibial tract is:
a) Gluteus medius
b) Sartorius
c) Tensor fasciae latae
d) Pectineus
Explanation: Tensor fasciae latae contributes fibers to the iliotibial tract, helping in hip abduction and medial rotation. Correct answer: c) Tensor fasciae latae.
3) Iliotibial band syndrome commonly affects:
a) Swimmers
b) Cyclists
c) Runners
d) Weightlifters
Explanation: Iliotibial band syndrome is an overuse injury commonly affecting runners due to repetitive friction over the lateral femoral condyle. Correct answer: c) Runners.
4) Iliotibial tract is a thickening of:
a) Crural fascia
b) Fascia lata
c) Scarpa’s fascia
d) Camper’s fascia
Explanation: The iliotibial tract is a specialized thickened band of fascia lata along the lateral thigh. Correct answer: b) Fascia lata.
5) Iliotibial tract stabilizes which joint?
a) Hip
b) Knee
c) Ankle
d) Shoulder
Explanation: The iliotibial tract stabilizes both hip and knee joints, particularly the lateral knee during walking and running. Correct answer: b) Knee.
6) A marathon runner complains of lateral knee pain, most likely diagnosis?
a) Meniscal tear
b) Iliotibial band syndrome
c) ACL injury
d) PCL injury
Explanation: Lateral knee pain in long-distance runners is often due to iliotibial band friction syndrome. Correct answer: b) Iliotibial band syndrome.
7) Iliotibial tract extends from:
a) Iliac crest to medial tibial condyle
b) Iliac crest to lateral tibial condyle
c) Ischium to patella
d) Femoral shaft to tibia
Explanation: Iliotibial tract extends from iliac crest to lateral tibial condyle (Gerdy’s tubercle). Correct answer: b) Iliac crest to lateral tibial condyle.
8) Which test assesses iliotibial band tightness?
a) Lachman test
b) Ober’s test
c) McMurray test
d) Thompson test
Explanation: Ober’s test is used clinically to check iliotibial band tightness. Correct answer: b) Ober’s test.
9) The iliotibial tract is thickest at:
a) Thigh
b) Leg
c) Foot
d) Knee joint line
Explanation: The iliotibial tract is thickest along the lateral thigh where it descends to insert on the tibia. Correct answer: a) Thigh.
10) Iliotibial tract aids gluteus maximus in:
a) Hip flexion
b) Hip extension
c) Knee flexion
d) Ankle plantarflexion
Explanation: The iliotibial tract transmits the action of gluteus maximus for hip extension and lateral stabilization. Correct answer: b) Hip extension.
Topic: Femoral Vessels and Related Structures
Subtopic: Adductor Canal (Hunter’s Canal)
Keyword Definitions:
Hunter’s Canal: Also called the adductor canal, it is an intermuscular passage in the middle third of thigh transmitting femoral vessels.
Femoral Artery: Major blood supply to lower limb passing through Hunter’s canal before becoming popliteal artery.
Saphenous Nerve: Largest cutaneous branch of femoral nerve traveling in Hunter’s canal.
Adductor Magnus: Muscle forming the posterior boundary of Hunter’s canal with opening for femoral vessels.
Lead Question - 2014
Hunter's canal is seen in?
a) Cubital fossa
b) Popliteal fossa
c) Thigh
d) Calf
Explanation: Hunter’s canal, or adductor canal, lies in the middle third of the thigh. It transmits femoral artery, femoral vein, saphenous nerve, and nerve to vastus medialis. It begins at apex of femoral triangle and ends at adductor hiatus. Correct answer: (c) Thigh.
1) Hunter’s canal begins at which landmark?
a) Apex of femoral triangle
b) Base of femoral triangle
c) Adductor tubercle
d) Popliteal fossa
Explanation: The Hunter’s canal begins at the apex of the femoral triangle and continues downward until it ends at adductor hiatus. Correct answer: (a) Apex of femoral triangle.
2) Which artery continues as popliteal artery after passing through Hunter’s canal?
a) Profunda femoris artery
b) Femoral artery
c) Anterior tibial artery
d) Posterior tibial artery
Explanation: The femoral artery passes through Hunter’s canal and exits via adductor hiatus to become the popliteal artery in the popliteal fossa. Correct answer: (b) Femoral artery.
3) Which nerve accompanies femoral artery in Hunter’s canal?
a) Saphenous nerve
b) Tibial nerve
c) Obturator nerve
d) Common peroneal nerve
Explanation: The saphenous nerve, a cutaneous branch of femoral nerve, accompanies femoral artery in Hunter’s canal but leaves before adductor hiatus. Correct answer: (a) Saphenous nerve.
4) A patient with thigh surgery has numbness along medial leg. Which structure was likely injured in Hunter’s canal?
a) Tibial nerve
b) Saphenous nerve
c) Sciatic nerve
d) Obturator nerve
Explanation: Injury to saphenous nerve in Hunter’s canal causes sensory loss along medial side of leg and foot. It is the only nerve passing through this canal. Correct answer: (b) Saphenous nerve.
5) Which muscle forms the posterior boundary of Hunter’s canal?
a) Adductor longus
b) Sartorius
c) Adductor magnus
d) Vastus medialis
Explanation: The posterior boundary of Hunter’s canal is formed by adductor longus and adductor magnus, while sartorius forms its roof. Correct answer: (c) Adductor magnus.
6) A 55-year-old with femoral artery occlusion undergoes bypass grafting. Which canal must surgeon access to reach distal femoral artery?
a) Inguinal canal
b) Hunter’s canal
c) Popliteal fossa
d) Femoral triangle
Explanation: The Hunter’s canal is accessed for surgical exposure of distal femoral artery before it enters popliteal fossa. Correct answer: (b) Hunter’s canal.
7) Which structure leaves Hunter’s canal before femoral vessels exit through adductor hiatus?
a) Femoral artery
b) Femoral vein
c) Saphenous nerve
d) Nerve to adductor magnus
Explanation: The saphenous nerve exits Hunter’s canal before adductor hiatus to follow great saphenous vein along medial leg. Correct answer: (c) Saphenous nerve.
8) Which nerve to quadriceps muscle is found in Hunter’s canal?
a) Nerve to vastus lateralis
b) Nerve to rectus femoris
c) Nerve to vastus medialis
d) Nerve to vastus intermedius
Explanation: The nerve to vastus medialis, a branch of femoral nerve, travels in Hunter’s canal alongside femoral vessels. Correct answer: (c) Nerve to vastus medialis.
9) A patient with arterial injury in Hunter’s canal presents with absent popliteal pulse. Which artery is affected?
a) Popliteal artery
b) Femoral artery
c) Anterior tibial artery
d) Profunda femoris artery
Explanation: Damage to the femoral artery in Hunter’s canal results in absent popliteal pulse and distal ischemia. Correct answer: (b) Femoral artery.
10) Which muscle forms the roof of Hunter’s canal?
a) Vastus medialis
b) Sartorius
c) Adductor longus
d) Pectineus
Explanation: The sartorius muscle forms the roof of Hunter’s canal by crossing over the femoral vessels. Correct answer: (b) Sartorius.
Topic: Knee Joint Stabilization
Subtopic: Muscular Control of Patellar Tracking
Keyword Definitions:
Patella: A sesamoid bone within quadriceps tendon that improves leverage of knee extension.
Lateral Dislocation: Common displacement of patella, usually toward the outer side of knee.
Vastus Medialis: Medial quadriceps muscle with oblique fibers preventing lateral patellar displacement.
Quadriceps Femoris: A group of four muscles extending the leg at the knee joint.
Lead Question - 2014
Lateral dislocation of patella is prevented by ?
a) Rectus femoris
b) Vastus intermedius
c) Vastus lateralis
d) Vastus medialis
Explanation: The vastus medialis oblique (VMO) fibers pull the patella medially and prevent its lateral dislocation. Rectus femoris aids flexion at hip and extension at knee, vastus lateralis tends to pull patella laterally, while intermedius acts centrally. Thus, the correct answer is (d) Vastus medialis.
1) Which muscle provides dynamic medial stabilization of the patella?
a) Vastus lateralis
b) Vastus intermedius
c) Vastus medialis
d) Rectus femoris
Explanation: The vastus medialis oblique provides dynamic medial stabilization by counteracting the lateral pull of vastus lateralis. Its oblique fibers are critical for proper patellar tracking. Correct answer: (c) Vastus medialis.
2) A 20-year-old athlete has recurrent patellar dislocation laterally. Weakness of which muscle predisposes to this?
a) Vastus medialis
b) Vastus lateralis
c) Biceps femoris
d) Sartorius
Explanation: Weakness of the vastus medialis reduces medial stabilization of patella, leading to recurrent lateral dislocation. Lateral muscles instead accentuate lateral pull. Correct answer: (a) Vastus medialis.
3) Which structure acts as a passive restraint against lateral patellar dislocation?
a) Medial patellofemoral ligament
b) Iliotibial band
c) Rectus femoris tendon
d) Lateral meniscus
Explanation: The medial patellofemoral ligament (MPFL) is the primary passive restraint against lateral patellar displacement. It works along with vastus medialis for stabilization. Correct answer: (a) Medial patellofemoral ligament.
4) A patient with lateral patellar dislocation most likely has tightness of which structure?
a) Iliotibial band
b) Vastus medialis
c) Semitendinosus
d) Gracilis
Explanation: Tightness of the iliotibial band laterally pulls the patella outward, favoring lateral dislocation. Vastus medialis provides medial correction, whereas gracilis and semitendinosus are not major stabilizers. Correct answer: (a) Iliotibial band.
5) Which quadriceps muscle inserts into the base of the patella without significant role in patellar tracking?
a) Vastus medialis
b) Rectus femoris
c) Vastus lateralis
d) Vastus intermedius
Explanation: The vastus intermedius inserts centrally into the base of the patella, contributing to extension but not significantly influencing patellar tracking. Correct answer: (d) Vastus intermedius.
6) A 19-year-old basketball player develops lateral patellar dislocation after a sudden pivot. Which muscle failure contributes most?
a) Vastus medialis
b) Vastus lateralis
c) Rectus femoris
d) Popliteus
Explanation: Failure of vastus medialis oblique to stabilize patella allows lateral displacement during dynamic sports. Vastus lateralis increases lateral pull, rectus femoris acts mainly on hip and knee extension. Correct answer: (a) Vastus medialis.
7) Which factor increases the risk of lateral patellar dislocation?
a) Shallow trochlear groove
b) Strong vastus medialis
c) Weak iliotibial band
d) Medial patellofemoral ligament thickening
Explanation: A shallow trochlear groove predisposes to lateral patellar dislocation as the patella cannot seat firmly during knee flexion. Other factors like vastus medialis strength and MPFL integrity provide stabilization. Correct answer: (a) Shallow trochlear groove.
8) Which imaging is best to assess trochlear dysplasia predisposing to patellar dislocation?
a) X-ray AP knee
b) CT scan
c) Ultrasound
d) MRI
Explanation: MRI provides best assessment of trochlear morphology, cartilage, ligaments, and dynamic alignment, useful for patellar instability evaluation. CT helps with bony detail but MRI offers comprehensive view. Correct answer: (d) MRI.
9) Which surgical procedure realigns patella in recurrent dislocations due to vastus medialis weakness?
a) MPFL reconstruction
b) ACL reconstruction
c) Meniscectomy
d) High tibial osteotomy
Explanation: Medial patellofemoral ligament (MPFL) reconstruction restores medial restraint in recurrent lateral patellar dislocations, often combined with strengthening of vastus medialis. Correct answer: (a) MPFL reconstruction.
10) A young woman presents with patellofemoral pain and lateral patellar tilt. Strengthening which muscle helps rehabilitation?
a) Vastus medialis
b) Vastus lateralis
c) Rectus femoris
d) Gluteus medius
Explanation: Strengthening the vastus medialis oblique is the primary rehabilitation strategy for lateral patellar tilt and anterior knee pain, as it restores medial stability. Correct answer: (a) Vastus medialis.
Topic: Lower Limb Muscles
Subtopic: Dorsiflexors of Foot
Keyword Definitions:
Dorsiflexion: Movement of the foot upward at the ankle joint, bringing the toes closer to the shin.
Tibialis anterior: Primary dorsiflexor of the foot, also inverts the foot.
Tibialis posterior: Muscle that plantarflexes and inverts the foot.
Peroneus brevis: Muscle that everts and weakly plantarflexes the foot.
Extensor digitorum brevis: Muscle on the dorsum of the foot, extends toes but does not dorsiflex significantly.
Lead Question - 2014
Which of the following dorsiflexes the foot?
a) Tibialis posterior
b) Tibialis anterior
c) Peroneus brevis
d) Extensor digitorum brevis
Explanation: Tibialis anterior is the chief dorsiflexor of the foot. It originates from the lateral tibia and inserts on the medial cuneiform and first metatarsal, allowing dorsiflexion and inversion. Other muscles listed do not act as strong dorsiflexors. Correct answer: b) Tibialis anterior.
Guessed Questions for NEET PG
1) Tibialis anterior inserts on?
a) Calcaneum
b) Navicular
c) Medial cuneiform and base of 1st metatarsal
d) Cuboid
Explanation: Tibialis anterior inserts on the medial cuneiform and base of the first metatarsal. This allows it to dorsiflex and invert the foot effectively. Correct answer: c) Medial cuneiform and base of 1st metatarsal.
2) Clinical: A patient with foot drop most likely has paralysis of?
a) Tibialis posterior
b) Tibialis anterior
c) Peroneus longus
d) Soleus
Explanation: Foot drop occurs due to paralysis of dorsiflexors, particularly tibialis anterior, commonly from damage to the common peroneal nerve. Correct answer: b) Tibialis anterior.
3) Which nerve innervates tibialis anterior?
a) Tibial nerve
b) Deep peroneal nerve
c) Superficial peroneal nerve
d) Femoral nerve
Explanation: Tibialis anterior is supplied by the deep peroneal nerve, a branch of the common peroneal nerve. This ensures dorsiflexion of the foot. Correct answer: b) Deep peroneal nerve.
4) Clinical: Weak dorsiflexion with sensory loss between first and second toe suggests injury to?
a) Tibial nerve
b) Deep peroneal nerve
c) Superficial peroneal nerve
d) Sural nerve
Explanation: The deep peroneal nerve supplies tibialis anterior and provides cutaneous sensation to the web between the first and second toes. Its injury causes weakness of dorsiflexion and sensory loss. Correct answer: b) Deep peroneal nerve.
5) Which movement is opposed by tibialis anterior?
a) Plantarflexion
b) Inversion
c) Eversion
d) Dorsiflexion
Explanation: Tibialis anterior causes dorsiflexion and inversion of the foot. It works against plantarflexion and eversion. Correct answer: a) Plantarflexion.
6) Clinical: Steppage gait is seen in lesions of?
a) Superficial peroneal nerve
b) Tibial nerve
c) Common peroneal nerve
d) Femoral nerve
Explanation: Steppage gait occurs in common peroneal nerve injury, due to paralysis of dorsiflexors like tibialis anterior. The patient lifts the leg high to prevent toe dragging. Correct answer: c) Common peroneal nerve.
7) Which of the following is NOT a dorsiflexor?
a) Extensor digitorum longus
b) Extensor hallucis longus
c) Tibialis posterior
d) Tibialis anterior
Explanation: Tibialis posterior is a plantarflexor and invertor, not a dorsiflexor. Other listed muscles dorsiflex the foot. Correct answer: c) Tibialis posterior.
8) Clinical: Which muscle is tested when patient is asked to dorsiflex and invert the foot against resistance?
a) Tibialis anterior
b) Tibialis posterior
c) Peroneus tertius
d) Soleus
Explanation: Dorsiflexion with inversion specifically tests tibialis anterior function, as it is the main muscle performing these combined actions. Correct answer: a) Tibialis anterior.
9) Clinical: Trauma to the lateral aspect of the knee may cause loss of dorsiflexion due to injury to?
a) Femoral nerve
b) Common peroneal nerve
c) Tibial nerve
d) Obturator nerve
Explanation: The common peroneal nerve winds around the neck of fibula, making it vulnerable to trauma. Its injury leads to foot drop from dorsiflexor paralysis. Correct answer: b) Common peroneal nerve.
10) Main action of tibialis anterior?
a) Plantarflexion and eversion
b) Dorsiflexion and inversion
c) Plantarflexion and inversion
d) Dorsiflexion and eversion
Explanation: Tibialis anterior produces dorsiflexion and inversion. It pulls the foot upwards and medially, crucial for normal gait. Correct answer: b) Dorsiflexion and inversion.
Topic: Lower Limb
Subtopic: Semitendinosus Muscle
Keyword Definitions:
Semitendinosus: A hamstring muscle located in the posterior thigh, important for knee flexion and hip extension.
Sciatic nerve: Largest nerve in the body, supplying the hamstring muscles through tibial and common peroneal divisions.
Pes anserinus: Combined tendon insertion of sartorius, gracilis, and semitendinosus on the medial tibia.
Knee flexion: Bending of the knee joint, performed by hamstring muscles.
Hip extension: Backward movement of the thigh, primarily performed by hamstrings and gluteus maximus.
Lead Question - 2014
True regarding semitendinosus ?
a) Supplied by common peroneal part of sciatic nerve
b) Proximal flashy distal thin
c) Distal flashy proximal thin
d) Proximal and distal thin middle fleshy
Explanation: Semitendinosus is supplied by tibial division of sciatic nerve, not common peroneal. The muscle is fleshy proximally and becomes thin distally before insertion at pes anserinus. Its unique structure helps in knee flexion and hip extension. Correct answer: b) Proximal flashy distal thin.
Guessed Questions for NEET PG
1) Semitendinosus inserts at?
a) Fibular head
b) Tibial tuberosity
c) Pes anserinus
d) Lateral tibial condyle
Explanation: Semitendinosus inserts with sartorius and gracilis as part of pes anserinus on the medial surface of the upper tibia. This insertion stabilizes the medial knee during walking. Correct answer: c) Pes anserinus.
2) Which nerve supplies semitendinosus?
a) Femoral nerve
b) Tibial division of sciatic
c) Common peroneal division
d) Obturator nerve
Explanation: Semitendinosus is innervated by the tibial division of the sciatic nerve, which supplies most hamstring muscles. This ensures both hip extension and knee flexion functions. Correct answer: b) Tibial division of sciatic.
3) Clinical: A patient with tibial division injury of sciatic nerve will lose which function of semitendinosus?
a) Knee extension
b) Hip flexion
c) Knee flexion
d) Plantarflexion
Explanation: Tibial division injury compromises hamstring function, including semitendinosus, leading to weakness of knee flexion and hip extension. Correct answer: c) Knee flexion.
4) The semitendinosus muscle lies?
a) Superficial to semimembranosus
b) Deep to semimembranosus
c) Lateral to biceps femoris
d) Between sartorius and gracilis
Explanation: Semitendinosus lies superficial to semimembranosus and medial to biceps femoris. Its tendon descends to form part of pes anserinus. Correct answer: a) Superficial to semimembranosus.
5) Clinical: Athlete presents with pain at medial tibial insertion after overuse. Which hamstring tendon is commonly involved?
a) Biceps femoris
b) Semitendinosus
c) Semimembranosus
d) Rectus femoris
Explanation: Overuse injury at medial tibial surface often involves pes anserinus tendons, especially semitendinosus. Pain occurs with resisted knee flexion and climbing. Correct answer: b) Semitendinosus.
6) Which of the following is NOT a hamstring muscle?
a) Biceps femoris (long head)
b) Semitendinosus
c) Semimembranosus
d) Adductor longus
Explanation: True hamstrings are biceps femoris long head, semitendinosus, and semimembranosus. Adductor longus is a medial thigh muscle, not a hamstring. Correct answer: d) Adductor longus.
7) Clinical: Semitendinosus harvesting is commonly used for?
a) Rotator cuff repair
b) ACL reconstruction
c) Meniscal repair
d) Tendon Achilles repair
Explanation: The semitendinosus tendon is frequently used in ACL reconstruction grafts due to its length and strength. This provides stability in anterior knee surgeries. Correct answer: b) ACL reconstruction.
8) Primary action of semitendinosus?
a) Knee extension
b) Knee flexion
c) Plantarflexion
d) Hip adduction
Explanation: Semitendinosus flexes the knee and extends the hip. Its tendon at pes anserinus also contributes to medial rotation of tibia. Correct answer: b) Knee flexion.
9) Clinical: A footballer sustains a hamstring tear with tenderness at ischial tuberosity. Which semitendinosus attachment is injured?
a) Pes anserinus
b) Lateral tibial condyle
c) Fibular head
d) Ischial tuberosity
Explanation: Semitendinosus originates from ischial tuberosity. Trauma here often results in proximal hamstring tear. Correct answer: d) Ischial tuberosity.
10) Semitendinosus assists in?
a) Lateral rotation of tibia
b) Medial rotation of tibia
c) Abduction of hip
d) Plantarflexion
Explanation: Through its pes anserinus insertion, semitendinosus assists medial rotation of tibia on femur during knee flexion, stabilizing the medial side. Correct answer: b) Medial rotation of tibia.
Topic: Lower Limb
Subtopic: Popliteus Muscle
Keyword Definitions:
Popliteus: Small triangular muscle at the back of the knee that unlocks the joint during flexion.
Tibial nerve: Major branch of the sciatic nerve supplying posterior leg muscles.
Intracapsular origin: Muscle origin lying within the capsule of a joint.
Knee locking: Stabilization mechanism in extension by quadriceps and ligaments.
Knee flexor: Muscles that bend the knee, e.g., hamstrings and popliteus.
Lead Question - 2014
True about popliteus are all except?
a) Flexor of knee
b) Intracapsular origin
c) Supplied by tibial nerve
d) Causes locking of knee
Explanation: The popliteus unlocks the knee, not locks it. It arises intracapsularly from lateral femoral condyle, supplied by tibial nerve, and assists knee flexion. Its unique role is initiating flexion by rotating femur laterally on tibia. Thus, the incorrect statement is “causes locking of knee.” Correct answer: d) Causes locking of knee.
Guessed Questions for NEET PG
1) Popliteus muscle originates from?
a) Medial femoral condyle
b) Lateral femoral condyle
c) Tibial tuberosity
d) Medial tibial condyle
Explanation: Popliteus originates intracapsularly from the lateral condyle of the femur and attaches to posterior tibia. This unique origin helps it unlock the knee joint during flexion. Therefore, the correct answer is b) Lateral femoral condyle.
2) Which nerve supplies popliteus?
a) Common peroneal nerve
b) Femoral nerve
c) Tibial nerve
d) Obturator nerve
Explanation: The tibial nerve, a division of the sciatic nerve, supplies the popliteus muscle. It provides both motor innervation and contributes to proprioceptive function of the knee. Hence, the correct answer is c) Tibial nerve.
3) Clinical: A patient cannot unlock his knee while walking. Which muscle is most likely weak?
a) Popliteus
b) Gastrocnemius
c) Quadriceps
d) Biceps femoris
Explanation: Unlocking of the knee during flexion initiation is specifically performed by the popliteus muscle. Weakness leads to difficulty starting knee flexion, especially from extended position. Hence, the correct answer is a) Popliteus.
4) Popliteus muscle action includes all EXCEPT?
a) Unlocking knee
b) Lateral rotation of femur
c) Assisting knee flexion
d) Locking knee
Explanation: Popliteus functions to unlock knee by rotating femur laterally on tibia, enabling flexion. It never locks the knee, which is done by quadriceps and ligaments. Therefore, the incorrect action is d) Locking knee.
5) Clinical: Injury to tibial nerve affects which popliteus function?
a) Knee extension
b) Knee locking
c) Knee unlocking
d) Plantarflexion
Explanation: The tibial nerve supplies popliteus, essential for unlocking knee. Nerve injury causes inability to initiate flexion due to failure in unlocking. Therefore, the correct answer is c) Knee unlocking.
6) Popliteus inserts on?
a) Posterior tibia above soleal line
b) Posterior tibia below soleal line
c) Medial tibial condyle
d) Tibial tuberosity
Explanation: The popliteus inserts on the posterior surface of tibia above the soleal line. This anatomical feature stabilizes posterior tibia during knee flexion. Hence, correct answer: a) Posterior tibia above soleal line.
7) Clinical: A football player suffers knee trauma, causing difficulty unlocking knee. Likely structure injured?
a) Popliteus
b) Vastus medialis
c) Anterior cruciate ligament
d) Gastrocnemius
Explanation: Inability to unlock knee strongly suggests popliteus muscle injury, as it is the only muscle initiating flexion by lateral femoral rotation. Therefore, the correct answer is a) Popliteus.
8) Popliteus acts by?
a) Medial rotation of tibia
b) Lateral rotation of tibia
c) Extension of knee
d) Abduction of tibia
Explanation: During open-chain movement, popliteus medially rotates tibia on femur; during closed-chain, it laterally rotates femur on tibia. This dual action helps in unlocking knee. Correct answer: a) Medial rotation of tibia.
9) Clinical: Patient with posterior knee pain and locking difficulty shows MRI lesion in muscle behind knee joint capsule. Which is most affected?
a) Popliteus
b) Biceps femoris
c) Semimembranosus
d) Gastrocnemius
Explanation: The only small intracapsular muscle at posterior knee capsule is popliteus. Lesion here causes pain and locking difficulties. Correct answer: a) Popliteus.
10) Popliteus forms relation with?
a) Floor of popliteal fossa
b) Roof of popliteal fossa
c) Medial wall of knee
d) Lateral wall of knee
Explanation: Popliteus forms the floor of popliteal fossa, along with posterior femur and posterior capsule of knee. This anatomical relation is crucial for surgical approaches. Correct answer: a) Floor of popliteal fossa.
Topic: Retinacula of Ankle
Subtopic: Inferior Extensor Retinaculum
Keyword Definitions:
Extensor retinaculum: Thickened bands of deep fascia at ankle that hold extensor tendons in place.
Inferior extensor retinaculum: Y-shaped band attached laterally to calcaneum and medially to medial malleolus and plantar fascia.
Calcaneum: Heel bone, provides attachment for inferior extensor retinaculum laterally.
Sole deep fascia: Thick fascial layer forming plantar aponeurosis, receives medial slip of retinaculum.
Fibula: Lateral bone of leg, superior slip does not attach here in inferior extensor retinaculum.
Lead Question - 2014
Not true about inferior extensor retinaculum?
a) Y shaped
b) Superior slip attached to lower end of fibula
c) Inferior slip attached to deep fascia of sole
d) Lateral attached to calcaneum
Explanation: Inferior extensor retinaculum is Y-shaped, laterally attached to calcaneum, with medial slips attaching to medial malleolus and plantar aponeurosis. It does not attach to the fibula. Hence option “superior slip attached to fibula” is incorrect. Correct answer: Superior slip attached to lower end of fibula.
1) The inferior extensor retinaculum is continuous with?
a) Flexor retinaculum
b) Superior extensor retinaculum
c) Peroneal retinaculum
d) Plantar aponeurosis
Explanation: Inferior extensor retinaculum is Y-shaped and related medially to the plantar aponeurosis. It is distinct from superior extensor retinaculum and flexor retinaculum. Correct answer: Plantar aponeurosis.
2) Which tendons pass deep to inferior extensor retinaculum?
a) Tibialis posterior, FDL
b) Tibialis anterior, EHL, EDL, peroneus tertius
c) Gastrocnemius, plantaris
d) Soleus, tibialis posterior
Explanation: Tendons of tibialis anterior, extensor hallucis longus, extensor digitorum longus, and peroneus tertius pass beneath inferior extensor retinaculum to reach foot. Correct answer: Tibialis anterior, EHL, EDL, peroneus tertius.
3) Clinical case: A runner with anterior ankle pain is found to have inflamed tendon under inferior extensor retinaculum. Most likely tendon is?
a) Tibialis posterior
b) Extensor hallucis longus
c) Peroneus longus
d) Flexor hallucis longus
Explanation: Extensor hallucis longus runs beneath inferior extensor retinaculum and is prone to frictional tenosynovitis in athletes. Correct answer: Extensor hallucis longus.
4) The Y-shaped inferior extensor retinaculum attaches laterally to?
a) Navicular bone
b) Calcaneum
c) Talus
d) Cuboid
Explanation: The inferior extensor retinaculum is fixed laterally to upper surface of calcaneum near sinus tarsi, forming its base. Correct answer: Calcaneum.
5) Which artery passes deep to inferior extensor retinaculum?
a) Posterior tibial artery
b) Anterior tibial artery / dorsalis pedis
c) Peroneal artery
d) Popliteal artery
Explanation: The anterior tibial artery continues as dorsalis pedis artery and passes beneath inferior extensor retinaculum with deep peroneal nerve. Correct answer: Anterior tibial artery / dorsalis pedis.
6) Which nerve accompanies dorsalis pedis artery under inferior extensor retinaculum?
a) Tibial nerve
b) Deep peroneal nerve
c) Superficial peroneal nerve
d) Sural nerve
Explanation: The deep peroneal nerve passes with dorsalis pedis artery under inferior extensor retinaculum and supplies dorsum of foot muscles and first webspace. Correct answer: Deep peroneal nerve.
7) Clinical case: Foot drop patient develops compression neuropathy under inferior extensor retinaculum. Which nerve is affected?
a) Tibial nerve
b) Deep peroneal nerve
c) Superficial peroneal nerve
d) Sural nerve
Explanation: Compression beneath inferior extensor retinaculum can entrap deep peroneal nerve, producing pain over dorsum and webspace with foot drop. Correct answer: Deep peroneal nerve.
8) Superior vs inferior extensor retinaculum: which is Y-shaped?
a) Superior
b) Inferior
c) Both
d) None
Explanation: Superior extensor retinaculum is a simple transverse band above ankle joint. Inferior extensor retinaculum is Y-shaped below ankle joint. Correct answer: Inferior.
9) Medial limb of inferior extensor retinaculum attaches to?
a) Lateral malleolus
b) Medial malleolus and plantar aponeurosis
c) Fibula shaft
d) Tarsal tunnel
Explanation: The medial limb divides into upper band to medial malleolus and lower band to plantar aponeurosis of sole. Correct answer: Medial malleolus and plantar aponeurosis.
10) Clinical case: Ankle arthroscopy incision near inferior extensor retinaculum risks injuring?
a) Posterior tibial nerve
b) Deep peroneal nerve and dorsalis pedis artery
c) Superficial peroneal nerve
d) Tibial artery
Explanation: Structures beneath inferior extensor retinaculum include dorsalis pedis artery and deep peroneal nerve. Injury during arthroscopy incision leads to vascular and neurological deficit. Correct answer: Deep peroneal nerve and dorsalis pedis artery.
Topic: Pectoral Region
Subtopic: Deltopectoral Groove
Keyword Definitions:
Deltopectoral groove: Depression between deltoid and pectoralis major, an important landmark in the pectoral region.
Cephalic vein: A superficial vein of upper limb draining into axillary vein, running in deltopectoral groove.
Axillary artery: Main artery of upper limb continuation of subclavian artery.
Basilic vein: Superficial vein draining medial forearm and arm into axillary vein.
Radial nerve: Major branch of brachial plexus supplying extensor compartment of arm and forearm.
Lead Question - 2014
Structure related to deltopectoral groove ?
a) Axillary artery
b) Cephalic vein
c) Baselic vein
d) Radial nerve
Explanation: The cephalic vein runs through the deltopectoral groove, lying between deltoid and pectoralis major. It pierces the clavipectoral fascia to join axillary vein. Other listed structures do not lie in this groove. Hence, the correct answer is Cephalic vein.
Guessed Questions:
1) Deltopectoral groove is situated between?
a) Deltoid and trapezius
b) Pectoralis major and trapezius
c) Deltoid and pectoralis major
d) Deltoid and subscapularis
Explanation: The deltopectoral groove is formed between deltoid laterally and pectoralis major medially. It is an anatomical landmark for venous access via cephalic vein. Correct answer is Deltoid and pectoralis major.
2) Which structure pierces the clavipectoral fascia in the deltopectoral triangle?
a) Basilic vein
b) Cephalic vein
c) Radial nerve
d) Thoracodorsal artery
Explanation: The cephalic vein passes through the deltopectoral triangle and pierces clavipectoral fascia to drain into axillary vein. This region is used surgically for catheter placement. Correct answer is Cephalic vein.
3) A surgeon inserting a pacemaker uses which anatomical landmark for venous access?
a) Cubital fossa
b) Deltopectoral groove
c) Axillary fold
d) Radial groove
Explanation: For pacemaker insertion, the cephalic vein in the deltopectoral groove is commonly accessed, providing a safe superficial route to axillary vein and superior vena cava. Correct answer is Deltopectoral groove.
4) Which muscle forms the lateral boundary of the deltopectoral groove?
a) Deltoid
b) Trapezius
c) Pectoralis major
d) Latissimus dorsi
Explanation: The deltoid muscle forms the lateral boundary of the deltopectoral groove, with pectoralis major on the medial side. Correct answer is Deltoid.
5) A patient undergoing surgery near the deltopectoral groove risks injury to which superficial vein?
a) Basilic vein
b) Cephalic vein
c) Median cubital vein
d) Great saphenous vein
Explanation: Surgical approaches near deltopectoral groove risk injury to the cephalic vein, which lies in this groove and drains into axillary vein. Correct answer is Cephalic vein.
6) The deltopectoral triangle is bounded superiorly by?
a) Clavicle
b) Scapula
c) Sternum
d) First rib
Explanation: The deltopectoral triangle is bounded by clavicle superiorly, deltoid laterally, and pectoralis major medially. Correct answer is Clavicle.
7) Which artery accompanies the cephalic vein in the deltopectoral groove?
a) Thoracoacromial artery
b) Axillary artery
c) Radial artery
d) Subclavian artery
Explanation: The deltoid branch of thoracoacromial artery accompanies the cephalic vein in deltopectoral groove. Correct answer is Thoracoacromial artery.
8) A catheter introduced through the cephalic vein in deltopectoral groove ultimately drains into?
a) Subclavian vein
b) Superior vena cava
c) Axillary vein
d) Internal jugular vein
Explanation: The cephalic vein drains into axillary vein after piercing clavipectoral fascia in deltopectoral triangle, hence catheters placed here ultimately reach superior vena cava. Correct answer is Axillary vein.
9) Which clinical procedure often utilizes the deltopectoral approach for exposure?
a) Rotator cuff repair
b) Pacemaker insertion
c) Coronary bypass
d) Appendectomy
Explanation: The deltopectoral approach provides surgical access to shoulder joint and proximal humerus, frequently used in fracture fixation and prosthesis insertion. Correct answer is Pacemaker insertion (via cephalic vein) and shoulder surgeries.
10) Deltopectoral groove serves as an anatomical landmark for which type of block?
a) Axillary block
b) Brachial plexus block
c) Interscalene block
d) Caudal block
Explanation: Deltopectoral groove is important for surgical access but not commonly used in anesthesia blocks. The axillary block targets brachial plexus, often guided by anatomical landmarks. Among given options, the closest association is with Axillary block.
Topic: Nipple and Areola
Subtopic: Lactiferous Ducts
Keyword Definitions:
Lactiferous ducts: Channels that carry milk from mammary glands to the nipple.
Nipple: The conical projection in the center of the areola where ducts open externally.
Areola: Pigmented circular skin around the nipple containing sebaceous glands.
Mammary gland: Modified sweat gland specialized in milk secretion.
Lobules: Functional units of breast, drained by lactiferous ducts.
Lead Question - 2014
How many lactiferous ducts open in nipple ?
a) 0 -10
b) 15 -20
c) 25 -50
d) 50 -75
Explanation: Each breast usually has 15 to 20 lactiferous ducts, each draining a separate lobe. These ducts converge and open independently on the nipple surface to deliver milk during lactation. Thus, the correct answer is 15–20.
Guessed Questions:
1) Which structure surrounds the openings of lactiferous ducts?
a) Areola
b) Sebaceous glands
c) Montgomery’s tubercles
d) Nipple
Explanation: The nipple contains the terminal openings of lactiferous ducts. It is surrounded by the areola, which houses sebaceous glands (Montgomery’s glands) that lubricate and protect during breastfeeding. Correct answer is Nipple.
2) A lactating mother presents with blockage of a single duct causing localized swelling. Which structure is obstructed?
a) Alveolus
b) Lactiferous sinus
c) Lactiferous duct
d) Areolar gland
Explanation: Obstruction of a lactiferous duct prevents drainage of milk from the corresponding lobe, producing localized swelling and pain. This condition can lead to mastitis if untreated. Correct answer is Lactiferous duct.
3) Which hormone primarily stimulates milk secretion from breast lobules?
a) Estrogen
b) Progesterone
c) Prolactin
d) Oxytocin
Explanation: Prolactin secreted from the anterior pituitary promotes milk synthesis in alveolar cells of breast lobules, while oxytocin causes milk ejection through ducts. Correct answer is Prolactin.
4) Which hormone is responsible for milk ejection reflex during suckling?
a) Estrogen
b) Prolactin
c) Oxytocin
d) Progesterone
Explanation: Oxytocin from the posterior pituitary causes contraction of myoepithelial cells around alveoli and ducts, resulting in milk letdown reflex during suckling. Correct answer is Oxytocin.
5) A 25-year-old lactating woman presents with cracked nipple and mastitis. Infection spreads most likely through?
a) Areolar glands
b) Lactiferous ducts
c) Lymphatic vessels
d) Intercostal nerves
Explanation: Infection of the breast often enters through cracks in the nipple and spreads via lactiferous ducts into glandular tissue, leading to mastitis. Correct answer is Lactiferous ducts.
6) Which type of gland is the mammary gland?
a) Apocrine gland
b) Holocrine gland
c) Merocrine gland
d) Modified sweat gland
Explanation: Mammary gland is a modified sweat gland of apocrine type, functioning in milk production under hormonal regulation. Correct answer is Modified sweat gland.
7) A newborn fails to suckle effectively. Which structure in the mother provides direct milk entry to the infant?
a) Areola
b) Lactiferous ducts
c) Nipple
d) Lobules
Explanation: The nipple is the direct structure delivering milk from lactiferous ducts to the infant’s mouth. Ineffective suckling prevents proper milk ejection. Correct answer is Nipple.
8) Which artery mainly supplies blood to the lactating breast?
a) Internal thoracic artery
b) Radial artery
c) Brachial artery
d) Axillary artery
Explanation: The breast receives blood supply mainly from perforating branches of internal thoracic artery, lateral thoracic artery, and intercostal arteries. Correct answer is Internal thoracic artery.
9) A 40-year-old woman presents with retracted nipple. This is commonly due to?
a) Obstruction of ducts
b) Fibrosis of lactiferous ducts
c) Blockage of lymphatics
d) Muscular spasm
Explanation: Carcinoma of the breast causes fibrosis and retraction of lactiferous ducts, leading to nipple retraction. Correct answer is Fibrosis of lactiferous ducts.
10) Which lymph nodes primarily drain the nipple and areola?
a) Parasternal nodes
b) Supraclavicular nodes
c) Axillary nodes
d) Infraclavicular nodes
Explanation: The axillary lymph nodes, especially anterior (pectoral) group, are the primary drainage site for nipple and areola, making them crucial in breast cancer staging. Correct answer is Axillary nodes.
Topic: Shoulder Joint
Subtopic: Intracapsular but Extrasynovial Structures
Keyword Definitions:
Intracapsular: Located within the fibrous capsule of a joint.
Extrasynovial: Outside the synovial membrane but still within the joint capsule.
Shoulder joint: A synovial ball-and-socket joint formed between the humerus and scapula.
Biceps tendon: The long head passes through the shoulder joint capsule but remains extrasynovial.
Triceps tendon: Attaches posteriorly, not intracapsular.
Lead Question - 2014
Intracapsular but extrasynovial is ?
a) Long head of triceps
b) Long head of biceps
c) Short head of biceps
d) Medial head of biceps
Explanation: The long head of biceps tendon lies within the capsule of the shoulder joint but remains outside the synovial cavity, making it intracapsular but extrasynovial. This feature protects the tendon while maintaining joint mobility. Correct answer is Long head of biceps.
Guessed Questions:
1) Which tendon passes through the intertubercular sulcus of the humerus?
a) Short head of biceps
b) Long head of biceps
c) Coracobrachialis
d) Teres major
Explanation: The long head of biceps tendon passes through the intertubercular sulcus and is held in place by the transverse humeral ligament. This orientation stabilizes the tendon during shoulder movement. Correct answer is Long head of biceps.
2) Which structure prevents dislocation of the long head of biceps tendon?
a) Glenoid labrum
b) Transverse humeral ligament
c) Rotator cuff
d) Deltoid muscle
Explanation: The transverse humeral ligament bridges across the intertubercular sulcus, preventing displacement of the long head of biceps tendon during arm movements. Correct answer is Transverse humeral ligament.
3) A 30-year-old man presents with shoulder pain. MRI reveals inflammation of the tendon passing through the intertubercular sulcus. Which tendon is affected?
a) Subscapularis
b) Long head of biceps
c) Supraspinatus
d) Coracobrachialis
Explanation: Bicipital tendinitis occurs due to inflammation of the long head of biceps tendon within the intertubercular sulcus. It often presents as anterior shoulder pain aggravated by flexion. Correct answer is Long head of biceps.
4) Which part of the glenoid labrum is attached to the long head of biceps tendon?
a) Inferior
b) Superior
c) Anterior
d) Posterior
Explanation: The long head of biceps tendon originates from the supraglenoid tubercle and the superior part of the glenoid labrum, helping stabilize the shoulder joint. Correct answer is Superior.
5) Intracapsular but extrasynovial tendon in the knee joint is?
a) Posterior cruciate ligament
b) Anterior cruciate ligament
c) Patellar ligament
d) Medial collateral ligament
Explanation: The cruciate ligaments (ACL and PCL) are located within the joint capsule but remain extrasynovial. This anatomical arrangement protects them from direct synovial exposure. Correct answer is Anterior cruciate ligament.
6) A patient develops rupture of intracapsular but extrasynovial tendon in the shoulder joint. Which movement will be most affected?
a) Shoulder abduction
b) Shoulder extension
c) Shoulder flexion
d) Shoulder adduction
Explanation: The long head of biceps assists in flexion of the shoulder joint. Rupture of this tendon leads to weakness of flexion and supination, particularly affecting overhead movements. Correct answer is Shoulder flexion.
7) The tendon of long head of biceps is enclosed in a synovial sheath. True or False?
a) True
b) False
Explanation: The long head of biceps tendon has a tubular synovial sheath, facilitating smooth gliding within the intertubercular sulcus during arm movements. Correct answer is True.
8) Which tendon rupture causes “Popeye deformity” in the arm?
a) Short head of biceps
b) Long head of biceps
c) Brachialis
d) Coracobrachialis
Explanation: Rupture of the long head of biceps tendon leads to distal bunching of the muscle belly, producing the “Popeye deformity.” It usually occurs in older adults or athletes. Correct answer is Long head of biceps.
9) Which tendon runs intracapsular in shoulder joint but remains extrasynovial?
a) Long head of triceps
b) Long head of biceps
c) Supraspinatus
d) Teres minor
Explanation: The long head of biceps tendon is the only tendon that lies intracapsular but remains extrasynovial in the shoulder joint. Correct answer is Long head of biceps.
10) Which structure keeps the tendon of long head of biceps in place during shoulder motion?
a) Coracohumeral ligament
b) Transverse humeral ligament
c) Glenohumeral ligament
d) Rotator cuff tendons
Explanation: The transverse humeral ligament stabilizes the long head of biceps tendon in the intertubercular sulcus, preventing its displacement during shoulder movements. Correct answer is Transverse humeral ligament.
Topic: Axilla
Subtopic: Axillary Folds
Keyword Definitions:
Axilla: Pyramidal space between arm and thorax transmitting major neurovascular structures.
Anterior axillary fold: Fold formed by lower border of pectoralis major.
Posterior axillary fold: Formed by latissimus dorsi and teres major.
Pectoralis major: Large anterior chest muscle forming anterior wall of axilla.
Pectoralis minor: Deeper chest muscle related to axillary vessels.
Subscapularis: Posterior axilla muscle, part of rotator cuff.
Teres major: Muscle forming part of posterior wall and fold.
Axillary folds: Landmark ridges important for surface anatomy and surgical orientation.
Lead Question - 2014
Anterior axillary fold is due to which muscle?
a) Pectoralis major
b) Pectoralis minor
c) Subscapularis
d) Teres major
Explanation: The anterior axillary fold is formed by the lower border of the pectoralis major muscle. Pectoralis minor lies deep but does not contribute. Subscapularis and teres major form posterior folds. Thus, the correct answer is a) Pectoralis major.
1) Which muscle forms the posterior axillary fold?
a) Pectoralis major
b) Latissimus dorsi and teres major
d) Pectoralis minor
Explanation: The posterior axillary fold is formed by latissimus dorsi and teres major muscles. These together create the posterior ridge visible at the axilla. Correct answer is b) Latissimus dorsi and teres major.
2) A surgeon palpates anterior axillary fold for clinical exam. Which nerve supplies its forming muscle?
a) Lateral and medial pectoral nerves
b) Thoracodorsal nerve
c) Long thoracic nerve
d) Axillary nerve
Explanation: The anterior axillary fold is formed by pectoralis major, supplied by both lateral and medial pectoral nerves. Correct answer is a) Lateral and medial pectoral nerves.
3) Which nerve injury affects posterior axillary fold?
a) Thoracodorsal nerve
b) Median nerve
c) Long thoracic nerve
d) Radial nerve
Explanation: The posterior axillary fold is mainly from latissimus dorsi, supplied by thoracodorsal nerve. Injury leads to weakened adduction and medial rotation. Correct answer is a) Thoracodorsal nerve.
4) Which structure forms the medial wall of axilla?
a) Serratus anterior
b) Subscapularis
c) Pectoralis minor
d) Latissimus dorsi
Explanation: The medial wall of axilla is formed by serratus anterior covering thoracic wall and supplied by long thoracic nerve. Correct answer is a) Serratus anterior.
5) A radical mastectomy may injure nerve causing "winged scapula". Which is it?
a) Long thoracic nerve
b) Axillary nerve
c) Thoracodorsal nerve
d) Median nerve
Explanation: Winging of scapula occurs after injury to the long thoracic nerve, paralyzing serratus anterior. Correct answer is a) Long thoracic nerve.
6) Which artery is most related to anterior wall of axilla?
a) Thoracoacromial artery
b) Profunda brachii artery
c) Posterior circumflex humeral artery
d) Subscapular artery
Explanation: The thoracoacromial artery pierces the clavipectoral fascia near pectoralis minor in the anterior wall of axilla. Correct answer is a) Thoracoacromial artery.
7) Floor of axilla consists of?
a) Skin, subcutaneous tissue, axillary fascia
b) Pectoralis major
c) Subscapularis
d) Teres major
Explanation: The floor is made by skin, subcutaneous tissue, and axillary fascia spanning between folds. Correct answer is a) Skin, subcutaneous tissue, axillary fascia.
8) Which lymph nodes are located along anterior axillary fold?
a) Pectoral nodes
b) Subscapular nodes
c) Apical nodes
d) Central nodes
Explanation: The pectoral (anterior) nodes lie along lateral thoracic vessels near anterior fold, draining breast and anterior thoracic wall. Correct answer is a) Pectoral nodes.
9) A patient with axillary abscess needs incision near anterior fold. Which vein is at risk?
a) Cephalic vein
b) Basilic vein
c) Median cubital vein
d) Brachial vein
Explanation: The cephalic vein ascends in the deltopectoral groove and pierces clavipectoral fascia near anterior fold. Correct answer is a) Cephalic vein.
10) Which muscle is deep to pectoralis major forming part of anterior wall?
a) Pectoralis minor
b) Subscapularis
c) Serratus anterior
d) Latissimus dorsi
Explanation: Pectoralis minor lies deep to pectoralis major and contributes to anterior wall of axilla. Correct answer is a) Pectoralis minor.
Copic: Axilla
Subtopic: Walls of Axilla
Keyword Definitions:
Axilla: Pyramidal space between upper limb and thorax, important for neurovascular passage.
Subscapularis: Muscle forming part of posterior wall of axilla.
Pectoralis major: Muscle forming anterior wall of axilla.
Latissimus dorsi: Muscle forming posterior axillary fold.
Teres major: Muscle forming part of posterior wall with subscapularis and latissimus dorsi.
Axillary artery: Main blood supply of axilla and upper limb.
Quadrangular space: Interval transmitting axillary nerve and posterior circumflex humeral artery.
Brachial plexus cords: Nerve bundles named in relation to axillary artery.
Lead Question - 2014
Posterior wall of axilla is formed by
a) Pectoralis major
b) Pectoralis minor
c) Subscapularis
d) Intercostal muscles
Explanation: The posterior wall of axilla is formed mainly by subscapularis in its upper part, and teres major with latissimus dorsi in the lower part. The anterior wall is by pectoralis muscles. Thus, correct answer is c) Subscapularis.
1) Which muscle forms the anterior wall of axilla?
a) Pectoralis major and minor
b) Subscapularis
c) Latissimus dorsi
d) Teres major
Explanation: The anterior wall of axilla is formed by pectoralis major and pectoralis minor muscles. These muscles form the anterior axillary fold and cover the neurovascular bundle. Correct answer is a) Pectoralis major and minor.
2) A surgeon retracts the posterior wall of axilla during lymph node dissection. Which major nerve is endangered?
a) Axillary nerve
b) Thoracodorsal nerve
c) Median nerve
d) Ulnar nerve
Explanation: The thoracodorsal nerve runs along the posterior wall supplying latissimus dorsi. It can be injured during axillary lymph node dissection, affecting arm adduction and internal rotation. Correct answer is b) Thoracodorsal nerve.
3) The lateral wall of axilla is formed by?
a) Intertubercular sulcus of humerus
b) Clavicle
c) Subscapularis
d) Pectoralis major
Explanation: The lateral wall of axilla is the narrowest and is formed by the intertubercular sulcus of humerus, which serves as the site for long head of biceps tendon. Correct answer is a) Intertubercular sulcus of humerus.
4) A patient with surgical neck fracture of humerus risks damage to structures passing through quadrangular space. Which artery is involved?
a) Posterior circumflex humeral artery
b) Profunda brachii artery
c) Subscapular artery
d) Superior thoracic artery
Explanation: The posterior circumflex humeral artery passes through the quadrangular space along with axillary nerve. It is vulnerable in surgical neck fractures of humerus. Correct answer is a) Posterior circumflex humeral artery.
5) Which nerve is closely related to the medial wall of axilla?
a) Long thoracic nerve
b) Radial nerve
c) Axillary nerve
d) Musculocutaneous nerve
Explanation: The long thoracic nerve runs along the medial wall of axilla, supplying serratus anterior. Injury causes winging of scapula. Correct answer is a) Long thoracic nerve.
6) A breast cancer patient undergoes radical mastectomy. Which nerve injury during axillary clearance leads to "winged scapula"?
a) Long thoracic nerve
b) Axillary nerve
c) Thoracodorsal nerve
d) Median nerve
Explanation: Injury to the long thoracic nerve during axillary clearance leads to paralysis of serratus anterior, producing winging of scapula. Correct answer is a) Long thoracic nerve.
7) Which structure forms the floor of axilla?
a) Skin, subcutaneous tissue, axillary fascia
b) Pectoralis major
c) Subscapularis
d) Clavicle
Explanation: The floor of axilla is formed by skin, subcutaneous tissue, and axillary fascia stretching between anterior and posterior folds. Correct answer is a) Skin, subcutaneous tissue, axillary fascia.
8) Which lymph nodes are located along the posterior wall of axilla?
a) Subscapular group
b) Pectoral group
c) Central group
d) Apical group
Explanation: The subscapular group of axillary lymph nodes lie along the posterior wall, accompanying subscapular vessels. They drain posterior thoracic wall and scapular region. Correct answer is a) Subscapular group.
9) A knife wound in posterior axilla injures latissimus dorsi. Which nerve is affected?
a) Thoracodorsal nerve
b) Long thoracic nerve
c) Axillary nerve
d) Median nerve
Explanation: The thoracodorsal nerve supplies latissimus dorsi and is located in the posterior wall of axilla. Injury weakens arm adduction and internal rotation. Correct answer is a) Thoracodorsal nerve.
10) Which muscle contributes to both posterior wall and posterior axillary fold?
a) Latissimus dorsi
b) Pectoralis major
c) Serratus anterior
d) Intercostal muscles
Explanation: The latissimus dorsi contributes to the posterior wall and forms the posterior axillary fold along with teres major. Correct answer is a) Latissimus dorsi.
Topic: Shoulder Region
Subtopic: Quadrilateral Space
Keyword Definitions:
Quadrilateral space: Anatomical space in posterior shoulder, transmits axillary nerve and posterior circumflex humeral artery.
Teres major: Muscle forming lower border of quadrilateral space.
Teres minor: Muscle forming upper border along with subscapularis and shoulder capsule.
Long head of triceps: Forms medial border of quadrilateral space.
Deltoid: Muscle overlying shoulder, not a boundary of quadrilateral space.
Neck of humerus: Lateral border of quadrilateral space.
Axillary nerve: Passes through quadrilateral space with posterior circumflex humeral artery.
Lead Question - 2014
Boundaries of quadrilateral space include all except?
a) Teres major
b) Long head of triceps
c) Neck of humerus
d) Deltoid
Explanation: The quadrilateral space is bordered superiorly by teres minor, inferiorly by teres major, medially by the long head of triceps, and laterally by the neck of humerus. The deltoid overlies the region but is not a boundary. Correct answer is Deltoid.
Guessed Questions
1. Nerve passing through quadrilateral space?
a) Radial nerve
b) Axillary nerve
c) Musculocutaneous nerve
d) Ulnar nerve
Explanation: The axillary nerve passes through the quadrilateral space along with posterior circumflex humeral artery, innervating deltoid and teres minor. Correct answer is Axillary nerve.
2. Artery passing through quadrilateral space?
a) Brachial artery
b) Posterior circumflex humeral artery
c) Subclavian artery
d) Radial artery
Explanation: The posterior circumflex humeral artery passes through the quadrilateral space with axillary nerve, supplying deltoid and shoulder joint. Correct answer is Posterior circumflex humeral artery.
3. Muscle forming superior border of quadrilateral space?
a) Teres major
b) Teres minor
c) Deltoid
d) Subscapularis
Explanation: The teres minor muscle forms the superior border of quadrilateral space, with teres major forming the inferior border. Correct answer is Teres minor.
4. Muscle forming medial border of quadrilateral space?
a) Long head of triceps
b) Lateral head of triceps
c) Deltoid
d) Supraspinatus
Explanation: The long head of triceps forms the medial border of the quadrilateral space, contributing to the safe passage of axillary nerve. Correct answer is Long head of triceps.
5. Lateral border of quadrilateral space?
a) Humeral shaft
b) Neck of humerus
c) Deltoid
d) Coracobrachialis
Explanation: The neck of humerus forms the lateral border of the quadrilateral space, providing a landmark for axillary nerve and posterior circumflex humeral artery passage. Correct answer is Neck of humerus.
6. Clinical test for axillary nerve injury?
a) Shoulder abduction against resistance
b) Elbow flexion
c) Wrist extension
d) Thumb opposition
Explanation: Axillary nerve injury can be assessed by testing shoulder abduction against resistance, as the deltoid is paralyzed. Sensory loss occurs over lateral shoulder. Correct answer is Shoulder abduction against resistance.
7. Fracture of surgical neck of humerus may damage?
a) Radial nerve
b) Axillary nerve
c) Median nerve
d) Ulnar nerve
Explanation: Fracture of the surgical neck of humerus may injure the axillary nerve passing through the quadrilateral space, causing deltoid paralysis and sensory deficit over lateral shoulder. Correct answer is Axillary nerve.
8. Muscle not contributing to quadrilateral space?
a) Teres minor
b) Deltoid
c) Teres major
d) Long head of triceps
Explanation: The deltoid overlies the quadrilateral space but does not form its boundary. Teres minor, teres major, and long head of triceps form superior, inferior, and medial borders respectively. Correct answer is Deltoid.
9. Posterior circumflex humeral artery arises from?
a) Axillary artery
b) Brachial artery
c) Subclavian artery
d) Radial artery
Explanation: The posterior circumflex humeral artery is a branch of the axillary artery passing through the quadrilateral space with axillary nerve. Correct answer is Axillary artery.
10. Injury to axillary nerve leads to?
a) Weak elbow extension
b) Weak shoulder abduction
c) Weak wrist flexion
d) Weak finger abduction
Explanation: Damage to the axillary nerve results in weakness of shoulder abduction due to deltoid paralysis and sensory loss over lateral shoulder. Correct answer is Weak shoulder abduction.
Chapter: Anatomy
Topic: Skull Anatomy
Subtopic: Superior Orbital Fissure
Keywords:
Superior Orbital Fissure: A gap between the greater and lesser wings of sphenoid bone transmitting nerves and vessels to the orbit.
Oculomotor Nerve (CN III): Motor nerve supplying most extraocular muscles controlling eye movement.
Trochlear Nerve (CN IV): Motor nerve supplying superior oblique muscle for eye movement.
Superior Ophthalmic Vein: Drains venous blood from the orbit into the cavernous sinus.
Lead Question - 2013:
Structure passing through superior orbital fissure?
a) Oculomotor nerve
b) Trochlear nerve
c) Superior ophthalmic vein
d) All of the above
Answer & Explanation:
Correct answer: d) All of the above.
Explanation: The superior orbital fissure transmits multiple important structures including the oculomotor nerve (CN III), trochlear nerve (CN IV), abducens nerve (CN VI), ophthalmic division of the trigeminal nerve (V1), and the superior ophthalmic vein. These structures facilitate eye movements, sensory supply, and venous drainage of the orbit.
MCQ 1:
The superior orbital fissure lies between which bones?
a) Frontal and ethmoid bones
b) Greater and lesser wings of sphenoid
c) Maxilla and palatine bones
d) Temporal and occipital bones
Answer & Explanation:
Correct answer: b) Greater and lesser wings of sphenoid.
Explanation: The superior orbital fissure is a gap between the greater and lesser wings of the sphenoid bone, allowing passage of important nerves and vessels to the orbit. Knowledge of this anatomy is critical in ophthalmologic and neurosurgical procedures.
MCQ 2:
Which nerve passing through the superior orbital fissure is responsible for pupil constriction?
a) Trochlear nerve (CN IV)
b) Oculomotor nerve (CN III)
c) Abducens nerve (CN VI)
d) Ophthalmic nerve (V1)
Answer & Explanation:
Correct answer: b) Oculomotor nerve (CN III).
Explanation: The oculomotor nerve (CN III) not only innervates most extraocular muscles but also carries parasympathetic fibers responsible for pupil constriction. Damage to CN III can cause ptosis, pupil dilation, and impaired eye movement, important in clinical neurology.
MCQ 3:
The trochlear nerve supplies which muscle?
a) Superior rectus
b) Inferior oblique
c) Superior oblique
d) Lateral rectus
Answer & Explanation:
Correct answer: c) Superior oblique.
Explanation: The trochlear nerve (CN IV) exclusively supplies the superior oblique muscle of the eye, controlling downward and lateral gaze. Lesions result in vertical diplopia and head tilt, key clinical features in diagnosing cranial nerve disorders.
MCQ 4 (Clinical):
Lesion in superior orbital fissure leads to which syndrome?
a) Horner’s syndrome
b) Superior orbital fissure syndrome
c) Cavernous sinus syndrome
d) Bell's palsy
Answer & Explanation:
Correct answer: b) Superior orbital fissure syndrome.
Explanation: Superior orbital fissure syndrome results from trauma, tumors, or inflammation affecting structures passing through the fissure. Symptoms include ophthalmoplegia, ptosis, proptosis, and sensory loss in the ophthalmic nerve distribution, requiring urgent diagnosis and management.
MCQ 5:
Which vein drains blood from the orbit?
a) Inferior ophthalmic vein
b) Superior ophthalmic vein
c) Angular vein
d) All of the above
Answer & Explanation:
Correct answer: d) All of the above.
Explanation: Both superior and inferior ophthalmic veins drain venous blood from the orbit into the cavernous sinus. The angular vein connects the facial vein to the ophthalmic veins. Their valveless nature is significant in the spread of infections from face to brain.
MCQ 6 (Clinical):
Damage to which nerve passing through the superior orbital fissure causes diplopia?
a) Optic nerve (CN II)
b) Oculomotor nerve (CN III)
c) Trigeminal nerve (CN V)
d) Hypoglossal nerve (CN XII)
Answer & Explanation:
Correct answer: b) Oculomotor nerve (CN III).
Explanation: Damage to the oculomotor nerve leads to ophthalmoplegia, ptosis, and diplopia due to impaired control of eye muscles. Superior orbital fissure lesions typically affect this nerve, making awareness essential in neuro-ophthalmological diagnostics.
MCQ 7:
The ophthalmic division of trigeminal nerve (V1) passes through?
a) Optic canal
b) Foramen rotundum
c) Superior orbital fissure
d) Foramen ovale
Answer & Explanation:
Correct answer: c) Superior orbital fissure.
Explanation: The ophthalmic division of the trigeminal nerve (V1) passes through the superior orbital fissure to provide sensory innervation to the forehead, scalp, and upper eyelid. Clinical relevance includes diagnosing neuralgia or trauma involving V1.
MCQ 8 (Clinical):
Superior orbital fissure syndrome primarily affects:
a) Motor and sensory function of orbit
b) Visual acuity
c) Hearing
d) Taste sensation
Answer & Explanation:
Correct answer: a) Motor and sensory function of orbit.
Explanation: Superior orbital fissure syndrome impairs motor control of extraocular muscles and sensory input from ophthalmic nerve (V1), leading to ophthalmoplegia, sensory loss, and proptosis. Early diagnosis is key to prevent permanent functional loss.
MCQ 9:
The abducens nerve (CN VI) passes through:
a) Optic canal
b) Superior orbital fissure
c) Foramen spinosum
d) Jugular foramen
Answer & Explanation:
Correct answer: b) Superior orbital fissure.
Explanation: The abducens nerve (CN VI) passes through the superior orbital fissure to innervate the lateral rectus muscle, which abducts the eye. Lesions result in diplopia due to inability to abduct the affected eye, important in neurological assessments.
MCQ 10 (Clinical):
Superior ophthalmic vein drains into:
a) Internal jugular vein
b) External jugular vein
c) Cavernous sinus
d) Subclavian vein
Answer & Explanation:
Correct answer: c) Cavernous sinus.
Explanation: The superior ophthalmic vein drains venous blood from the orbit directly into the cavernous sinus. Its valveless nature can facilitate retrograde infection spread from the face, emphasizing the need for care in treating periorbital infections.
Chapter: Anatomy
Topic: Tongue Anatomy
Subtopic: Muscles of the Tongue
Keywords:
Styloglossus: Muscle that retracts and elevates the tongue; innervated by hypoglossal nerve (CN XII).
Hyoglossus: Muscle that depresses the tongue; innervated by hypoglossal nerve (CN XII).
Palatoglossus: Muscle that elevates posterior tongue and closes oropharyngeal isthmus; innervated by vagus nerve (CN X).
Genioglossus: Muscle responsible for tongue protrusion; innervated by hypoglossal nerve (CN XII).
Lead Question - 2013:
Protrusion of tongue not possible in damage of?
a) Styloglossus
b) Hyoglossus
c) Palatoglossus
d) Genioglossus
Answer & Explanation:
Correct answer: d) Genioglossus.
Explanation: The genioglossus muscle is the primary muscle responsible for tongue protrusion. It is innervated by the hypoglossal nerve (CN XII). Damage to this muscle or its nerve results in inability to protrude the tongue and deviation toward the side of the lesion, critical in neurological exams.
MCQ 1:
Which nerve innervates the genioglossus muscle?
a) Vagus nerve (CN X)
b) Hypoglossal nerve (CN XII)
c) Glossopharyngeal nerve (CN IX)
d) Trigeminal nerve (CN V)
Answer & Explanation:
Correct answer: b) Hypoglossal nerve (CN XII).
Explanation: The genioglossus muscle receives motor innervation from the hypoglossal nerve (CN XII). It is responsible for protruding the tongue. Damage to the hypoglossal nerve leads to tongue deviation and atrophy, which are key clinical signs in neurological assessments.
MCQ 2:
The palatoglossus muscle is innervated by which nerve?
a) Hypoglossal nerve (CN XII)
b) Vagus nerve (CN X)
c) Facial nerve (CN VII)
d) Accessory nerve (CN XI)
Answer & Explanation:
Correct answer: b) Vagus nerve (CN X).
Explanation: Unlike other tongue muscles, the palatoglossus is innervated by the vagus nerve (CN X) via the pharyngeal plexus. It elevates the posterior tongue and plays a role in closing the oropharyngeal isthmus, important in swallowing. Damage may cause swallowing difficulties.
MCQ 3:
Which muscle retracts and elevates the tongue?
a) Styloglossus
b) Genioglossus
c) Hyoglossus
d) Palatoglossus
Answer & Explanation:
Correct answer: a) Styloglossus.
Explanation: The styloglossus muscle, innervated by the hypoglossal nerve (CN XII), retracts and elevates the tongue. It aids in swallowing and speech. Dysfunction may affect tongue retraction, causing speech and swallowing impairments, requiring clinical assessment during neurological or ENT examinations.
MCQ 4 (Clinical):
Which muscle is paralyzed if tongue deviates toward one side on protrusion?
a) Hyoglossus
b) Genioglossus
c) Palatoglossus
d) Styloglossus
Answer & Explanation:
Correct answer: b) Genioglossus.
Explanation: Paralysis of the genioglossus muscle leads to tongue deviation toward the affected side upon protrusion, as the unopposed contralateral muscle pushes the tongue sideways. This is a key clinical sign of hypoglossal nerve damage, aiding in early neurological diagnosis.
MCQ 5:
Function of the hyoglossus muscle is to:
a) Protrude tongue
b) Retract tongue
c) Depress the tongue
d) Elevate tongue
Answer & Explanation:
Correct answer: c) Depress the tongue.
Explanation: The hyoglossus muscle, innervated by the hypoglossal nerve (CN XII), depresses the sides of the tongue. It plays a critical role in tongue movement for articulation and swallowing. Damage leads to weakness in tongue depression, contributing to speech and swallowing difficulties.
MCQ 6 (Clinical):
After hypoglossal nerve injury, which symptom is expected?
a) Difficulty in tongue protrusion
b) Inability to blink
c) Loss of palatal elevation
d) Deviation of uvula
Answer & Explanation:
Correct answer: a) Difficulty in tongue protrusion.
Explanation: Hypoglossal nerve (CN XII) injury results in paralysis of tongue muscles (except palatoglossus), leading to tongue deviation towards the affected side and difficulty protruding the tongue. These signs are essential in clinical evaluation of cranial nerve lesions and in differential diagnosis of bulbar palsies.
MCQ 7:
Which muscle elevates the posterior tongue?
a) Styloglossus
b) Palatoglossus
c) Hyoglossus
d) Genioglossus
Answer & Explanation:
Correct answer: b) Palatoglossus.
Explanation: The palatoglossus muscle elevates the posterior tongue and closes the oropharyngeal isthmus during swallowing. It is innervated by the vagus nerve (CN X). Dysfunction may present as difficulty in swallowing and speech impairment, making it a clinically important muscle in cranial nerve assessment.
MCQ 8 (Clinical):
Which nerve damage leads to inability to retract the tongue?
a) Hypoglossal nerve
b) Vagus nerve
c) Trigeminal nerve
d) Facial nerve
Answer & Explanation:
Correct answer: a) Hypoglossal nerve.
Explanation: The hypoglossal nerve (CN XII) supplies the styloglossus and hyoglossus muscles that retract the tongue. Damage to this nerve leads to weakness in tongue retraction and deviation toward the affected side, providing crucial diagnostic information in neurology.
MCQ 9:
The muscle primarily responsible for tongue protrusion is:
a) Palatoglossus
b) Styloglossus
c) Hyoglossus
d) Genioglossus
Answer & Explanation:
Correct answer: d) Genioglossus.
Explanation: The genioglossus muscle is the prime mover for tongue protrusion. Innervated by the hypoglossal nerve (CN XII), it pulls the tongue forward. Damage leads to inability to protrude the tongue and deviation to the affected side, important in clinical examinations for cranial nerve dysfunction.
MCQ 10 (Clinical):
Which clinical test assesses the function of the hypoglossal nerve?
a) Asking the patient to smile
b) Asking the patient to protrude the tongue
c) Testing the gag reflex
d) Checking pupil size
Answer & Explanation:
Correct answer: b) Asking the patient to protrude the tongue.
Explanation: Testing the hypoglossal nerve (CN XII) involves asking the patient to protrude the tongue. Any deviation or weakness indicates nerve dysfunction. This simple clinical test helps localize neurological lesions and is essential during cranial nerve examination in patients with suspected neurological disorders.
Topic: Neck Anatomy
Subtopic: Muscular Layers of Neck
Keywords:
Platysma: A superficial muscle in the anterior neck involved in facial expression.
Trapezius: A large muscle extending over the back of the neck and shoulders, moving scapula and supporting arm.
Sternocleidomastoid: A prominent muscle in the neck responsible for head rotation and flexion.
Longus Colli: A deep neck muscle located along the anterior vertebral column, stabilizing the cervical spine.
Lead Question - 2013:
Deep injury of neck always involves?
a) Platysma
b) Trapezius
c) Sternocleidomastoid
d) Longus colli
Answer & Explanation:
Correct answer: a) Platysma.
Explanation: The platysma is the most superficial muscle of the anterior neck. Any deep penetrating neck injury must traverse this thin muscle layer first. Its involvement is a critical marker during trauma assessment, as it helps determine the depth of injury and the need for further surgical exploration or imaging.
MCQ 1:
Platysma is supplied by which nerve?
a) Facial nerve (CN VII)
b) Accessory nerve (CN XI)
c) Hypoglossal nerve (CN XII)
d) Vagus nerve (CN X)
Answer & Explanation:
Correct answer: a) Facial nerve (CN VII).
Explanation: The platysma muscle is innervated by the cervical branch of the facial nerve (CN VII). It plays a role in facial expression by tensing the skin of the neck. Knowledge of this innervation is important during neck surgeries to avoid iatrogenic nerve damage.
MCQ 2:
Which muscle is primarily responsible for head rotation?
a) Platysma
b) Sternocleidomastoid
c) Longus colli
d) Trapezius
Answer & Explanation:
Correct answer: b) Sternocleidomastoid.
Explanation: The sternocleidomastoid muscle, innervated by the accessory nerve (CN XI), is responsible for head rotation and flexion. Injury to this muscle can limit neck movement and should be evaluated in neck trauma or surgical cases for functional assessment and appropriate management.
MCQ 3:
Which layer is first encountered in a penetrating neck injury?
a) Longus colli
b) Platysma
c) Sternocleidomastoid
d) Prevertebral fascia
Answer & Explanation:
Correct answer: b) Platysma.
Explanation: The platysma is the most superficial muscle layer in the neck, lying just beneath the skin. Any penetrating injury that reaches deep structures must pass through the platysma. This layer's involvement helps clinicians assess the severity and depth of neck trauma during evaluation.
MCQ 4 (Clinical):
A deep stab wound in the neck penetrates the platysma. What is the next critical assessment?
a) Check for vascular injury
b) Assess skin laceration only
c) Examine for hairline fracture
d) Evaluate external auditory canal
Answer & Explanation:
Correct answer: a) Check for vascular injury.
Explanation: Once the platysma is breached in neck trauma, there is a high risk of injury to vital structures such as the carotid artery and jugular vein. Immediate assessment for vascular injury is essential to prevent exsanguination or air embolism, requiring rapid imaging or surgical exploration.
MCQ 5:
The longus colli muscle lies in relation to which structure?
a) Anterior to vertebral bodies
b) Posterior to sternocleidomastoid
c) Lateral to carotid sheath
d) Superficial to platysma
Answer & Explanation:
Correct answer: a) Anterior to vertebral bodies.
Explanation: The longus colli is a deep muscle situated along the anterior surface of cervical vertebrae, stabilizing and flexing the neck. Injury to this muscle is rare due to its deep position, but its anatomical relation to vertebrae is important in cervical spine trauma or surgery.
MCQ 6 (Clinical):
A patient presents with loss of neck flexion strength after trauma. Which muscle is likely injured?
a) Trapezius
b) Longus colli
c) Platysma
d) Sternocleidomastoid
Answer & Explanation:
Correct answer: b) Longus colli.
Explanation: The longus colli muscle plays a key role in cervical spine flexion and stabilization. Trauma to this deep neck structure may result in weakness during neck flexion and cervical instability. Diagnosis requires imaging studies, and management involves conservative or surgical treatment depending on severity.
MCQ 7:
The sternocleidomastoid inserts into which bony structure?
a) Clavicle and sternum
b) Temporal bone
c) Mandible
d) Hyoid bone
Answer & Explanation:
Correct answer: a) Clavicle and sternum.
Explanation: The sternocleidomastoid muscle originates from the manubrium of the sternum and the clavicle and inserts onto the mastoid process of the temporal bone. Its contraction results in head rotation and flexion, and injury may impair these movements, impacting neck function assessment.
MCQ 8 (Clinical):
In penetrating neck trauma, platysma penetration indicates?
a) Superficial wound
b) Deep injury potentially involving vital structures
c) Skin laceration only
d) Mandibular fracture
Answer & Explanation:
Correct answer: b) Deep injury potentially involving vital structures.
Explanation: Penetration of the platysma in neck trauma signifies a deep wound. Vital structures such as major blood vessels and the trachea may be involved, requiring immediate medical attention with imaging or surgical exploration to prevent life-threatening complications.
MCQ 9:
Which nerve innervates the sternocleidomastoid muscle?
a) Accessory nerve (CN XI)
b) Hypoglossal nerve (CN XII)
c) Vagus nerve (CN X)
d) Facial nerve (CN VII)
Answer & Explanation:
Correct answer: a) Accessory nerve (CN XI).
Explanation: The accessory nerve (CN XI) supplies motor innervation to the sternocleidomastoid and trapezius muscles. Injury to CN XI during surgical procedures in the neck can cause shoulder droop and impaired head rotation, making its anatomical course critical to avoid damage.
MCQ 10 (Clinical):
A patient has difficulty turning head to the opposite side after neck trauma. Which muscle is most likely affected?
a) Platysma
b) Sternocleidomastoid
c) Trapezius
d) Longus colli
Answer & Explanation:
Correct answer: b) Sternocleidomastoid.
Explanation: The sternocleidomastoid muscle contracts to rotate the head to the opposite side. Injury to this muscle or its nerve (accessory nerve) following trauma can cause reduced head rotation strength, highlighting the need for careful evaluation during physical and neurological examination of neck injuries.
Chapter: Anatomy
Topic: Ear Anatomy
Subtopic: Chorda Tympani
Keywords:
Chorda Tympani: A branch of the facial nerve that carries taste sensations and parasympathetic fibers.
Middle Ear: Air-filled cavity in the temporal bone containing auditory ossicles.
Inner Ear: Contains cochlea and vestibular apparatus, responsible for hearing and balance.
External Auditory Canal: Tube running from the outer ear to the tympanic membrane (eardrum).
Lead Question - 2013:
Chorda tympani is a part of?
a) Middle ear
b) Inner ear
c) External auditory canal
d) None of the above
Answer & Explanation:
Correct answer: a) Middle ear.
Explanation: The chorda tympani is a branch of the facial nerve (CN VII) that traverses the middle ear cavity. It carries taste fibers from the anterior two-thirds of the tongue and parasympathetic fibers to salivary glands. Its anatomical course makes it vulnerable during middle ear surgeries, potentially causing taste disturbances.
MCQ 1:
The chorda tympani carries which type of fibers?
a) Motor fibers only
b) Sensory fibers only
c) Taste and parasympathetic fibers
d) Somatic sensory fibers
Answer & Explanation:
Correct answer: c) Taste and parasympathetic fibers.
Explanation: The chorda tympani carries taste fibers from the anterior two-thirds of the tongue and parasympathetic fibers to the submandibular and sublingual salivary glands. Understanding its anatomy is essential during middle ear surgeries to prevent postoperative taste disorders or xerostomia.
MCQ 2:
Chorda tympani is a branch of which cranial nerve?
a) Trigeminal nerve (CN V)
b) Facial nerve (CN VII)
c) Glossopharyngeal nerve (CN IX)
d) Vagus nerve (CN X)
Answer & Explanation:
Correct answer: b) Facial nerve (CN VII).
Explanation: The chorda tympani branches from the facial nerve and carries taste sensations and parasympathetic fibers. Damage during middle ear surgery can cause loss of taste in the anterior tongue and reduced salivation, emphasizing the need for careful surgical technique in otologic procedures.
MCQ 3:
Which structure does the chorda tympani pass through?
a) Internal auditory canal
b) Middle ear cavity
c) External auditory canal
d) Cochlear duct
Answer & Explanation:
Correct answer: b) Middle ear cavity.
Explanation: The chorda tympani runs through the middle ear cavity between the malleus and incus. It is susceptible to injury during middle ear surgeries, such as tympanoplasty or mastoidectomy, which may result in taste disturbances or dry mouth due to loss of parasympathetic innervation.
MCQ 4 (Clinical):
A patient complains of loss of taste in the anterior two-thirds of the tongue after ear surgery. Which nerve is likely injured?
a) Glossopharyngeal nerve
b) Hypoglossal nerve
c) Chorda tympani
d) Vagus nerve
Answer & Explanation:
Correct answer: c) Chorda tympani.
Explanation: The chorda tympani carries taste sensation from the anterior two-thirds of the tongue. Injury during middle ear procedures leads to loss of taste in that region. Recognizing this risk allows surgeons to plan safer approaches, preserving nerve integrity and reducing postoperative complications.
MCQ 5:
The parasympathetic fibers of the chorda tympani innervate which glands?
a) Parotid gland
b) Submandibular and sublingual glands
c) Lacrimal gland
d) Thyroid gland
Answer & Explanation:
Correct answer: b) Submandibular and sublingual glands.
Explanation: The chorda tympani carries parasympathetic fibers to the submandibular and sublingual salivary glands, facilitating saliva production. Disruption of these fibers during ear surgeries can cause xerostomia, impacting oral health and digestion, making anatomical knowledge vital for clinicians.
MCQ 6 (Clinical):
A patient presents with dry mouth and loss of taste in the anterior tongue post ear infection. Which nerve is affected?
a) Hypoglossal nerve
b) Chorda tympani
c) Glossopharyngeal nerve
d) Mandibular nerve
Answer & Explanation:
Correct answer: b) Chorda tympani.
Explanation: An ear infection involving the middle ear can inflame or damage the chorda tympani, leading to loss of taste in the anterior two-thirds of the tongue and decreased salivation from submandibular and sublingual glands. Recognizing this aids in targeted treatment to restore function.
MCQ 7:
Which other cranial nerve contributes to taste sensation besides the chorda tympani?
a) Trigeminal nerve
b) Glossopharyngeal nerve
c) Vagus nerve
d) Hypoglossal nerve
Answer & Explanation:
Correct answer: b) Glossopharyngeal nerve.
Explanation: The glossopharyngeal nerve (CN IX) supplies taste sensation to the posterior one-third of the tongue, while the chorda tympani supplies the anterior two-thirds. Damage to either can result in ageusia, so differentiating these pathways is essential in clinical diagnosis of taste disturbances.
MCQ 8 (Clinical):
During middle ear surgery, which structure is at risk if not carefully identified?
a) Eustachian tube
b) Chorda tympani
c) Tympanic membrane
d) Semicircular canals
Answer & Explanation:
Correct answer: b) Chorda tympani.
Explanation: The chorda tympani runs through the middle ear cavity and is at risk during surgeries like tympanoplasty. Accidental injury can cause loss of taste in the anterior tongue and reduced salivary flow, necessitating meticulous surgical technique to preserve nerve integrity and prevent postoperative complications.
MCQ 9:
The chorda tympani merges with which nerve to reach the tongue?
a) Lingual nerve
b) Hypoglossal nerve
c) Glossopharyngeal nerve
d) Facial nerve trunk
Answer & Explanation:
Correct answer: a) Lingual nerve.
Explanation: The chorda tympani joins the lingual nerve, a branch of the mandibular nerve (V3), to carry taste sensations and parasympathetic fibers to the anterior two-thirds of the tongue. Understanding this anatomical relationship helps in avoiding nerve injury during oral or otologic surgeries.
MCQ 10 (Clinical):
A patient reports altered taste and dry mouth after facial nerve palsy. Which branch is most likely affected?
a) Temporal branch
b) Zygomatic branch
c) Chorda tympani
d) Buccal branch
Answer & Explanation:
Correct answer: c) Chorda tympani.
Explanation: In facial nerve palsy, the chorda tympani may be affected, causing loss of taste in the anterior two-thirds of the tongue and decreased salivation from submandibular and sublingual glands. This emphasizes the importance of preserving nerve branches during facial surgeries to maintain function.
Chapter: Anatomy
Topic: Skull and Nasal Cavity
Subtopic: Olfactory Region
Keywords:
Olfactory Region: The superior part of the nasal cavity responsible for smell perception.
Nasal Bone: Paired bones forming the bridge of the nose.
Cribriform Plate of Ethmoid: Horizontal plate of the ethmoid bone containing foramina for olfactory nerve fibers.
Sphenoid Bone: Bone located at the base of the skull, behind the ethmoid bone.
Temporal Bone: Bone forming part of the side and base of the skull.
Lead Question - 2013:
The roof of the olfactory region is formed by?
a) Nasal bone
b) Cribriform plate of ethmoid
c) Sphenoid
d) Temporal bone
Answer & Explanation:
Correct answer: b) Cribriform plate of ethmoid.
Explanation: The cribriform plate of the ethmoid bone forms the roof of the olfactory region. It has multiple small foramina that allow passage of olfactory nerve fibers into the nasal cavity. Its fragile nature makes it a potential site for cerebrospinal fluid leakage in trauma cases.
MCQ 1:
Which structure transmits olfactory nerve fibers into the nasal cavity?
a) Foramen magnum
b) Cribriform plate of ethmoid
c) Jugular foramen
d) Optic canal
Answer & Explanation:
Correct answer: b) Cribriform plate of ethmoid.
Explanation: The cribriform plate of the ethmoid bone contains tiny foramina allowing passage of olfactory nerve fibers from the nasal cavity to the olfactory bulb. This anatomical pathway is crucial for the sense of smell and is clinically important as it can be disrupted in skull base fractures.
MCQ 2:
Which nerve is responsible for the sense of smell?
a) Trigeminal nerve
b) Olfactory nerve
c) Facial nerve
d) Glossopharyngeal nerve
Answer & Explanation:
Correct answer: b) Olfactory nerve.
Explanation: The olfactory nerve (CN I) is responsible for transmitting smell sensations from the nasal mucosa to the olfactory bulb. Its fibers pass through the cribriform plate. Damage to this nerve leads to anosmia, which is the loss of the sense of smell, often following head trauma.
MCQ 3:
The cribriform plate is part of which bone?
a) Frontal bone
b) Sphenoid bone
c) Ethmoid bone
d) Occipital bone
Answer & Explanation:
Correct answer: c) Ethmoid bone.
Explanation: The cribriform plate is a horizontal part of the ethmoid bone. It forms the roof of the nasal cavity and supports the olfactory bulb. The small perforations allow the olfactory nerve fibers to pass, and any injury here can lead to cerebrospinal fluid leaks.
MCQ 4 (Clinical):
A patient presents with anosmia following head trauma. Which structure is most likely injured?
a) Cribriform plate
b) Nasal septum
c) Sphenoid sinus
d) Frontal sinus
Answer & Explanation:
Correct answer: a) Cribriform plate.
Explanation: Trauma to the cribriform plate of the ethmoid bone may damage the olfactory nerve fibers passing through its foramina, leading to anosmia. Early identification of this injury is crucial for managing CSF leaks and preventing complications such as meningitis.
MCQ 5:
Which bone does NOT contribute to the nasal cavity structure?
a) Nasal bone
b) Maxilla
c) Temporal bone
d) Ethmoid bone
Answer & Explanation:
Correct answer: c) Temporal bone.
Explanation: The temporal bone does not contribute to the structure of the nasal cavity. Instead, it forms part of the lateral skull base. The nasal bone, maxilla, and ethmoid bone collectively form the framework of the nasal cavity, supporting nasal structures and passage of nerves and vessels.
MCQ 6 (Clinical):
CSF rhinorrhea following head injury suggests a breach in which anatomical structure?
a) Nasal bone
b) Cribriform plate
c) Sphenoid bone
d) Maxillary sinus
Answer & Explanation:
Correct answer: b) Cribriform plate.
Explanation: A fracture of the cribriform plate of the ethmoid bone can lead to cerebrospinal fluid leakage into the nasal cavity, manifesting as CSF rhinorrhea. This condition increases the risk of meningitis and requires surgical repair for the breach to prevent complications.
MCQ 7:
Which of the following is a clinical test for olfactory nerve function?
a) Finger-nose test
b) Smell identification test
c) Pupillary light reflex
d) Gag reflex
Answer & Explanation:
Correct answer: b) Smell identification test.
Explanation: The smell identification test assesses olfactory nerve function by having patients identify familiar odors. This helps diagnose anosmia or hyposmia, which may be caused by trauma to the cribriform plate, neurodegenerative diseases, or infections.
MCQ 8 (Clinical):
A patient with a tumor at the cribriform plate may present with which symptom?
a) Hearing loss
b) Loss of smell
c) Double vision
d) Facial muscle weakness
Answer & Explanation:
Correct answer: b) Loss of smell.
Explanation: A tumor at the cribriform plate can compress the olfactory nerve, leading to anosmia. Such lesions are clinically significant as they may go unnoticed until advanced stages. Early detection is vital to prevent further neurological deficits and enable appropriate surgical management.
MCQ 9:
Which nerve fiber type passes through the cribriform plate?
a) Motor fibers
b) Sensory fibers
c) Mixed fibers
d) Autonomic fibers
Answer & Explanation:
Correct answer: b) Sensory fibers.
Explanation: The olfactory nerve consists of sensory fibers that transmit smell information from the nasal mucosa to the olfactory bulb through the cribriform plate. This pathway is critical for olfaction, and damage results in loss of smell perception, a common issue after head trauma.
MCQ 10 (Clinical):
A patient with anosmia and clear nasal discharge likely has which condition?
a) Chronic sinusitis
b) Allergic rhinitis
c) CSF rhinorrhea
d) Deviated nasal septum
Answer & Explanation:
Correct answer: c) CSF rhinorrhea.
Explanation: Anosmia accompanied by clear, watery nasal discharge suggests CSF rhinorrhea due to cribriform plate injury. This condition poses infection risks like meningitis. Identifying the origin of the leak using beta-2 transferrin testing is essential for proper surgical management and preventing complications.
.
Chapter: Anatomy
Topic: Neck Anatomy
Subtopic: Occipital Triangle Contents
Keywords:
Occipital Triangle: A subdivision of the posterior triangle of the neck containing nerves and vessels.
Great Auricular Nerve: A sensory nerve from the cervical plexus that supplies the skin over the parotid gland and auricle.
Suprascapular Nerve: A nerve from the upper trunk of the brachial plexus providing motor function to the supraspinatus and infraspinatus muscles.
Lesser Occipital Nerve: A sensory nerve providing sensation to the skin of the scalp posterior and superior to the ear.
Occipital Artery: A branch of the external carotid artery supplying the posterior scalp.
Lead Question - 2013:
All are contents of occipital triangle except:
a) Great auricular nerve
b) Suprascapular nerve
c) Lesser occipital nerve
d) Occipital artery
Answer & Explanation:
Correct answer: a) Great auricular nerve.
Explanation: The occipital triangle typically contains the accessory nerve, suprascapular nerve, lesser occipital nerve, and occipital artery. The great auricular nerve is found in the submandibular region. Recognizing anatomical contents is critical for surgical approaches and avoiding nerve injuries during neck surgeries.
MCQ 1:
Which nerve passes through the occipital triangle?
a) Hypoglossal nerve
b) Accessory nerve
c) Vagus nerve
d) Mandibular nerve
Answer & Explanation:
Correct answer: b) Accessory nerve.
Explanation: The accessory nerve (CN XI) traverses the occipital triangle, providing motor innervation to the sternocleidomastoid and trapezius muscles. This anatomical feature is clinically important during procedures in the neck region to prevent iatrogenic nerve injury and consequent shoulder dysfunction.
MCQ 2:
The suprascapular nerve arises from which part of the brachial plexus?
a) Lateral cord
b) Posterior cord
c) Upper trunk
d) Medial cord
Answer & Explanation:
Correct answer: c) Upper trunk.
Explanation: The suprascapular nerve originates from the upper trunk of the brachial plexus. It passes through the occipital triangle to innervate the supraspinatus and infraspinatus muscles. Injury to this nerve may result in weakness of shoulder abduction and external rotation, affecting daily activities.
MCQ 3:
The occipital artery is a branch of which artery?
a) Internal carotid artery
b) Subclavian artery
c) External carotid artery
d) Vertebral artery
Answer & Explanation:
Correct answer: c) External carotid artery.
Explanation: The occipital artery branches from the external carotid artery, supplying blood to the posterior scalp. In the occipital triangle, its anatomical course is relevant during surgical procedures to prevent inadvertent bleeding and ensure adequate blood supply post-intervention.
MCQ 4 (Clinical):
A patient presents with drooping shoulder after lymph node biopsy in the neck. Which nerve is most likely injured?
a) Suprascapular nerve
b) Accessory nerve
c) Lesser occipital nerve
d) Great auricular nerve
Answer & Explanation:
Correct answer: b) Accessory nerve.
Explanation: The accessory nerve supplies the sternocleidomastoid and trapezius muscles. Injury during lymph node biopsy in the posterior triangle can lead to shoulder drooping and limited arm abduction. Knowledge of its course in the occipital triangle is vital to prevent surgical complications and ensure functional preservation.
MCQ 5:
The lesser occipital nerve provides sensory innervation to:
a) Anterior neck skin
b) Posterior scalp above the ear
c) Lower face
d) Suprascapular region
Answer & Explanation:
Correct answer: b) Posterior scalp above the ear.
Explanation: The lesser occipital nerve arises from the cervical plexus and supplies the skin posterior and superior to the ear. Damage can result in sensory loss or neuralgia. Clinicians should understand this anatomy for accurate diagnosis of cervical neuralgia and during neck dissections.
MCQ 6 (Clinical):
Following neck trauma, a patient has loss of sensation behind the ear. Which nerve is likely affected?
a) Great auricular nerve
b) Lesser occipital nerve
c) Accessory nerve
d) Hypoglossal nerve
Answer & Explanation:
Correct answer: b) Lesser occipital nerve.
Explanation: The lesser occipital nerve provides sensation to the scalp posterior and superior to the ear. Trauma in the occipital triangle may damage this nerve, leading to localized sensory deficits. Early identification helps guide management and prevents chronic neuralgia complications.
MCQ 7:
Which structure is NOT a content of the occipital triangle?
a) Accessory nerve
b) Suprascapular artery
c) Brachial plexus trunks
d) Vagus nerve
Answer & Explanation:
Correct answer: d) Vagus nerve.
Explanation: The vagus nerve does not pass through the occipital triangle; it exits the skull through the jugular foramen. The accessory nerve, suprascapular artery, and brachial plexus trunks are present in the occipital triangle, knowledge crucial for surgical and diagnostic procedures in the cervical region.
MCQ 8 (Clinical):
During radical neck dissection, which nerve must be preserved to maintain shoulder function?
a) Accessory nerve
b) Vagus nerve
c) Hypoglossal nerve
d) Suprascapular nerve
Answer & Explanation:
Correct answer: a) Accessory nerve.
Explanation: The accessory nerve innervates the trapezius and sternocleidomastoid muscles, critical for shoulder elevation and head rotation. Preserving it during radical neck dissection prevents postoperative shoulder dysfunction. This highlights the importance of detailed anatomical knowledge to reduce surgical morbidity.
MCQ 9:
The great auricular nerve primarily supplies sensation to:
a) Scalp
b) Parotid region and area over the mastoid
c) Anterior neck
d) Upper limb
Answer & Explanation:
Correct answer: b) Parotid region and area over the mastoid.
Explanation: The great auricular nerve, from the cervical plexus, provides sensory innervation to the skin over the parotid gland, mastoid process, and ear. It does not pass through the occipital triangle but is relevant in surgical landmarks for neck procedures.
MCQ 10 (Clinical):
A patient reports a burning sensation over the scalp behind the ear. Which nerve is implicated?
a) Great auricular nerve
b) Accessory nerve
c) Lesser occipital nerve
d) Suprascapular nerve
Answer & Explanation:
Correct answer: c) Lesser occipital nerve.
Explanation: A burning scalp sensation behind the ear is characteristic of lesser occipital neuralgia. The nerve’s anatomical path in the occipital triangle makes it susceptible to compression or trauma. Recognizing this helps clinicians in managing neuralgic pain effectively.
Chapter: Anatomy
Topic: Thorax
Subtopic: Trachea and Carina
Keyword Definitions:
Carina: Ridge of cartilage at the bifurcation of trachea into right and left main bronchi.
Trachea: Windpipe connecting larynx to bronchi, conducting air to lungs.
Thoracic vertebrae: 12 vertebrae in mid-back, important landmarks for mediastinal structures.
Clinical relevance: Carina is highly sensitive; foreign bodies contacting it trigger cough reflex. Important in bronchoscopy.
Superior mediastinum: Upper compartment of mediastinum, contains trachea, great vessels, and thymus.
Middle mediastinum: Contains heart, pericardium, main bronchi, and part of tracheal bifurcation.
Lead Question - 2013
Carina is situated at which level ?
a) T3
b) T4
c) T6
d) T9
Explanation: The carina, the bifurcation point of the trachea into right and left main bronchi, is located at the level of T4 vertebra, at the sternal angle. It is a crucial landmark in bronchoscopy and chest imaging. Correct answer is b) T4.
Guessed Question 2
The trachea bifurcates into right and left main bronchi at which anatomical landmark?
a) Sternal angle
b) Xiphoid process
c) Clavicle
d) Manubrium
Explanation: Trachea bifurcation at the carina occurs at the level of the sternal angle (Angle of Louis), corresponding to T4 vertebra. This landmark aids in thoracic procedures and radiology. Correct answer is a) Sternal angle.
Guessed Question 3
Which bronchus is more vertical and prone to aspiration?
a) Right main bronchus
b) Left main bronchus
c) Both equal
d) Segmental bronchi
Explanation: The right main bronchus is wider, shorter, and more vertical, making it a common site for aspirated foreign bodies. Correct answer is a) Right main bronchus.
Guessed Question 4
Carina is lined by which type of epithelium?
a) Stratified squamous
b) Pseudostratified ciliated columnar
c) Simple cuboidal
d) Transitional
Explanation: The carina, like the trachea, is lined by pseudostratified ciliated columnar epithelium, aiding mucus clearance. It contains abundant sensory receptors for cough reflex. Correct answer is b) Pseudostratified ciliated columnar.
Guessed Question 5
Which mediastinal compartment contains the carina?
a) Anterior
b) Middle
c) Posterior
d) Superior
Explanation: The carina lies in the middle mediastinum at the tracheal bifurcation, posterior to the ascending aorta and anterior to esophagus. Correct answer is b) Middle.
Guessed Question 6
Enlarged carinal lymph nodes affect which structure?
a) Trachea
b) Main bronchi
c) Both trachea and bronchi
d) Esophagus only
Explanation: Carinal lymph nodes surround the tracheal bifurcation; enlargement can compress both trachea and main bronchi, causing cough, dyspnea, or atelectasis. Correct answer is c) Both trachea and bronchi.
Guessed Question 7
During bronchoscopy, touching the carina triggers?
a) Sneezing
b) Cough reflex
c) Gag reflex
d) Bradycardia
Explanation: Carina has high density of sensory receptors; contact during bronchoscopy triggers a strong cough reflex, protecting lower airways. Correct answer is b) Cough reflex.
Guessed Question 8
Carina level in relation to sternal angle is?
a) At same level
b) Above
c) Below
d) At xiphoid
Explanation: The carina is at the level of the sternal angle (Angle of Louis) and T4 vertebra, serving as a surface landmark for tracheal bifurcation in imaging and procedures. Correct answer is a) At same level.
Guessed Question 9
Which artery is closely related to carina in posterior mediastinum?
a) Ascending aorta
b) Right pulmonary artery
c) Descending aorta
d) Superior vena cava
Explanation: The right pulmonary artery passes horizontally behind the ascending aorta and anterior to the tracheal bifurcation near the carina, significant in imaging and surgery. Correct answer is b) Right pulmonary artery.
Guessed Question 10
Carina is at which thoracic vertebral level during expiration?
a) T2
b) T4
c) T6
d) T8
Explanation: In adults, the carina is typically at T4–T5 vertebral level at sternal angle during normal respiration. Slight movement occurs with inspiration and expiration. Correct answer is b) T4.
Guessed Question 11
Clinical importance of carina in imaging is?
a) Landmark for mediastinal shift
b) Landmark for tracheal intubation
c) Guide for central line
d) Landmark for thoracotomy only
Explanation: The carina is a key radiological landmark for tracheal bifurcation, detecting mediastinal shift, and guiding endotracheal tube placement during intubation. Correct answer is b) Landmark for tracheal intubation.
Chapter: Anatomy
Topic: Thorax
Subtopic: Thoracic Duct
Keyword Definitions:
Thoracic duct: Main lymphatic channel draining most of the body except right upper quadrant.
Right lymphatic duct: Drains right upper limb, right thorax, and right head and neck.
Cisterna chyli: Dilated sac at L1–L2 origin of thoracic duct.
Mediastinum: Central thoracic compartment containing heart, vessels, and ducts.
Lymphatic drainage: Essential for immune surveillance and fluid balance.
Clinical relevance: Thoracic duct injury can cause chylothorax after surgery or trauma.
Lead Question - 2013
Thoracic duct does not drains ?
a) Right upper part of body
b) Left upper part of body
c) Right lower part of body
d) Left lower part of body
Explanation: The thoracic duct drains lymph from both lower limbs, abdomen, left thorax, left upper limb, and left side of head and neck. It does not drain the right upper quadrant of the body, which is drained by the right lymphatic duct. Correct answer is a) Right upper part of body.
Guessed Question 2
Thoracic duct opens into?
a) Right venous angle
b) Left venous angle
c) Right atrium
d) Superior vena cava
Explanation: The thoracic duct terminates at the left venous angle, the junction of the left internal jugular and left subclavian veins, returning lymph to venous circulation. Correct answer is b) Left venous angle.
Guessed Question 3
Origin of thoracic duct is from?
a) Cisterna chyli
b) Right lymphatic duct
c) Thoracoabdominal artery
d) Azygous vein
Explanation: The thoracic duct begins at the cisterna chyli, a dilated sac at the level of L1–L2, formed by intestinal and lumbar lymph trunks. Correct answer is a) Cisterna chyli.
Guessed Question 4
Which structure lies to the right of thoracic duct in thorax?
a) Descending aorta
b) Azygous vein
c) Esophagus
d) Left subclavian vein
Explanation: In most of the thoracic course, the thoracic duct is flanked by the azygous vein on the right and descending aorta on the left, with the esophagus anterior. Correct answer is b) Azygous vein.
Guessed Question 5
Injury to thoracic duct causes?
a) Pneumothorax
b) Chylothorax
c) Hemothorax
d) Hydrothorax
Explanation: Damage to the thoracic duct during surgery or trauma leads to leakage of lymph (chyle) into the pleural cavity, producing chylothorax, which causes respiratory distress and fluid loss. Correct answer is b) Chylothorax.
Guessed Question 6
Thoracic duct passes through diaphragm via?
a) Caval opening
b) Esophageal opening
c) Aortic opening
d) Foramen ovale
Explanation: The thoracic duct enters thorax through the aortic opening of the diaphragm, along with the descending aorta and azygous vein, at T12 level. Correct answer is c) Aortic opening.
Guessed Question 7
Which lymphatic trunk drains lower limb?
a) Jugular trunk
b) Subclavian trunk
c) Lumbar trunk
d) Bronchomediastinal trunk
Explanation: The lumbar lymphatic trunks drain the lower limbs, pelvis, and kidneys, and they join to form the cisterna chyli, the origin of thoracic duct. Correct answer is c) Lumbar trunk.
Guessed Question 8
Thoracic duct lies in which mediastinum for most of its course?
a) Anterior
b) Middle
c) Posterior
d) Superior
Explanation: The thoracic duct ascends through the posterior mediastinum between the aorta, azygous vein, and esophagus, before arching to the left in the superior mediastinum. Correct answer is c) Posterior.
Guessed Question 9
Thoracic duct arches to left at the level of?
a) T2
b) T4–T5
c) T8
d) T10
Explanation: The thoracic duct crosses from right to left side at the T4–T5 vertebral level, behind the esophagus, before its termination at the left venous angle. Correct answer is b) T4–T5.
Guessed Question 10
Which of the following is not drained by thoracic duct?
a) Abdomen
b) Both lower limbs
c) Left head and neck
d) Right upper limb
Explanation: Right upper limb drains into right lymphatic duct, not thoracic duct. The thoracic duct drains abdomen, lower limbs, left head and neck, left thorax, and left upper limb. Correct answer is d) Right upper limb.
Guessed Question 11
During esophageal carcinoma surgery, thoracic duct is at risk in which region?
a) Posterior mediastinum
b) Anterior mediastinum
c) Pericardial sac
d) Caval hiatus
Explanation: Esophageal mobilization in posterior mediastinum places thoracic duct at risk of injury, potentially causing postoperative chylothorax. Correct answer is a) Posterior mediastinum.
C
Chapter: Anatomy
Topic: Thorax
Subtopic: Heart Surfaces and Relations
Keyword Definitions:
Base of heart: Posterior surface formed mainly by left atrium, partly by right atrium.
Atrium: Upper chamber of the heart; right atrium receives venous blood, left atrium receives pulmonary venous blood.
Ventricle: Lower chamber of the heart; right ventricle pumps to lungs, left ventricle to systemic circulation.
Posterior mediastinum: Part of thorax behind the heart containing esophagus and great vessels.
Clinical relevance: Knowledge of cardiac surfaces is essential in imaging, echocardiography, and surgical approaches.
Lead Question - 2013
Posterior surface of heart is formed by -
a) RA
b) LA
c) LV
d) RV
Explanation: The posterior surface of the heart, also called the base, is formed mainly by the left atrium, which receives pulmonary veins. The right atrium contributes slightly. Thus, the correct answer is b) LA.
Guessed Question 2
The diaphragmatic surface of the heart is formed mainly by?
a) Right atrium
b) Right ventricle
c) Left ventricle
d) Left atrium
Explanation: The diaphragmatic surface of the heart rests on the central tendon of the diaphragm and is formed mainly by the left ventricle with a small contribution from the right ventricle. The correct answer is c) Left ventricle.
Guessed Question 3
The right border of the heart is formed by?
a) Right atrium
b) Right ventricle
c) Left atrium
d) Left ventricle
Explanation: The right border of the heart seen on chest X-ray is formed by the right atrium, which receives venous blood from the SVC and IVC. The correct answer is a) Right atrium.
Guessed Question 4
The apex of the heart is formed by?
a) Right ventricle
b) Left ventricle
c) Left atrium
d) Right atrium
Explanation: The apex beat, palpable in the left 5th intercostal space, is formed by the tip of the left ventricle. The correct answer is b) Left ventricle.
Guessed Question 5
Which chamber of the heart lies closest to the esophagus?
a) Right atrium
b) Left atrium
c) Right ventricle
d) Left ventricle
Explanation: The left atrium forms the posterior surface of the heart and lies directly in front of the esophagus in the posterior mediastinum. The correct answer is b) Left atrium.
Guessed Question 6
On a lateral chest X-ray, the posterior border of the heart is formed by?
a) Right ventricle
b) Left ventricle
c) Left atrium
d) Right atrium
Explanation: In lateral chest radiographs, the posterior border of the heart is formed by the left atrium and left ventricle. The correct answer is c) Left atrium.
Guessed Question 7
Which chamber enlargement causes dysphagia by compressing esophagus?
a) Right atrium
b) Left atrium
c) Left ventricle
d) Right ventricle
Explanation: Enlargement of the left atrium can compress the esophagus, leading to dysphagia. This is often seen in mitral stenosis. The correct answer is b) Left atrium.
Guessed Question 8
The anterior surface of the heart is mainly formed by?
a) Left atrium
b) Right ventricle
c) Left ventricle
d) Right atrium
Explanation: The sternocostal or anterior surface of the heart is formed mainly by the right ventricle. The correct answer is b) Right ventricle.
Guessed Question 9
Which chamber of the heart contributes most to the left border in chest X-ray?
a) Right ventricle
b) Left ventricle
c) Right atrium
d) Left atrium
Explanation: The left border of the cardiac shadow on chest X-ray is formed mainly by the left ventricle. The correct answer is b) Left ventricle.
Guessed Question 10
Which cardiac chamber is most posterior in location?
a) Right atrium
b) Right ventricle
c) Left atrium
d) Left ventricle
Explanation: The left atrium forms the base of the heart and is the most posteriorly placed chamber. The correct answer is c) Left atrium.
Guessed Question 11
Atrial fibrillation commonly originates due to abnormal conduction from which heart chamber?
a) Right atrium
b) Left atrium
c) Right ventricle
d) Left ventricle
Explanation: Atrial fibrillation often originates from ectopic foci near the openings of pulmonary veins in the left atrium. The correct answer is b) Left atrium.
Chapter: Anatomy
Topic: Thorax
Subtopic: Mediastinal Relations of Lungs
Keyword Definitions:
Mediastinal surface of lung: Surface facing the mediastinum, showing impressions of adjacent structures.
Right lung mediastinal relations: Superior vena cava, right atrium, azygos vein, esophagus.
Left lung mediastinal relations: Arch of aorta, left ventricle, thoracic aorta, left subclavian artery.
SVC (Superior Vena Cava): Large vein draining blood from upper half of body into right atrium.
Pulmonary trunk: Major vessel carrying deoxygenated blood from right ventricle to lungs.
Clinical relevance: Knowledge of lung relations is vital in thoracic surgery, mediastinal tumor excision, and radiology.
Lead Question - 2013
Which of the following is related to mediastinal part of right lung?
a) Arch of aorta
b) SVC
c) Pulmonary trunk
d) Left ventricle
Explanation: The mediastinal surface of the right lung is related to the superior vena cava, right atrium, and azygos vein. The left lung is related to arch of aorta and left ventricle. Thus, the correct answer is b) SVC.
Guessed Question 2
Which structure leaves a groove on the mediastinal surface of the left lung?
a) Superior vena cava
b) Arch of aorta
c) Right atrium
d) Azygos vein
Explanation: The arch of aorta produces a large concave impression on the mediastinal surface of the left lung. The correct answer is b) Arch of aorta.
Guessed Question 3
The cardiac impression on the right lung is mainly due to?
a) Right atrium
b) Left ventricle
c) Right ventricle
d) Left atrium
Explanation: The right atrium lies adjacent to the mediastinal surface of the right lung, producing the cardiac impression. The correct answer is a) Right atrium.
Guessed Question 4
The esophagus lies in relation to which lung?
a) Right lung
b) Left lung
c) Both lungs
d) Neither
Explanation: The esophagus runs along the posterior mediastinum and makes an impression on the mediastinal surface of the right lung. The correct answer is a) Right lung.
Guessed Question 5
Which lung shows a groove for azygos vein?
a) Right lung
b) Left lung
c) Both lungs
d) Neither
Explanation: The azygos vein arches over the root of the right lung before entering SVC, leaving a groove on the mediastinal surface of the right lung. The correct answer is a) Right lung.
Guessed Question 6
The pulmonary trunk lies in close relation with?
a) Right lung root
b) Left lung root
c) Both lung roots
d) Neither
Explanation: The pulmonary trunk arises from right ventricle and lies closer to the root of the left lung. The correct answer is b) Left lung root.
Guessed Question 7
Which part of the heart produces impression on the mediastinal surface of the left lung?
a) Right atrium
b) Left ventricle
c) Right ventricle
d) Left atrium
Explanation: The left ventricle lies adjacent to the mediastinal surface of the left lung, producing a cardiac impression. The correct answer is b) Left ventricle.
Guessed Question 8
In CT thorax, a groove on right lung for large vein near its root corresponds to?
a) Superior vena cava
b) Inferior vena cava
c) Right pulmonary vein
d) Azygos vein
Explanation: The superior vena cava lies adjacent to the upper part of the mediastinal surface of right lung, forming a groove. The correct answer is a) Superior vena cava.
Guessed Question 9
Which great vessel does NOT leave an impression on the right lung?
a) Superior vena cava
b) Azygos vein
c) Inferior vena cava
d) Arch of aorta
Explanation: The arch of aorta lies in relation to the left lung and does not leave an impression on the right lung. The correct answer is d) Arch of aorta.
Guessed Question 10
During right pneumonectomy, which mediastinal relation must be preserved to prevent venous obstruction?
a) Azygos vein
b) Thoracic duct
c) Arch of aorta
d) Left ventricle
Explanation: The azygos vein arches over the right lung root and drains into SVC. Damage during surgery can cause venous obstruction and swelling of upper body. The correct answer is a) Azygos vein.
Guessed Question 11
Which clinical feature is expected in tumor compressing SVC related to right lung?
a) Dysphagia
b) Hoarseness of voice
c) Swelling of face and upper limb
d) Hemoptysis
Explanation: Superior vena cava obstruction causes venous congestion of face, neck, and upper limb, leading to swelling and dilated veins. The correct answer is c) Swelling of face and upper limb.
Chapter: Anatomy
Topic: Thorax
Subtopic: Mediastinum and Esophagus
Keyword Definitions:
Esophagus: Muscular tube connecting pharynx to stomach, located in thorax and abdomen.
Mediastinum: Central compartment of thoracic cavity, divided into superior, anterior, middle, and posterior parts.
Posterior Mediastinum: Contains esophagus, descending thoracic aorta, thoracic duct, azygos system.
Anterior Mediastinum: Located in front of pericardium, contains thymus (in children), fat, lymph nodes.
Middle Mediastinum: Contains heart, pericardium, roots of great vessels, phrenic nerves.
Superior Mediastinum: Contains trachea, esophagus, great vessels, thymus remnants.
Clinical Relevance: Esophageal lesions may compress adjacent mediastinal structures like trachea or left atrium.
Lead Question - 2013
Esophagus is present in which mediastinum?
a) Anterior
b) Posterior
c) Middle
d) None
Explanation: The esophagus lies mainly in the posterior mediastinum, extending from superior mediastinum down to the diaphragm. It is absent from anterior and middle mediastinum. Therefore, the correct answer is b) Posterior.
Guessed Question 2
Which structure is located immediately posterior to the left atrium?
a) Trachea
b) Esophagus
c) Descending aorta
d) Pulmonary trunk
Explanation: The esophagus runs just posterior to the left atrium. Enlargement of the left atrium can compress the esophagus, leading to dysphagia. The correct answer is b) Esophagus.
Guessed Question 3
At what vertebral level does the esophagus pass through the diaphragm?
a) T8
b) T9
c) T10
d) T12
Explanation: The esophagus passes through the diaphragm at T10 vertebral level via the esophageal hiatus, accompanied by vagal trunks. The correct answer is c) T10.
Guessed Question 4
Which nerve forms the esophageal plexus in thorax?
a) Vagus nerve
b) Phrenic nerve
c) Intercostal nerve
d) Sympathetic chain
Explanation: The esophageal plexus is formed by vagus nerve fibers along with sympathetic fibers, facilitating autonomic innervation of esophagus. The correct answer is a) Vagus nerve.
Guessed Question 5
Which vein drains the thoracic esophagus?
a) Azygos vein
b) Hemiazygos vein
c) Accessory hemiazygos vein
d) All of the above
Explanation: The thoracic esophagus drains into azygos, hemiazygos, and accessory hemiazygos veins, forming porto-systemic anastomoses. The correct answer is d) All of the above.
Guessed Question 6
A patient with dysphagia lusoria has esophageal compression by?
a) Aberrant right subclavian artery
b) Left brachiocephalic vein
c) Descending aorta
d) Right pulmonary artery
Explanation: Dysphagia lusoria occurs when an aberrant right subclavian artery arises abnormally and compresses the esophagus posteriorly. The correct answer is a) Aberrant right subclavian artery.
Guessed Question 7
Which structure does the esophagus lie anterior to in posterior mediastinum?
a) Descending thoracic aorta
b) Trachea
c) Left atrium
d) Pulmonary trunk
Explanation: In the posterior mediastinum, the esophagus lies anterior to the descending thoracic aorta and thoracic duct. The correct answer is a) Descending thoracic aorta.
Guessed Question 8
Which clinical investigation best shows left atrial impression on esophagus?
a) Barium swallow
b) Chest X-ray
c) Ultrasound
d) CT thorax
Explanation: A barium swallow clearly shows indentations on esophagus caused by enlarged left atrium, a classic radiological sign. The correct answer is a) Barium swallow.
Guessed Question 9
Which artery provides blood supply to thoracic esophagus?
a) Bronchial arteries
b) Left gastric artery
c) Inferior phrenic arteries
d) All of the above
Explanation: Thoracic esophagus receives blood supply from bronchial arteries, branches of thoracic aorta, and inferior phrenic arteries. The correct answer is d) All of the above.
Guessed Question 10
Posterior mediastinum contains all except?
a) Esophagus
b) Thoracic duct
c) Azygos vein
d) Thymus
Explanation: The thymus is located in anterior mediastinum (prominent in children). Posterior mediastinum includes esophagus, azygos system, thoracic duct, descending aorta. The correct answer is d) Thymus.
Guessed Question 11
Which lymph nodes drain the thoracic esophagus?
a) Posterior mediastinal nodes
b) Paratracheal nodes
c) Left gastric nodes
d) All of the above
Explanation: Thoracic esophagus drains into posterior mediastinal and paratracheal nodes, and through abdominal continuation into left gastric nodes. The correct answer is d) All of the above.
Chapter: Anatomy
Topic: Thorax
Subtopic: Mediastinum and Esophagus
Keyword Definitions:
Esophagus: Muscular tube connecting pharynx to stomach, passes through thorax and diaphragm.
Mediastinum: Central compartment of thoracic cavity divided into superior, anterior, middle, and posterior parts.
Superior Mediastinum: Contains trachea, esophagus, great vessels, thymus remnants.
Middle Mediastinum: Contains heart, pericardium, phrenic nerves, and root of great vessels.
Posterior Mediastinum: Contains esophagus, thoracic duct, descending aorta, azygos system.
Anterior Mediastinum: Contains thymus (in children), fat, lymph nodes, connective tissue.
Clinical Relevance: Esophageal pathology may compress mediastinal structures causing dysphagia, chest pain, or dyspnea.
Lead Question - 2013
Esophagus is present in all except?
a) Superior mediastinum
b) Middle mediastinum
c) Anterior mediastinum
d) Posterior mediastinum
Explanation: The esophagus runs through superior and posterior mediastinum but not the anterior mediastinum, which mainly contains thymic tissue and fat. Middle mediastinum is primarily cardiac. Thus, the esophagus is absent from anterior mediastinum. The correct answer is c) Anterior mediastinum.
Guessed Question 2
Which structure lies immediately posterior to the left atrium in the thorax?
a) Descending thoracic aorta
b) Esophagus
c) Trachea
d) Pulmonary artery
Explanation: The esophagus lies immediately posterior to the left atrium. This close relation allows left atrial enlargement to compress the esophagus, producing dysphagia. The correct answer is b) Esophagus.
Guessed Question 3
Thoracic duct drains into venous system at the junction of?
a) Right internal jugular and subclavian vein
b) Left internal jugular and subclavian vein
c) Right brachiocephalic vein
d) Left brachiocephalic vein
Explanation: The thoracic duct ascends in the posterior mediastinum and drains into the venous system at the left internal jugular and left subclavian vein junction. The correct answer is b) Left internal jugular and subclavian vein.
Guessed Question 4
Which nerve runs anterior to the root of lung?
a) Vagus nerve
b) Phrenic nerve
c) Sympathetic trunk
d) Intercostal nerve
Explanation: The phrenic nerve runs anterior to the root of the lung within the middle mediastinum, supplying diaphragm motor innervation. The vagus nerve runs posterior to the root. The correct answer is b) Phrenic nerve.
Guessed Question 5
A patient with esophageal carcinoma shows hoarseness of voice. Which nerve is involved?
a) Right recurrent laryngeal nerve
b) Left recurrent laryngeal nerve
c) Glossopharyngeal nerve
d) Phrenic nerve
Explanation: The left recurrent laryngeal nerve, looping around the aortic arch in superior mediastinum, can be compressed by esophageal carcinoma, causing hoarseness. The correct answer is b) Left recurrent laryngeal nerve.
Guessed Question 6
At the level of T10 vertebra, the esophagus passes through?
a) Aortic hiatus
b) Caval opening
c) Esophageal hiatus
d) Foramen ovale
Explanation: The esophagus passes through diaphragm at T10 vertebral level via esophageal hiatus, accompanied by vagal trunks. The correct answer is c) Esophageal hiatus.
Guessed Question 7
Which structure passes through aortic hiatus of diaphragm at T12 level?
a) Thoracic duct
b) Esophagus
c) Inferior vena cava
d) Phrenic nerve
Explanation: The aortic hiatus at T12 transmits the descending aorta, thoracic duct, and azygos vein. The esophagus passes at T10, IVC at T8. The correct answer is a) Thoracic duct.
Guessed Question 8
Posterior mediastinum contains all except?
a) Thoracic duct
b) Azygos vein
c) Phrenic nerve
d) Esophagus
Explanation: Phrenic nerve is located in middle mediastinum, running anterior to lung roots. Posterior mediastinum includes esophagus, thoracic duct, azygos system, and descending aorta. The correct answer is c) Phrenic nerve.
Guessed Question 9
Compression of esophagus by enlarged left atrium is best seen on which investigation?
a) Barium swallow
b) Chest X-ray PA view
c) CT thorax
d) Ultrasound
Explanation: A barium swallow study shows indentation of esophagus by enlarged left atrium, producing characteristic posterior impression. The correct answer is a) Barium swallow.
Guessed Question 10
A patient with dysphagia lusoria has compression of esophagus by?
a) Aberrant right subclavian artery
b) Left common carotid artery
c) Pulmonary trunk
d) Inferior vena cava
Explanation: Dysphagia lusoria occurs when an aberrant right subclavian artery arises abnormally and compresses esophagus posteriorly, causing difficulty in swallowing. The correct answer is a) Aberrant right subclavian artery.
Guessed Question 11
Which vein drains blood from esophagus into systemic circulation?
a) Azygos vein
b) Hemiazygos vein
c) Accessory hemiazygos vein
d) All of the above
Explanation: Esophageal veins drain into azygos, hemiazygos, and accessory hemiazygos systems in posterior mediastinum, forming porto-systemic anastomosis with left gastric vein. The correct answer is d) All of the above.
Keyword Definitions
• Sacral promontory – The anterior projecting edge of the first sacral vertebra, used as an anatomical landmark.
• Aortic bifurcation – Occurs at the level of L4 vertebra, below the sacral promontory.
• Inferior mesenteric artery (IMA) – Arises from the abdominal aorta at L3, supplying the hindgut.
• Superior mesenteric artery (SMA) – Arises at L1, supplying the midgut.
• Presacral nerve – Autonomic plexus lying anterior to sacrum, not directly related to sacral promontory.
• Pelvic inlet – Boundary between abdominal and pelvic cavities, sacral promontory forms its posterior margin.
• True pelvis – Space below the pelvic brim, relevant for obstetric diameters.
• Obstetric conjugate – Distance from sacral promontory to pubic symphysis, important in labor assessment.
• Ureter crossing – Ureters cross pelvic brim near bifurcation of common iliac arteries close to sacral promontory.
• Clinical relevance – Used in obstetrics, surgery, and radiology as a key anatomical reference.
Chapter: Anatomy / Abdomen & Pelvis
Topic: Landmarks in Pelvis
Subtopic: Sacral Promontory and Related Structures
Lead Question – 2013
Sacral promontory is the landmark for
a) Origin of superior mesenteric artery
b) Termination of presacral nerve
c) Origin of inferior mesenteric artery
d) None of the above
Explanation: The sacral promontory marks the posterior boundary of the pelvic inlet. It is not the landmark for SMA (L1) or IMA (L3). Presacral nerve plexus lies anterior to sacrum but termination is not at the promontory. Correct answer: None of the above.
Guessed Questions for NEET PG
1) The sacral promontory contributes to measurement of which pelvic diameter?
a) Transverse diameter
b) Obstetric conjugate
c) Interspinous diameter
d) Diagonal conjugate
Explanation: The obstetric conjugate runs from the sacral promontory to the superior margin of the pubic symphysis, critical for assessing childbirth feasibility. Diagonal conjugate is measured clinically but relates to the same point. Correct answer: Obstetric conjugate.
2) The level of bifurcation of the abdominal aorta is?
a) L1
b) L2
c) L3
d) L4
Explanation: The abdominal aorta bifurcates into common iliac arteries at L4, slightly below the sacral promontory, an important landmark for surgeons and interventional radiologists. Correct answer: L4.
3) A surgeon during pelvic surgery uses sacral promontory to identify?
a) Start of rectum
b) Pelvic brim
c) Common iliac bifurcation
d) Superior mesenteric artery
Explanation: The sacral promontory is a key landmark forming the posterior boundary of the pelvic brim, helping orient pelvic surgery and dissection. Correct answer: Pelvic brim.
4) Inferior mesenteric artery arises at which vertebral level?
a) L1
b) L2
c) L3
d) L4
Explanation: The IMA arises at the level of L3, supplying descending colon, sigmoid, and upper rectum. It does not originate at the sacral promontory level. Correct answer: L3.
5) During obstetric assessment, the diagonal conjugate is measured from?
a) Sacral promontory to inferior pubic symphysis
b) Coccyx to pubic symphysis
c) Sacral promontory to ischial spine
d) Pubic tubercle to ischial tuberosity
Explanation: The diagonal conjugate is measured from the sacral promontory to the inferior border of the pubic symphysis, used clinically to estimate true conjugate. Correct answer: Sacral promontory to inferior pubic symphysis.
6) Which artery lies closest to the sacral promontory?
a) Inferior mesenteric artery
b) Middle sacral artery
c) Superior mesenteric artery
d) Internal iliac artery
Explanation: The middle sacral artery descends anterior to sacrum from near the aortic bifurcation and is closely related to the sacral promontory. Correct answer: Middle sacral artery.
7) In pelvic brim, the sacral promontory forms?
a) Anterior boundary
b) Posterior boundary
c) Lateral boundary
d) Inferior boundary
Explanation: The sacral promontory forms the posterior boundary of the pelvic brim, while the pubic symphysis forms the anterior limit. Correct answer: Posterior boundary.
8) Which clinical test involves palpating sacral promontory per vaginum?
a) Bimanual examination
b) Pelvic outlet test
c) Assessment of diagonal conjugate
d) Assessment of transverse diameter
Explanation: In gynecological examination, diagonal conjugate is estimated by palpating sacral promontory and measuring distance to pubic symphysis. Correct answer: Assessment of diagonal conjugate.
9) In pregnancy, an obstetric conjugate a) Cephalopelvic disproportion
b) Sacralization of lumbar vertebra
c) Lordosis
d) Normal pelvis
Explanation: Obstetric conjugate shorter than 10 cm indicates reduced pelvic inlet, leading to cephalopelvic disproportion and labor complications. Correct answer: Cephalopelvic disproportion.
10) Which nerve plexus lies just anterior to sacral promontory?
a) Lumbar plexus
b) Presacral plexus
c) Pudendal plexus
d) Coccygeal plexus
Explanation: The presacral (hypogastric) plexus lies anterior to sacrum and sacral promontory, providing autonomic supply to pelvic organs. Correct answer: Presacral plexus.
Keyword Definitions
• Hip flexion – Movement decreasing the angle between thigh and trunk.
• Iliopsoas – Primary hip flexor formed by psoas major and iliacus; inserts on lesser trochanter.
• Psoas major – Lumbar-origin muscle (T12–L5) flexing hip; innervated by L1–L3 (direct branches).
• Iliacus – Iliac fossa muscle joining psoas; femoral nerve (L2–L3).
• Rectus femoris – Biarticular quadriceps head; flexes hip, extends knee; origin AIIS.
• Sartorius – ASIS origin; flexes, abducts, laterally rotates hip; flexes knee.
• Tensor fasciae latae (TFL) – Assists hip flexion/abduction; superior gluteal nerve (L4–S1).
• Femoral nerve – L2–L4 nerve to anterior thigh flexors/extensors.
• Thomas test – Assesses fixed flexion deformity of hip/iliopsoas contracture.
• Psoas sign – Pain with resisted hip flexion/extension of hip suggesting psoas irritation (e.g., retrocecal appendicitis).
Chapter: Anatomy / Lower Limb
Topic: Muscles of the Hip
Subtopic: Flexors of the Hip Joint
Lead Question – 2013
Muscle causing flexion of hip ?
a) Biceps femoris
b) Psoas major
c) Gluteus maximus
d) TFL
Explanation: The prime mover for hip flexion is iliopsoas; among options, psoas major is the principal flexor. TFL assists but is not the main flexor. Biceps femoris mainly extends hip and flexes knee; gluteus maximus extends hip. Correct answer: Psoas major.
Guessed Questions for NEET PG
1) Primary insertion site of the main hip flexor complex is?
a) Greater trochanter
b) Lesser trochanter
c) Intertrochanteric line
d) Iliac crest
Explanation: Iliopsoas (psoas major + iliacus) inserts onto the lesser trochanter of the femur via a common tendon, providing a powerful flexion vector across the hip with some external rotation. Correct answer: Lesser trochanter.
2) Nerve supply of iliacus is via?
a) Obturator nerve
b) Femoral nerve
c) Sciatic nerve
d) Superior gluteal nerve
Explanation: Iliacus is innervated by the femoral nerve (L2–L3). In contrast, psoas major receives direct branches from the lumbar plexus (L1–L3). Understanding dual innervation patterns aids clinical localization of weakness. Correct answer: Femoral nerve.
3) A footballer with pain on resisted straight-leg raise; MRI shows AIIS avulsion. Weakness likely in?
a) Hip extension
b) Hip flexion
c) Knee flexion
d) Hip adduction
Explanation: AIIS avulsion implicates rectus femoris origin. Rectus femoris flexes the hip and extends the knee; acute injury produces painful/weak hip flexion on straight-leg raise. Correct answer: Hip flexion.
4) Thomas test detects contracture of which muscle group?
a) Hip abductors
b) Hip extensors
c) Hip flexors (iliopsoas)
d) Hip adductors
Explanation: Thomas test identifies fixed flexion deformity due to tightness/contracture of iliopsoas. The contralateral hip is maximally flexed; lumbar lordosis flattens; a rising contralateral thigh indicates flexor contracture. Correct answer: Hip flexors (iliopsoas).
5) Which muscle is NOT a primary hip flexor?
a) Sartorius
b) Pectineus
c) Gluteus medius
d) Rectus femoris
Explanation: Gluteus medius chiefly abducts and stabilizes pelvis; it is not a hip flexor. Sartorius and rectus femoris flex the hip; pectineus mainly adducts but contributes to flexion. Correct answer: Gluteus medius.
6) Superior gluteal nerve injury most reduces which assisting action at hip?
a) Flexion by TFL
b) Extension by gluteus maximus
c) Adduction by gracilis
d) External rotation by piriformis
Explanation: Superior gluteal nerve (L4–S1) supplies TFL, gluteus medius, and minimus. Injury impairs abduction and internal rotation and reduces TFL-assisted hip flexion. Correct answer: Flexion by TFL.
7) In femoral nerve palsy, which combined movement is most impaired?
a) Hip flexion with knee extension
b) Hip extension with knee flexion
c) Hip adduction with knee flexion
d) Hip abduction with knee extension
Explanation: Femoral nerve supplies iliacus and quadriceps (including rectus femoris). Palsy weakens hip flexion and abolishes knee extension, impairing sit-to-stand and stair climbing. Correct answer: Hip flexion with knee extension.
8) A patient with retrocecal appendicitis has pain on passive extension of right hip. Irritation involves?
a) Gluteus maximus
b) Psoas major
c) Adductor magnus
d) Obturator externus
Explanation: The psoas sign indicates psoas major irritation; stretching the inflamed muscle by hip extension elicits pain. This localizes retroperitoneal inflammation near the psoas. Correct answer: Psoas major.
9) Which root value chiefly powers hip flexion during straight-leg raise?
a) L1
b) L2–L3
c) L4–L5
d) S1–S2
Explanation: Hip flexion strength correlates with L2–L3 myotomes (iliopsoas). Testing resisted hip flexion helps localize radiculopathy to upper lumbar roots. Correct answer: L2–L3.
10) A sprinter has pain at ASIS with weakness in crossing legs. Most likely muscle injured?
a) Sartorius
b) Rectus femoris
c) Pectineus
d) Iliopsoas
Explanation: Sartorius originates at ASIS and flexes, abducts, and laterally rotates the hip—used to cross legs (“tailor’s muscle”). ASIS tenderness with these deficits suggests sartorius strain/avulsion. Correct answer: Sartorius.
Keyword Definitions
• Ossification center – Specific area where bone formation begins.
• Primary ossification center – Appears before birth, usually in diaphysis of long bones.
• Secondary ossification center – Appears after birth, mostly at epiphyses.
• Epiphysis – End part of long bone, separated by growth plate.
• Growth plate (physis) – Cartilaginous zone responsible for bone lengthening.
• Femur – Longest bone in the body, crucial for weight bearing.
• Lower end of femur – Includes medial and lateral condyles, important for knee joint stability.
• Clinical correlation – Injuries near growth plate may cause deformity in children.
• Fusion of ossification centers – Indicates skeletal maturity, useful in forensic medicine.
• Pathology – Delay in ossification may suggest rickets or endocrinological disorders.
Chapter: Anatomy / Lower Limb
Topic: Ossification of Femur
Subtopic: Lower End of Femur
Lead Question – 2013
Lower end of femur is ossified from how many ossification centers?
a) 1
b) 2
c) 3
d) 4
Explanation: The lower end of femur is ossified from a single secondary ossification center, which appears at birth and is the largest epiphyseal center in the body. It helps determine gestational age in newborn radiographs. Correct answer: 1.
Guessed Questions for NEET PG
1) The first secondary ossification center to appear in the body is?
a) Head of femur
b) Lower end of femur
c) Upper end of tibia
d) Calcaneus
Explanation: The lower end of femur and upper end of tibia are the earliest secondary ossification centers, both appearing at birth. These are crucial in neonatal skeletal age estimation. Correct answer: Lower end of femur.
2) Which ossification center is used to determine fetal maturity in X-rays?
a) Upper end of humerus
b) Lower end of femur
c) Upper end of fibula
d) Clavicle
Explanation: The presence of the ossification center in the lower end of femur indicates intrauterine maturity after 36 weeks of gestation. Correct answer: Lower end of femur.
3) The lower end of femur fuses with shaft at what age?
a) 12 years
b) 16 years
c) 20 years
d) 25 years
Explanation: Fusion of the lower end of femur with diaphysis occurs around 20 years of age, making it a reliable marker for skeletal maturity in forensic medicine. Correct answer: 20 years.
4) Which epiphysis is the largest secondary ossification center?
a) Proximal humerus
b) Distal femur
c) Proximal tibia
d) Iliac crest
Explanation: The distal femoral epiphysis is the largest secondary ossification center, covering the condylar region. It plays a crucial role in knee growth. Correct answer: Distal femur.
5) In rickets, which ossification center shows delayed appearance?
a) Lower end of femur
b) Upper end of tibia
c) Distal radius
d) All of the above
Explanation: Rickets causes generalized delay in appearance of secondary ossification centers, including lower femur, tibia, and wrist bones. Correct answer: All of the above.
6) The ossification center at the head of femur appears at?
a) Birth
b) 1 year
c) 3 months
d) 6 years
Explanation: The head of femur ossification center appears at around 1 year of age, useful in pediatric radiology. Correct answer: 1 year.
7) Which bone has both membranous and cartilaginous ossification?
a) Femur
b) Clavicle
c) Tibia
d) Radius
Explanation: The clavicle develops from both intramembranous and endochondral ossification, unlike femur which is purely cartilaginous in origin. Correct answer: Clavicle.
8) A neonate with no ossification center at lower femur is likely?
a) Term baby
b) Preterm baby
c) Post-term baby
d) Growth restricted baby only
Explanation: Absence of ossification center at the lower femur suggests prematurity (
9) Which of the following is true about epiphyseal injuries in femur?
a) Common in adults
b) Affect growth potential
c) Do not cause deformity
d) Heals without complications
Explanation: Epiphyseal injuries in the distal femur can affect growth and cause angular deformities due to damage of growth plate. Correct answer: Affect growth potential.
10) The nutrient artery of femur enters from?
a) Upper end
b) Middle third posterior surface
c) Lower end
d) Anterior surface
Explanation: The nutrient artery enters the shaft of femur from the middle third on its posterior surface, directed towards the knee ("to the elbow I go, from the knee I flee"). Correct answer: Middle third posterior surface.
Keyword Definitions
• Lymph nodes – Small immune structures filtering lymphatic fluid.
• Superficial inguinal lymph nodes – Drain superficial structures of lower limb, external genitalia, and lower abdominal wall.
• Deep inguinal nodes – Located beneath fascia lata, drain deep lymphatics of lower limb.
• External iliac nodes – Drain lymph from pelvic organs and deep inguinal nodes.
• Internal iliac nodes – Drain pelvic viscera, perineum, and gluteal region.
• Great toe lymphatics – Superficial drainage follows great saphenous vein to superficial inguinal nodes.
• Popliteal lymph nodes – Located behind knee, drain deep tissues of leg.
• Clinical correlation – Swelling in groin may indicate infection or malignancy in drainage territory.
• Saphenous vein – Long superficial vein of leg, associated with superficial lymphatics.
• Sentinel lymph node – First node to receive lymph from cancer site, important in oncology.
Chapter: Anatomy / Lower Limb
Topic: Lymphatic Drainage
Subtopic: Drainage of Foot and Great Toe
Lead Question – 2013
Skin and fascia of great toe drains into?
a) Superficial inguinal lymph nodes
b) External iliac nodes
c) Internal iliac nodes
d) Deep inguinal nodes
Explanation: The superficial lymphatics of the great toe accompany the great saphenous vein and drain primarily into the superficial inguinal lymph nodes. Deep lymphatics, however, drain into deep inguinal and external iliac nodes. Correct answer: Superficial inguinal lymph nodes.
Guessed Questions for NEET PG
1) Lymph from the glans penis drains into?
a) Superficial inguinal nodes
b) Deep inguinal nodes
c) External iliac nodes
d) Internal iliac nodes
Explanation: Lymph from the glans penis and clitoris drains into deep inguinal lymph nodes (node of Cloquet). This clinical correlation is important in genitourinary cancers. Correct answer: Deep inguinal nodes.
2) Infection at the lateral side of the foot drains initially into?
a) Popliteal nodes
b) Superficial inguinal nodes
c) Deep inguinal nodes
d) External iliac nodes
Explanation: Lymphatics from the lateral foot follow the small saphenous vein and drain into popliteal lymph nodes before reaching deeper nodes. Correct answer: Popliteal nodes.
3) Which lymph nodes are involved in carcinoma of the anal canal below pectinate line?
a) Internal iliac nodes
b) External iliac nodes
c) Superficial inguinal nodes
d) Para-aortic nodes
Explanation: The anal canal below the pectinate line drains into superficial inguinal nodes, explaining why inguinal swelling may be an early sign of malignancy. Correct answer: Superficial inguinal nodes.
4) The node of Cloquet is located in?
a) Femoral ring
b) Adductor canal
c) Inguinal ligament
d) Popliteal fossa
Explanation: The node of Cloquet is the highest deep inguinal lymph node, located in the femoral canal, and communicates with external iliac nodes. Correct answer: Femoral ring.
5) Which lymph nodes drain the uterus near the round ligament?
a) Internal iliac nodes
b) Para-aortic nodes
c) Superficial inguinal nodes
d) External iliac nodes
Explanation: Lymphatics from the uterus near the round ligament follow the ligament to reach the superficial inguinal nodes. Correct answer: Superficial inguinal nodes.
6) Which is the sentinel node in carcinoma of the cervix?
a) Internal iliac nodes
b) External iliac nodes
c) Superficial inguinal nodes
d) Para-aortic nodes
Explanation: The primary lymphatic drainage of cervix is to the internal iliac and sacral nodes, making them sentinel nodes for carcinoma cervix. Correct answer: Internal iliac nodes.
7) Enlargement of which nodes may indicate infection in the great toe?
a) Deep inguinal nodes
b) Superficial inguinal nodes
c) External iliac nodes
d) Para-aortic nodes
Explanation: Infection of great toe skin or fascia drains to superficial inguinal nodes, which enlarge clinically. Correct answer: Superficial inguinal nodes.
8) Popliteal lymph nodes drain all except?
a) Lateral side of sole
b) Heel region
c) Lateral border of foot
d) Medial side of great toe
Explanation: The medial side of great toe drains into superficial inguinal nodes, not popliteal nodes. Correct answer: Medial side of great toe.
9) Lymphatic obstruction in the femoral canal primarily affects?
a) Deep inguinal nodes
b) Superficial inguinal nodes
c) Popliteal nodes
d) Internal iliac nodes
Explanation: The femoral canal contains the node of Cloquet, which is part of deep inguinal lymphatic drainage. Obstruction here causes lower limb lymphedema. Correct answer: Deep inguinal nodes.
10) Which lymph nodes are first affected in carcinoma of the testis?
a) Superficial inguinal nodes
b) Deep inguinal nodes
c) Para-aortic nodes
d) External iliac nodes
Explanation: Testicular lymphatics follow gonadal vessels and drain into para-aortic (lumbar) nodes, not inguinal nodes. Correct answer: Para-aortic nodes.
Keyword Definitions
• Midinguinal point – Midpoint between anterior superior iliac spine and pubic symphysis; landmark for femoral artery.
• Adductor tubercle – Bony prominence on medial femoral condyle; insertion point for adductor magnus.
• Femoral artery – Main artery of thigh, continuation of external iliac artery.
• Inferior epigastric artery – Branch of external iliac artery, runs upward in anterior abdominal wall.
• Superior epigastric artery – Continuation of internal thoracic artery, supplies anterior abdominal wall.
• Adductor canal (Hunter’s canal) – Intermuscular tunnel through which femoral artery passes before becoming popliteal artery.
• Popliteal artery – Continuation of femoral artery after adductor hiatus.
• Femoral triangle – Triangular space in upper thigh, containing femoral nerve, artery, vein.
• Surface marking – Clinical method of tracing arteries/veins along landmarks.
• Aneurysm – Pathological dilatation of artery, can occur in femoral artery.
Chapter: Anatomy / Lower Limb
Topic: Femoral Region
Subtopic: Surface Marking of Femoral Artery
Lead Question – 2013
Line from midinguinal point to adductor tubercle represent?
a) Inferior epigastric artery
b) Femoral artery
c) Superior epigastric artery
d) None of the above
Explanation: The surface marking of the femoral artery is a line drawn from the midinguinal point to the adductor tubercle. It helps in palpating pulse and guiding surgical procedures. Correct answer: Femoral artery.
Guessed Questions for NEET PG
1) Which structure passes through the adductor hiatus with femoral artery?
a) Femoral vein
b) Femoral nerve
c) Great saphenous vein
d) Obturator nerve
Explanation: The femoral vein accompanies the femoral artery through the adductor hiatus, where they become the popliteal vessels. Correct answer: Femoral vein.
2) Which is not a content of femoral triangle?
a) Femoral nerve
b) Femoral artery
c) Femoral vein
d) Obturator artery
Explanation: The femoral triangle contains femoral nerve, artery, vein, and lymphatics. Obturator artery lies deeper and is not a content. Correct answer: Obturator artery.
3) Which artery is palpated at the midinguinal point?
a) Femoral artery
b) Popliteal artery
c) Posterior tibial artery
d) Dorsalis pedis artery
Explanation: The femoral artery pulse is palpated at the midinguinal point, a key clinical landmark in lower limb examination. Correct answer: Femoral artery.
4) Adductor canal transmits all except:
a) Femoral artery
b) Femoral vein
c) Saphenous nerve
d) Femoral nerve
Explanation: Femoral nerve does not pass through the adductor canal; only its saphenous branch does. Correct answer: Femoral nerve.
5) Which branch of femoral artery supplies head of femur?
a) Profunda femoris
b) Medial circumflex femoral
c) Lateral circumflex femoral
d) Superior gluteal
Explanation: The medial circumflex femoral artery (branch of profunda femoris) supplies most of the femoral head through retinacular branches. Correct answer: Medial circumflex femoral artery.
6) Which artery is continuation of femoral artery beyond adductor hiatus?
a) Popliteal artery
b) Anterior tibial artery
c) Posterior tibial artery
d) Peroneal artery
Explanation: The femoral artery continues as popliteal artery after passing through adductor hiatus. Correct answer: Popliteal artery.
7) Which structure lies medial to femoral artery in femoral sheath?
a) Femoral nerve
b) Femoral vein
c) Great saphenous vein
d) Inguinal ligament
Explanation: Within femoral sheath, femoral vein lies medial to femoral artery. Correct answer: Femoral vein.
8) Profunda femoris artery is a branch of:
a) Femoral artery
b) External iliac artery
c) Internal iliac artery
d) Obturator artery
Explanation: The profunda femoris artery arises from the femoral artery, 3–5 cm below the inguinal ligament. Correct answer: Femoral artery.
9) Which clinical test assesses patency of femoral artery?
a) Trendelenburg test
b) Allen’s test
c) Palpation of femoral pulse
d) Buerger’s test
Explanation: The femoral artery can be directly assessed by palpating its pulse at the midinguinal point. Correct answer: Palpation of femoral pulse.
10) Which nerve accompanies femoral artery in adductor canal?
a) Femoral nerve
b) Saphenous nerve
c) Obturator nerve
d) Tibial nerve
Explanation: The saphenous nerve, a branch of femoral nerve, accompanies femoral artery in the adductor canal. Correct answer: Saphenous nerve.
Chapter: Anatomy
Topic: Pelvis and Perineum
Subtopic: Sacrotuberous Ligament and Related Structures
Keyword Definitions:
Sacrotuberous Ligament: Strong ligament connecting sacrum to ischial tuberosity, stabilizing the sacroiliac joint.
Perforating Cutaneous Nerve: Small branch of sacral plexus that pierces sacrotuberous ligament to supply buttock skin.
Posterior Femoral Cutaneous Nerve: Nerve supplying posterior thigh and gluteal skin, passing beneath gluteus maximus.
Sciatic Nerve: Largest nerve of the body, leaves pelvis via greater sciatic foramen below piriformis.
Superior Gluteal Nerve: Nerve passing through greater sciatic foramen above piriformis to supply gluteus medius, minimus, and tensor fascia lata.
Clinical Relevance: Injury to sacrotuberous ligament region can cause entrapment neuropathy or gluteal pain syndromes.
Lead Question - 2013
Sacrotuberous ligament is pierced by
a) Perforating cutaneous nerve
b) Posterior femoral cutaneous
c) Superior gluteal nerve
d) Sciatic nerve
Explanation: The sacrotuberous ligament is pierced only by the perforating cutaneous nerve, which arises from the sacral plexus and supplies the medial gluteal region skin. Other nerves pass through sciatic foramina. The correct answer is a) Perforating cutaneous nerve.
Guessed Question 2
Which ligament forms the lower boundary of the lesser sciatic foramen?
a) Sacrospinous ligament
b) Sacrotuberous ligament
c) Inguinal ligament
d) Obturator membrane
Explanation: The sacrotuberous ligament extends from sacrum to ischial tuberosity, forming the lower boundary of lesser sciatic foramen. The sacrospinous ligament forms its upper boundary. The correct answer is b) Sacrotuberous ligament.
Guessed Question 3
Which nerve passes below the piriformis through the greater sciatic foramen?
a) Sciatic nerve
b) Superior gluteal nerve
c) Obturator nerve
d) Femoral nerve
Explanation: The sciatic nerve passes through the greater sciatic foramen below piriformis, running deep to gluteus maximus. Compression here may cause sciatica. The correct answer is a) Sciatic nerve.
Guessed Question 4
Which ligament along with sacrospinous converts sciatic notch into foramen?
a) Sacrotuberous ligament
b) Iliofemoral ligament
c) Pubofemoral ligament
d) Ischiofemoral ligament
Explanation: Sacrotuberous and sacrospinous ligaments transform greater and lesser sciatic notches into foramina through which pelvic nerves and vessels pass. The correct answer is a) Sacrotuberous ligament.
Guessed Question 5
In piriformis syndrome, which nerve is compressed as it passes beneath piriformis?
a) Sciatic nerve
b) Pudendal nerve
c) Posterior femoral cutaneous nerve
d) Inferior gluteal nerve
Explanation: In piriformis syndrome, the sciatic nerve gets compressed as it passes below piriformis, producing pain radiating to posterior thigh and leg. The correct answer is a) Sciatic nerve.
Guessed Question 6
Which nerve leaves pelvis through the lesser sciatic foramen?
a) Pudendal nerve
b) Superior gluteal nerve
c) Posterior femoral cutaneous nerve
d) Sciatic nerve
Explanation: The pudendal nerve exits through greater sciatic foramen, hooks around ischial spine and sacrospinous ligament, and re-enters through lesser sciatic foramen into the perineum. The correct answer is a) Pudendal nerve.
Guessed Question 7
Which structure passes between sacrotuberous and sacrospinous ligaments?
a) Pudendal nerve
b) Obturator nerve
c) Femoral nerve
d) Genitofemoral nerve
Explanation: Pudendal nerve, internal pudendal vessels, and nerve to obturator internus pass between sacrospinous and sacrotuberous ligaments to enter lesser sciatic foramen. The correct answer is a) Pudendal nerve.
Guessed Question 8
Damage to superior gluteal nerve affects which function?
a) Hip abduction
b) Hip extension
c) Hip adduction
d) Knee flexion
Explanation: Superior gluteal nerve supplies gluteus medius and minimus. Damage weakens hip abduction and causes positive Trendelenburg sign due to failure of pelvic support. The correct answer is a) Hip abduction.
Guessed Question 9
Which ligament resists posterior rotation of sacrum?
a) Sacrotuberous ligament
b) Anterior sacroiliac ligament
c) Iliofemoral ligament
d) Inguinal ligament
Explanation: Sacrotuberous ligament helps resist posterior rotation of sacrum at sacroiliac joint, stabilizing pelvis against body weight. The correct answer is a) Sacrotuberous ligament.
Guessed Question 10
Which nerve supplies posterior thigh skin but does not pierce sacrotuberous ligament?
a) Posterior femoral cutaneous nerve
b) Perforating cutaneous nerve
c) Obturator nerve
d) Pudendal nerve
Explanation: Posterior femoral cutaneous nerve passes below gluteus maximus to supply posterior thigh skin but does not pierce sacrotuberous ligament. The correct answer is a) Posterior femoral cutaneous nerve.
Guessed Question 11
Entrapment of perforating cutaneous nerve causes pain in which region?
a) Medial gluteal region
b) Posterior thigh
c) Perineum
d) Lateral leg
Explanation: The perforating cutaneous nerve, piercing sacrotuberous ligament, supplies medial gluteal region. Entrapment or irritation causes localized gluteal pain. The correct answer is a) Medial gluteal region.
Keyword Definitions
• Femoral head – Spherical upper end of femur that articulates with acetabulum to form hip joint.
• Medial circumflex femoral artery – Major artery supplying femoral head and neck via retinacular branches.
• Lateral circumflex femoral artery – Smaller contribution to anterior femoral neck and head.
• Ligamentum teres artery – Small artery within ligament of head of femur; significant in children only.
• Profunda femoris artery – Deep femoral artery; gives rise to medial and lateral circumflex branches.
• Retinacular vessels – Arterial branches running along femoral neck beneath joint capsule to supply femoral head.
• Avascular necrosis (AVN) – Bone death due to interruption of blood supply; common complication of femoral neck fracture.
• Intracapsular fracture – Femoral neck fracture within capsule; disrupts blood supply leading to AVN.
• Hip dislocation – Displacement of femoral head; can stretch or rupture retinacular arteries.
• Artery of ligamentum teres – Vestigial in adults but may partially supply femoral head in young children.
Chapter: Anatomy / Lower Limb
Topic: Hip Joint
Subtopic: Blood Supply of Femoral Head
Lead Question – 2013
The blood supply to femoral head is mostly by?
a) Lateral epiphyseal artery
b) Medial epiphyseal artery
c) Ligamentous teres artery
d) Profunda femoris
Explanation: The main supply to the femoral head in adults is from retinacular branches of the medial circumflex femoral artery. The lateral circumflex contributes minimally. The artery of ligamentum teres is significant only in children. Profunda femoris is the parent trunk. Correct answer: Medial epiphyseal artery.
Guessed Questions for NEET PG
1) Which artery is most at risk of injury in intracapsular fracture of femoral neck?
a) Medial circumflex femoral
b) Lateral circumflex femoral
c) Profunda femoris
d) Obturator
Explanation: Intracapsular fractures damage retinacular branches of the medial circumflex femoral artery, the primary supplier of the femoral head. This disruption causes avascular necrosis. Correct answer: Medial circumflex femoral artery.
2) In children, which artery contributes significantly to femoral head supply?
a) Artery of ligamentum teres
b) Medial circumflex femoral
c) Lateral circumflex femoral
d) Inferior gluteal
Explanation: In children, the artery of ligamentum teres (branch of obturator artery) is important for femoral head vascularity. With age, this vessel regresses and becomes less significant. Correct answer: Artery of ligamentum teres.
3) Which complication most commonly follows fracture of femoral neck in elderly?
a) Osteoarthritis
b) Avascular necrosis of femoral head
c) Osteomyelitis
d) Sciatic nerve injury
Explanation: Disruption of medial circumflex femoral artery branches leads to avascular necrosis of the femoral head, particularly after intracapsular neck fractures. Correct answer: Avascular necrosis of femoral head.
4) Lateral circumflex femoral artery is mainly a branch of:
a) Internal iliac artery
b) Profunda femoris artery
c) Common femoral artery
d) External iliac artery
Explanation: The lateral circumflex femoral artery is a branch of profunda femoris artery. It supplies anterior thigh and gives ascending branches to the femoral neck. Correct answer: Profunda femoris artery.
5) Avascular necrosis of femoral head is best diagnosed initially by:
a) Plain X-ray
b) CT scan
c) MRI
d) Ultrasound
Explanation: MRI is the most sensitive modality to detect early avascular necrosis of femoral head before radiographic changes appear. Correct answer: MRI.
6) Which artery forms cruciate anastomosis around hip joint?
a) Superior gluteal
b) Inferior gluteal
c) Medial circumflex femoral
d) All of the above
Explanation: Cruciate anastomosis is formed by inferior gluteal, medial circumflex femoral, lateral circumflex femoral (transverse branch), and first perforator of profunda femoris. Correct answer: All of the above (with contributions).
7) Which ligament encloses the artery of ligamentum teres?
a) Iliofemoral ligament
b) Ischiofemoral ligament
c) Ligament of head of femur
d) Pubofemoral ligament
Explanation: The artery of ligamentum teres passes through the ligament of head of femur (ligamentum teres). Correct answer: Ligament of head of femur.
8) In posterior hip dislocation, blood supply to femoral head is compromised because:
a) Ligamentum teres is torn
b) Retinacular vessels from medial circumflex femoral artery are damaged
c) Obturator nerve compression
d) Venous drainage blocked
Explanation: Posterior dislocation stretches and tears retinacular vessels of medial circumflex femoral artery, causing avascular necrosis. Correct answer: Damage to retinacular vessels from medial circumflex artery.
9) Which artery does not participate in trochanteric anastomosis?
a) Superior gluteal
b) Inferior gluteal
c) Medial circumflex femoral
d) Anterior tibial
Explanation: Trochanteric anastomosis includes superior gluteal, inferior gluteal, medial circumflex femoral, and lateral circumflex femoral arteries. Anterior tibial does not contribute. Correct answer: Anterior tibial artery.
10) Best management for avascular necrosis of femoral head in advanced stage is:
a) Core decompression
b) Bisphosphonates
c) Total hip replacement
d) Bone marrow injection
Explanation: In advanced avascular necrosis with collapse of femoral head, total hip replacement is the definitive treatment. Early stages may respond to decompression. Correct answer: Total hip replacement.
Keyword Definitions
• Gastrocnemius – Superficial, two-headed calf muscle crossing knee and ankle; powerful plantarflexor.
• Soleus – Deep to gastrocnemius, single-headed, plantarflexes ankle (postural muscle).
• Triceps surae – Collective term for gastrocnemius + soleus muscles forming the calf.
• Calcaneal (Achilles) tendon – Common tendon of triceps surae inserting into calcaneus.
• Tibial nerve – Branch of sciatic nerve supplying posterior compartment of leg and plantar foot.
• Plantarflexion – Downward movement of foot at ankle produced by triceps surae.
• Sural nerve – Sensory nerve formed by contributions from tibial and common peroneal nerves; supplies lateral foot.
• Posterior tibial artery – Major artery supplying posterior compartment and plantar foot.
• Baker’s cyst – Popliteal synovial cyst that may compress neurovascular structures in popliteal fossa.
• Achilles tendon rupture – Clinical injury causing inability to plantarflex and toe-raise; Thompson test positive.
Chapter: Anatomy / Lower Limb
Topic: Posterior Compartment of Leg
Subtopic: Triceps Surae (Gastrocnemius & Soleus)
Lead Question – 2013
Which muscles is known as 'Triceps surae'?
a) Gastro-soleus
b) Popliteus
c) EHL
d) EDL
Explanation: “Triceps surae” refers to the gastrocnemius and soleus acting together as the calf complex. Their common insertion via the calcaneal (Achilles) tendon produces powerful plantarflexion. Popliteus, EHL, and EDL are distinct muscles with different functions. Correct answer: Gastro-soleus (triceps surae).
Guessed Questions for NEET PG
1) Which nerve supplies triceps surae?
a) Common peroneal
b) Tibial
c) Femoral
d) Sural
Explanation: Triceps surae (gastrocnemius and soleus) are supplied by the tibial nerve, a branch of the sciatic nerve. Motor fibers reach muscles in the posterior compartment and sensory branches form part of the sural nerve. Correct answer: Tibial nerve. Clinically important in sciatic lesions.
2) The common tendon of triceps surae inserts into the:
a) Navicular
b) Calcaneus
c) Cuboid
d) Talus
Explanation: The calcaneal (Achilles) tendon attaches the triceps surae to the posterior calcaneus. This insertion transmits strong plantarflexor force. Rupture here causes inability to plantarflex and a positive Thompson test. Correct answer: Calcaneus.
3) Thompson test assesses rupture of which structure?
a) Plantaris tendon
b) Calcaneal (Achilles) tendon
c) Tibialis posterior tendon
d) Peroneus brevis tendon
Explanation: Squeezing the calf normally causes plantarflexion; absence indicates Achilles tendon rupture. Thompson test is thus specific for calcaneal tendon discontinuity, often from sudden dorsiflexion injury. Correct answer: Calcaneal (Achilles) tendon.
4) Which artery mainly supplies the triceps surae muscles?
a) Anterior tibial artery
b) Posterior tibial artery and its branches (peroneal included)
c) Dorsalis pedis artery
d) Femoral artery only
Explanation: The posterior tibial artery and contributions from the peroneal (fibular) artery supply the posterior compartment including triceps surae. Anterior tibial and dorsalis pedis supply anterior structures. Correct answer: Posterior tibial artery and branches. Important in ischemic leg evaluation.
5) Gastrocnemius contributes more to plantarflexion when the knee is:
a) Flexed
b) Extended
c) Neutral – unaffected by knee position
d) Internally rotated
Explanation: Gastrocnemius crosses the knee; it generates more plantarflexion when the knee is extended, as knee flexion slackens it. Soleus (monoarticular) acts regardless of knee position. Correct answer: Extended. Clinical tests consider knee position when assessing calf strength.
6) A positive calf squeeze with no plantarflexion indicates lesion of:
a) Posterior tibial nerve distal to gastrocnemius
b) Achilles tendon rupture
c) Tibialis anterior nerve injury
d) Peroneal nerve palsy
Explanation: The Thompson (calf squeeze) test shows absent plantarflexion in Achilles tendon rupture, not in isolated nerve palsy. Tendon discontinuity prevents force transmission despite intact muscle. Correct answer: Achilles tendon rupture. Prompt management needed to restore push-off.
7) Plantaris muscle, when present, lies between which two structures related to triceps surae?
a) Tibialis anterior and EHL
b) Gastrocnemius and soleus, with a long tendon medial to Achilles
c) Peroneus longus and brevis
d) Flexor hallucis longus and tibialis posterior
Explanation: Plantaris is a small accessory muscle with a long slender tendon that runs between gastrocnemius and soleus and often parallels the Achilles tendon medially. It is sometimes used as a tendon graft. Correct answer: Gastrocnemius and soleus region.
8) Rupture of the Achilles tendon most commonly results in loss of which gait phase action?
a) Heel strike
b) Toe-off/pushoff (plantarflexion phase)
c) Mid-stance stability only
d) Swing phase clearance
Explanation: Achilles rupture prevents effective plantarflexion needed for toe-off/pushoff, leading to impaired propulsion and altered gait. Patients have weak push-off and may show a palpable gap. Correct answer: Toe-off/pushoff. Surgical repair often restores function.
9) Which clinical condition involves pain and tightness of triceps surae with increased compartment pressure?
a) Plantar fasciitis
b) Posterior compartment syndrome of leg
c) Lateral ankle sprain
d) Deep vein thrombosis only
Explanation: Exertional or acute posterior compartment syndrome involves the deep posterior compartment including triceps surae, causing pain, tense swelling, and neurovascular compromise. Urgent fasciotomy is required. Correct answer: Posterior compartment syndrome (affecting calf muscles).
10) Sural nerve sensory distribution relates closely to triceps surae because it supplies:
a) Medial plantar surface of foot
b) Lateral aspect of foot and posterior calf skin overlying gastrocnemius
d) Plantar aspect of toes 2–4
Explanation: The sural nerve provides cutaneous innervation to the posterior calf and lateral foot, areas overlying the gastrocnemius and Achilles tendon. Injury causes sensory loss here and may accompany procedures on the calf. Correct answer: Lateral foot and posterior calf skin.
Keyword Definitions
• Flexor retinaculum (ankle) – Fibrous band over medial ankle forming the roof of the tarsal tunnel.
• Tarsal tunnel – Space deep to flexor retinaculum transmitting tendons, vessels, and nerve into the foot.
• Posterior tibial artery – Major artery passing through tarsal tunnel to supply plantar foot; palpable as posterior tibial pulse.
• Tibialis anterior tendon – Anterior compartment tendon crossing dorsum of foot; does NOT pass under flexor retinaculum.
• Peroneus tertius – Anterior-lateral tendon inserting on dorsum of 5th metatarsal; not in tarsal tunnel.
• Long saphenous (great saphenous) vein – Superficial vein running anterior to medial malleolus, superficial to retinaculum.
• Tom, Dick And Very Nervous Harry – Mnemonic for structures deep to flexor retinaculum: Tibialis posterior, flexor Digitorum longus, posterior tibial Artery, posterior tibial Vein, tibial Nerve, flexor Hallucis longus.
• Tinel’s sign (at ankle) – Tapping over flexor retinaculum producing tingling in tunnel distribution, suggests tibial nerve entrapment.
• Posterior tibial pulse – Palpated just posterior to medial malleolus deep to flexor retinaculum; important in vascular exam.
• Clinical relevance – Tarsal tunnel syndrome results from compression of structures under flexor retinaculum causing plantar numbness/pain.
Chapter: Anatomy / Lower Limb
Topic: Ankle Region
Subtopic: Flexor Retinaculum (Tarsal Tunnel) Contents
Lead Question – 2013
Structure passing deep to flexor retinaculum is ?
a) Posterior tibial artery
b) Long saphenous vein
c) Tibialis anterior tendon
d) Peroneus tertius
Explanation: The posterior tibial artery passes deep to the flexor retinaculum within the tarsal tunnel alongside tendons and the tibial nerve. The long saphenous vein is superficial, tibialis anterior and peroneus tertius are anterior tendons. Correct answer: (a) Posterior tibial artery. Clinically the posterior tibial pulse is palpable here.
Guessed Questions for NEET PG
1) The tarsal tunnel contains all EXCEPT:
a) Tibialis posterior tendon
b) Flexor digitorum longus tendon
c) Peroneus longus tendon
d) Posterior tibial nerve
Explanation: The peroneus longus runs laterally and passes under the cuboid (peroneal groove), not through the tarsal tunnel. The tunnel contains tibialis posterior, FDL, posterior tibial vessels, tibial nerve, and FHL. Correct answer: Peroneus longus. Tarsal tunnel syndrome spares lateral tendons.
2) Posterior tibial pulse is best palpated:
a) Anterior to lateral malleolus
b) Posterior to medial malleolus beneath flexor retinaculum
c) On dorsum of foot lateral to EHL tendon
d) In popliteal fossa only
Explanation: The posterior tibial artery runs deep to flexor retinaculum posterior to the medial malleolus; its pulse is palpated there. Correct answer: Posterior to medial malleolus beneath flexor retinaculum. Loss of this pulse suggests distal arterial compromise.
3) Tinel’s sign at the tarsal tunnel tests for entrapment of which nerve?
a) Superficial peroneal nerve
b) Deep peroneal nerve
c) Tibial nerve (posterior tibial nerve in tunnel)
d) Sural nerve
Explanation: Tinel’s tapping over flexor retinaculum reproduces paresthesia in tibial nerve distribution (sole) when entrapped. Correct answer: Tibial nerve. Positive test aids diagnosis of tarsal tunnel syndrome which presents with plantar numbness and burning pain.
4) Which tendon is most medial under the flexor retinaculum (medial to lateral order)?
a) Flexor hallucis longus
b) Tibialis posterior
c) Flexor digitorum longus
d) Peroneus brevis
Explanation: Medial-to-lateral order in tarsal tunnel is tibialis posterior, flexor digitorum longus, posterior tibial vessels/nerve, then flexor hallucis longus more laterally. Peroneal tendons are lateral. Correct answer: Tibialis posterior. Important during surgical decompression.
5) Compression of posterior tibial nerve in the tarsal tunnel causes loss of sensation over:
a) Dorsum of foot only
b) Plantar surface of foot and toes
c) Lateral calf only
d) Medial thigh
Explanation: Tibial nerve supplies plantar cutaneous nerves; entrapment in tarsal tunnel causes plantar burning, numbness, and possible intrinsic muscle weakness. Correct answer: Plantar surface of foot and toes. Tarsal tunnel mimics plantar fasciitis clinically sometimes.
6) A patient with rupture of posterior tibial artery in ankle trauma will most likely present with:
a) Loss of dorsalis pedis pulse only
b) Absent posterior tibial pulse, ischemic plantar changes
c) Isolated foot drop
d) Loss of great saphenous waveform only
Explanation: Rupture of posterior tibial artery abolishes its palpable pulse and can compromise plantar circulation causing ischemic changes. Dorsalis pedis may be maintained via anterior tibial flow. Correct answer: Absent posterior tibial pulse, ischemic plantar changes. Urgent vascular assessment required.
7) The mnemonic “Tom, Dick, And Very Nervous Harry” lists structures in tarsal tunnel in which order?
a) Tibialis posterior, flexor Digitorum longus, posterior tibial Artery, posterior tibial Vein, tibial Nerve, flexor Hallucis longus
b) Tibialis anterior, extensor Digitorum longus, anterior tibial Artery…
c) Peroneus longus, peroneus brevis, sural nerve…
d) Flexor hallucis longus first then others
Explanation: The mnemonic correctly orders tibialis posterior, FDL, posterior tibial artery, posterior tibial vein, tibial nerve, and flexor hallucis longus (from medial to lateral). Correct answer: (a). Surgeons use this to identify structures during decompression.
8) Which vessel provides collateral supply to plantar arch if posterior tibial artery is occluded?
a) Anterior tibial (via dorsalis pedis and perforating branches)
b) Great saphenous vein
c) Peroneal artery exclusively without connections
d) Small saphenous vein
Explanation: Anterior tibial continues as dorsalis pedis and via perforating branches can contribute to plantar arches, providing collateral flow when posterior tibial artery is occluded. Correct answer: Anterior tibial (via dorsalis pedis). Clinical: Important in planning bypass and assessing ischemia.
9) Long saphenous vein at the ankle is located relative to flexor retinaculum as:
a) Deep to retinaculum within tarsal tunnel
b) Superficial to retinaculum anterior to medial malleolus
c) Passing through lateral retinaculum
d) Within tarsal tunnel posterior to tibial nerve
Explanation: The great saphenous vein is superficial on the medial ankle, anterior to the medial malleolus and superficial to the flexor retinaculum, used for venous cutdown access. Correct answer: Superficial to retinaculum anterior to medial malleolus. Preserve saphenous nerve during harvest.
10) Surgical decompression for tarsal tunnel syndrome requires incision of which structure?
a) Flexor retinaculum (tarsal tunnel roof)
b) Extensor retinaculum
c) Plantar aponeurosis only
d) Lateral ankle ligament complex
Explanation: Tarsal tunnel release involves incising the flexor retinaculum to decompress tibial nerve and associated structures. Correct answer: Flexor retinaculum. Timing is important as chronic compression can cause irreversible neuropathy and intrinsic foot muscle atrophy.
Keyword Definitions
• Anterior compartment (leg) – The muscular compartment on the front of the leg containing tibialis anterior, EDL, EHL, and popliteo-tibial neurovascular structures.
• Tibialis anterior (TA) – Primary dorsiflexor and inverter of foot; tendon medial to bundle at ankle.
• Extensor hallucis longus (EHL) – Extends big toe; its tendon lies between TA and EDL at the dorsum.
• Extensor digitorum longus (EDL) – Extends toes 2–5; lateral to EHL and often associated with peroneus tertius.
• Peroneus tertius – Variable muscle, tendon joins EDL over lateral dorsum, aids eversion.
• Anterior tibial artery – Main artery of anterior compartment, continues as dorsalis pedis on dorsum of foot.
• Deep peroneal (fibular) nerve – Motor to anterior compartment, sensory to web space between 1st and 2nd toes.
• Extensor retinaculum – Holds extensor tendons and neurovascular bundle at the ankle.
• Clinical correlation – Localization of bundle important for dorsalis pedis pulse and ankle compartment surgery.
• Surgical relevance – Avoid injury to anterior tibial vessels and deep peroneal nerve during ankle procedures.
Chapter: Anatomy / Lower Limb
Topic: Leg — Anterior Compartment
Subtopic: Neurovascular relations at the ankle
Lead Question – 2013
Neurovascular bundle of anterior compartment of leg passes between the tendons of ?
a) EHL and EDL
b) EDL and peroneus tertius
c) Tibialis anterior and EHL
d) None of the above
Explanation: The anterior tibial vessels and deep peroneal nerve travel in the anterior compartment and, at the ankle beneath the extensor retinaculum, lie between the tendons of tibialis anterior (medial) and extensor hallucis longus (lateral). Correct answer: (c) Tibialis anterior and EHL. Clinically palpable as dorsalis pedis pulse distal to this region.
Guessed Questions for NEET PG
1) The artery continuing from anterior tibial artery onto the dorsum of the foot is:
a) Posterior tibial artery
b) Dorsalis pedis artery
c) Peroneal artery
d) Medial plantar artery
Explanation: The anterior tibial artery continues over the ankle as the dorsalis pedis artery on the dorsum of the foot. Correct answer: Dorsalis pedis artery. Clinically the dorsalis pedis pulse is used to assess peripheral perfusion and arterial injury in the foot.
2) Sensory supply of the web space between first and second toes is by:
a) Superficial peroneal nerve
b) Deep peroneal nerve
c) Saphenous nerve
d) Tibial nerve
Explanation: The deep peroneal (fibular) nerve supplies the skin between the first and second toes. Correct answer: Deep peroneal nerve. Clinically, numbness here suggests injury to the deep peroneal nerve at the ankle or leg.
3) A patient with foot drop after fibular neck fracture likely has injury to:
a) Superficial peroneal nerve
b) Deep peroneal nerve
c) Tibial nerve
d) Sural nerve
Explanation: Common fibular (peroneal) nerve wraps around fibular neck and its deep branch supplies anterior compartment dorsiflexors. Injury causes foot drop. Correct answer: Deep peroneal (branch of common peroneal). Clinically presents with steppage gait.
4) The extensor retinaculum prevents bowstringing of:
a) Flexor tendons only
b) Extensor tendons only
c) Both flexor and extensor tendons equally
d) Peroneal tendons only
Explanation: The extensor retinaculum secures extensor tendons at the ankle, preventing bowstringing during dorsiflexion. Correct answer: Extensor tendons only. Clinically, tight retinaculum can cause tenosynovitis and pain over the dorsum of ankle.
5) A weak dorsalis pedis pulse with intact posterior tibial pulse suggests occlusion of:
a) Anterior tibial artery
b) Posterior tibial artery
c) Peroneal artery
d) Femoral artery
Explanation: Diminished dorsalis pedis with normal posterior tibial pulse suggests anterior tibial artery compromise. Correct answer: Anterior tibial artery. Clinically important in acute limb ischemia and after tibial fractures.
6) In anterior compartment syndrome, which action is most affected?
a) Plantarflexion of ankle
b) Dorsiflexion of ankle
c) Inversion of foot only
d) Toe abduction
Explanation: Anterior compartment contains dorsiflexors (tibialis anterior, EDL, EHL); raised pressure causes ischemia and loss of dorsiflexion. Correct answer: Dorsiflexion of ankle. Clinically urgent fasciotomy prevents permanent deficit.
7) The tendon order across the dorsum (medial to lateral) just distal to the ankle is:
a) TA, EHL, EDL, peroneus tertius
b) EDL, EHL, TA, peroneus tertius
c) Peroneus tertius, EDL, EHL, TA
d) TA, EDL, EHL, peroneus tertius
Explanation: From medial to lateral the tendons are tibialis anterior, extensor hallucis longus, extensor digitorum longus, then peroneus tertius. Correct answer: TA, EHL, EDL, peroneus tertius. This ordering helps localize neurovascular structures and tendinous pathology.
8) Injury to deep peroneal nerve at the ankle will cause sensory loss where?
a) Lateral foot dorsum
b) Medial sole
c) First web space between toes 1 and 2
d) Posterior calf
Explanation: Deep peroneal nerve supplies sensation to the first web space dorsally. Correct answer: First web space. Clinically, combined motor (dorsiflexion) and this specific sensory loss indicate deep peroneal lesion.
9) The best site to palpate the dorsalis pedis pulse is lateral to which tendon?
a) Tibialis anterior tendon
b) Extensor hallucis longus tendon
c) Extensor digitorum longus tendon
d) Peroneus tertius tendon
Explanation: The dorsalis pedis artery is palpated lateral to the tendon of extensor hallucis longus on the dorsum of the foot. Correct answer: Extensor hallucis longus tendon. Clinically used in vascular exams of the foot.
10) Surgical release of anterior compartment at the leg should avoid damaging which structure running in the compartment?
a) Great saphenous vein
b) Posterior tibial nerve
c) Deep peroneal nerve and anterior tibial vessels
d) Superficial peroneal nerve only
Explanation: The deep peroneal nerve and anterior tibial vessels run in the anterior compartment and must be preserved during fasciotomy or debridement. Correct answer: Deep peroneal nerve and anterior tibial vessels. Clinically, careful technique prevents vascular and motor-sensory loss.
Keyword Definitions
• Great saphenous vein – Longest superficial vein, runs from the medial foot to the groin, drains into femoral vein at saphenous (femoral) opening.
• Dorsal venous arch – Venous network on dorsum of foot; medial end gives origin to great saphenous vein.
• Saphenous nerve – Sensory branch of femoral nerve that accompanies the great saphenous vein along medial leg.
• Small (lesser) saphenous vein – Superficial posterior leg vein draining to popliteal vein; accompanied by sural nerve.
• Saphenous opening (fossa ovalis) – Gap in fascia lata where great saphenous vein passes to join femoral vein.
• Perforator veins – Connect superficial and deep systems (e.g., Cockett perforators); incompetence causes varicose veins.
• Varicose veins – Dilated, tortuous superficial veins due to valvular incompetence, commonly involve great saphenous tributaries.
• Saphenous cutdown – Surgical exposure of great saphenous vein at medial ankle for venous access.
• CABG conduit – Great saphenous vein commonly harvested for coronary artery bypass grafting.
• Clinical relevance – Knowledge of relationships important for varicose vein surgery, saphenous nerve preservation, and venous access.
Chapter: Anatomy / Lower Limb
Topic: Superficial Venous System of Lower Limb
Subtopic: Great Saphenous Vein – Anatomy and Clinical Correlates
Lead Question – 2013
True statement about great saphenous vein
a) It begins at lateral end of dorsal venous arch
b) It runs anterior to medial malleolus
c) It is accompanied by sural nerve
d) Terminates into popliteal vein
Explanation: The great saphenous vein arises from the **medial** end of the dorsal venous arch, ascends **anterior to the medial malleolus**, and is accompanied by the saphenous nerve. It terminates into the femoral vein at the saphenous opening, not the popliteal. Correct answer: (b). Clinically vital for varicose vein surgery and graft harvest.
Guessed Questions for NEET PG
1) The great saphenous vein terminates into the:
a) Popliteal vein
b) Femoral vein at saphenous opening
c) Small saphenous vein
d) Posterior tibial vein
Explanation: The great saphenous vein ascends the medial leg and pierces the fascia lata at the saphenous (femoral) opening to drain into the femoral vein. Correct answer: Femoral vein at saphenous opening. Clinically, this junction is inspected during varicose vein surgery and duplex scanning.
2) Which nerve accompanies the great saphenous vein along the medial leg?
a) Sural nerve
b) Saphenous nerve
c) Superficial peroneal nerve
d) Tibial nerve
Explanation: The saphenous nerve (branch of femoral nerve) runs with the great saphenous vein along the medial aspect of the leg and ankle carrying cutaneous sensation. Correct answer: Saphenous nerve. Clinical: Preserve this nerve during vein harvest to avoid medial leg numbness.
3) The commonest complication of great saphenous vein valve incompetence is:
a) Deep vein thrombosis only
b) Varicose veins of medial leg and thigh
c) Plantar fasciitis
d) Morton's neuroma
Explanation: Incompetence of valves in great saphenous and its tributaries leads to venous hypertension and varicose veins, typically along the medial leg and thigh. Correct answer: Varicose veins. Clinical: Presents with aching, swelling, and skin changes; treated by ablation or stripping.
4) The origin of the great saphenous vein is at the:
a) Lateral end of dorsal venous arch
b) Medial end of dorsal venous arch (near first metatarsal)
c) Posterior aspect of heel
d) Popliteal fossa
Explanation: Great saphenous vein begins at the **medial** end of the dorsal venous arch near the medial side of the foot and first metatarsal region. Correct answer: Medial end of dorsal venous arch. Clinically, this is the landmark for saphenous cutdown access.
5) The small saphenous vein typically drains into the:
a) Femoral vein
b) Popliteal vein
c) Great saphenous vein
d) Anterior tibial vein
Explanation: The small (lesser) saphenous vein ascends posterior calf and usually drains into the popliteal vein in the popliteal fossa. Correct answer: Popliteal vein. Clinical distinction helps in planning venous ablation and thrombosis evaluation.
6) Which perforator group is classically associated with great saphenous reflux in the lower leg?
a) Cockett perforators (lower leg)
b) Dodd perforators (thigh only)
c) Boyd perforators (popliteal crease only)
d) No perforators in lower leg
Explanation: Cockett perforators are located in the lower calf and commonly allow reflux from deep to superficial veins contributing to great saphenous varicosities. Correct answer: Cockett perforators. Clinical: Identified and ligated in surgical treatment of varicose veins.
7) For coronary artery bypass grafting, the great saphenous vein is harvested because it is:
a) Short and deep
b) Long, superficial, and of suitable caliber
c) Accompanied by artery
d) Immune to atherosclerosis
Explanation: The great saphenous vein is long, superficial, readily accessible, and generally of adequate caliber for bypass grafting. Correct answer: Long, superficial, and of suitable caliber. Clinical: Harvest technique must preserve side branches and avoid injury to saphenous nerve.
8) In saphenous vein cutdown for emergency venous access, the incision is usually made just anterior to the:
a) Lateral malleolus
b) Medial malleolus
c) Tibial tuberosity
d) Popliteal fossa
Explanation: Saphenous vein cutdown is performed anterior to the medial malleolus where the great saphenous vein is superficial and fixed, facilitating cannulation. Correct answer: Medial malleolus. Clinical: Useful when peripheral access is difficult; take care to avoid saphenous nerve.
9) Which statement about great saphenous vein valves is correct?
a) Valves prevent flow from superficial to deep veins only
b) Valves are absent in great saphenous vein
c) Valve incompetence leads to retrograde flow and varicosities
d) Valves force blood toward foot
Explanation: Venous valves normally permit unidirectional flow toward the heart; incompetence in the great saphenous system causes retrograde flow and varicose veins. Correct answer: Valve incompetence leads to retrograde flow and varicosities. Clinical: Duplex ultrasound assesses valve function before intervention.
10) A patient with thrombosis limited to the great saphenous vein (superficial thrombophlebitis) most importantly requires evaluation for:
a) Pulmonary embolism risk and extension into deep venous system
b) Immediate limb amputation
c) Coronary artery disease
d) Spinal cord compression
Explanation: Superficial thrombophlebitis of great saphenous vein may extend into deep veins, risking DVT and pulmonary embolism; evaluate with duplex ultrasound and treat accordingly. Correct answer: Pulmonary embolism risk and extension into deep venous system. Clinical: Anticoagulation and surveillance may be needed.
Keyword Definitions
• Interosseous membrane – Fibrous sheet between radius and ulna, provides attachment for muscles and transmits forces.
• Anterior interosseous artery – Branch of common interosseous artery running on anterior surface of interosseous membrane supplying deep forearm muscles.
• Posterior interosseous artery – Branch that reaches the posterior compartment, often passes through/perforates the interosseous membrane to supply extensors.
• Common interosseous artery – Short trunk from ulnar artery dividing into anterior and posterior interosseous arteries.
• Interosseous space – The gap between radius and ulna occupied by membrane and vessels; communicates between compartments.
• Posterior interosseous nerve – Deep branch of radial nerve running in posterior compartment with posterior interosseous vessels.
• Perforating branches – Small vessels that traverse the interosseous membrane to connect anterior and posterior circulations.
• Clinical relevance – Knowledge is vital in forearm fractures and surgical approaches to avoid vascular injury.
• Supination/pronation force transmission – Interosseous membrane transmits axial loads from radius to ulna during weight-bearing.
• Surgical landmark – Interosseous membrane used as reference during forearm reconstructive procedures.
Chapter: Anatomy / Upper Limb
Topic: Forearm Vessels and Membranes
Subtopic: Interosseous Membrane and its Perforators
Lead Question – 2013
Interosseous membrane of forearm is pierced by?
a) Brachial artery
b) Anterior interosseous artery
c) Posterior interosseous artery
d) Ulnar recurrent artery
Explanation: The posterior interosseous artery typically pierces the interosseous membrane to reach the posterior compartment, accompanying the posterior interosseous nerve. The anterior interosseous artery runs on the anterior surface and sends perforators. Correct answer: Posterior interosseous artery. Clinically important in posterior compartment surgeries and fractures.
Guessed Questions for NEET PG
1) The anterior interosseous artery is a branch of:
a) Radial artery
b) Ulnar artery (via common interosseous)
c) Brachial artery directly
d) Posterior interosseous artery
Explanation: The anterior interosseous artery arises from the common interosseous branch of the ulnar artery and runs on the anterior surface of the interosseous membrane. Correct answer: Ulnar artery (via common interosseous). Clinical: AIN and artery are vulnerable in proximal forearm trauma.
2) Which nerve accompanies the posterior interosseous artery in the posterior compartment?
a) Superficial radial nerve
b) Posterior interosseous nerve (deep branch of radial)
c) Median nerve
d) Ulnar nerve
Explanation: The posterior interosseous nerve (deep branch of radial nerve) accompanies the posterior interosseous artery in the posterior compartment to supply extensor muscles. Correct answer: Posterior interosseous nerve. Clinical: Injury causes finger extension weakness without sensory loss.
3) Perforating branches of the interosseous arteries allow communication between:
a) Radial and ulnar arteries only
b) Anterior and posterior compartments of forearm
c) Superficial and deep palmar arches
d) Brachial and radial arteries
Explanation: Perforators through the interosseous membrane connect anterior and posterior interosseous arteries, providing collateral circulation between forearm compartments. Correct answer: Anterior and posterior compartments. Clinical: Important when primary vessels are injured.
4) Injury to posterior interosseous artery in proximal forearm most likely causes:
a) Pure sensory loss in hand
b) Ischemia of posterior compartment muscles
c) Loss of pronation only
d) Thumb adduction loss
Explanation: Damage to posterior interosseous artery reduces blood supply to posterior (extensor) compartment leading to ischemic pain and weakness. Correct answer: Ischemia of posterior compartment muscles. Clinical: May accompany fractures or surgical insults.
5) The common interosseous artery usually arises from:
a) Radial artery
b) Ulnar artery
c) Brachial artery at cubital fossa
d) Profunda brachii artery
Explanation: The common interosseous artery branches from the ulnar artery shortly after the ulnar origin, then divides into anterior and posterior interosseous arteries. Correct answer: Ulnar artery. Clinical: Variant anatomy can affect flap planning.
6) Which structure runs along the anterior surface of the interosseous membrane?
a) Posterior interosseous artery
b) Anterior interosseous artery and nerve
c) Superficial radial nerve
d) Ulnar nerve
Explanation: The anterior interosseous artery and anterior interosseous branch of median nerve run on the anterior surface of the interosseous membrane supplying deep flexors. Correct answer: Anterior interosseous artery and nerve. Clinical: AIN palsy causes pure motor deficits.
7) The posterior interosseous artery usually reaches the posterior compartment via a gap near which landmark?
a) Lister’s tubercle
b) Proximal border of interosseous membrane near supinator
c) Ulnar styloid process
d) Pisiform bone
Explanation: The posterior interosseous artery commonly passes to the posterior compartment near the proximal border of the interosseous membrane in the region of the supinator. Correct answer: Proximal border of interosseous membrane near supinator. Clinical: Supinator syndrome may compromise vessels and nerve.
8) In a Galeazzi fracture (distal radius with DRUJ disruption), which artery's flow might be compromised affecting interosseous communication?
a) Brachial artery
b) Anterior interosseous artery
c) Posterior tibial artery
d) Median artery
Explanation: A distal radius fracture can disturb branches including anterior interosseous artery or its perforators, impairing collateral flow between compartments. Correct answer: Anterior interosseous artery. Clinical: Assess distal perfusion and nerve function in such injuries.
9) The anterior interosseous artery supplies all EXCEPT:
a) Flexor pollicis longus
b) Pronator quadratus
c) Lateral part of flexor digitorum profundus
d) Extensor digitorum communis
Explanation: The AIN supplies FPL, pronator quadratus, and lateral FDP; it does not supply extensor digitorum communis (posterior compartment). Correct answer: Extensor digitorum communis. Clinical: AIN lesions cause weak thumb and index flexion.
10) Surgical exposure of the posterior forearm should avoid injury to which vessel that pierces the interosseous membrane?
a) Radial artery
b) Posterior interosseous artery
c) Ulnar artery
d) Cephalic vein
Explanation: The posterior interosseous artery pierces the interosseous membrane to reach the posterior compartment and must be preserved during surgical approaches to avoid ischemia of extensor muscles. Correct answer: Posterior interosseous artery. Careful dissection around supinator is required.
Keyword Definitions
• Anterior interosseous nerve (AIN) – A pure motor branch of the median nerve that supplies deep forearm flexors.
• Flexor pollicis longus (FPL) – Muscle of thumb flexion at interphalangeal joint, supplied by AIN.
• Flexor digitorum profundus (FDP) – Medial part supplied by ulnar nerve, lateral part by AIN.
• Flexor digitorum superficialis (FDS) – Flexes proximal interphalangeal joints, supplied by median nerve proper (not AIN).
• Flexor carpi ulnaris (FCU) – Flexes and adducts wrist, supplied by ulnar nerve.
• Brachioradialis – Forearm flexor in mid-pronation, supplied by radial nerve.
• Abductor pollicis brevis (APB) – Thenar muscle, supplied by recurrent branch of median nerve.
• Kiloh–Nevin syndrome – Clinical syndrome due to anterior interosseous nerve palsy.
• Froment’s sign – Indicates ulnar nerve palsy when adductor pollicis is weak.
• Nerve lesions – Important in differential diagnosis of anterior compartment weakness.
Chapter: Anatomy / Upper Limb
Topic: Nerve Supply
Subtopic: Anterior Interosseous Nerve
Lead Question – 2013
All are supplied by anterior interosseous nerve except –
a) Flexor carpi ulnaris
b) Brachioradialis
c) Abductor pollicis brevis
d) Flexor pollicis longus
e) Flexor digitorum superficialis
Explanation: The anterior interosseous nerve supplies flexor pollicis longus, pronator quadratus, and the lateral half of flexor digitorum profundus. Muscles like FCU (ulnar nerve), brachioradialis (radial nerve), APB (recurrent median), and FDS (median nerve proper) are not supplied by AIN. Correct answer: All except FPL.
Guessed Questions for NEET PG
1) Anterior interosseous nerve is a branch of:
a) Ulnar nerve
b) Radial nerve
c) Median nerve
d) Musculocutaneous nerve
Explanation: The anterior interosseous nerve is a motor branch of the median nerve that arises in the proximal forearm. It runs along the interosseous membrane supplying deep flexors. Correct answer: Median nerve. It carries no cutaneous fibers, making its lesions purely motor.
2) Kiloh–Nevin syndrome refers to:
a) Ulnar nerve palsy
b) Radial nerve entrapment
c) Anterior interosseous nerve palsy
d) Posterior interosseous nerve palsy
Explanation: Kiloh–Nevin syndrome is anterior interosseous nerve palsy, presenting with inability to make the “OK sign” due to weakness of FPL and FDP (index finger). Correct answer: Anterior interosseous nerve palsy. It is often due to compression or neuritis.
3) Inability to flex thumb IP joint is seen in lesion of:
a) Median nerve at wrist
b) Ulnar nerve at elbow
c) Anterior interosseous nerve
d) Radial nerve in spiral groove
Explanation: The flexor pollicis longus, innervated by AIN, flexes thumb IP joint. Its palsy causes inability to flex the thumb tip. Correct answer: Anterior interosseous nerve. This finding is a diagnostic clue for AIN syndrome.
4) Which muscle is NOT supplied by AIN?
a) Pronator quadratus
b) FPL
c) FDP (lateral half)
d) FDS
Explanation: The anterior interosseous nerve supplies pronator quadratus, FPL, and lateral half of FDP. The FDS is supplied by the main trunk of the median nerve, not AIN. Correct answer: FDS. This helps localize nerve lesions in clinical practice.
5) Patient unable to flex index finger DIP joint likely has lesion in:
a) Radial nerve
b) Ulnar nerve
c) AIN
d) Musculocutaneous nerve
Explanation: The lateral part of flexor digitorum profundus (index and middle fingers) is supplied by AIN. Inability to flex DIP of index suggests AIN palsy. Correct answer: AIN. Ulnar supplies medial part for ring and little fingers.
6) Which test detects AIN palsy?
a) Phalen’s test
b) Froment’s sign
c) Pinch “OK” sign test
d) Tinel’s sign
Explanation: In AIN palsy, patient cannot make a round “O” with thumb and index, instead forming a triangular pinch due to weakness of FPL and FDP. Correct answer: Pinch “OK” sign test. This is diagnostic of AIN syndrome.
7) Which nerve supplies pronator quadratus?
a) Radial
b) Ulnar
c) Anterior interosseous
d) Posterior interosseous
Explanation: Pronator quadratus, a deep forearm pronator, is supplied exclusively by the anterior interosseous nerve. Correct answer: Anterior interosseous. Lesion impairs pronation, especially when forearm is flexed, and reduces grip strength.
8) A forearm fracture with isolated motor palsy (no sensory loss) indicates lesion of:
a) Ulnar nerve
b) Radial nerve
c) Anterior interosseous nerve
d) Median nerve proper
Explanation: Since the anterior interosseous nerve is a pure motor branch without cutaneous innervation, its injury causes motor weakness only. Correct answer: Anterior interosseous nerve. This differentiates it from other mixed nerves.
9) Which thenar muscle is NOT supplied by anterior interosseous nerve?
a) Abductor pollicis brevis
b) FPL
c) Opponens pollicis
d) Adductor pollicis
Explanation: Abductor pollicis brevis and other thenar muscles are supplied by the recurrent branch of the median nerve. Adductor pollicis is supplied by ulnar. Only FPL is under AIN supply. Correct answer: Abductor pollicis brevis.
10) A patient with supracondylar fracture develops inability to flex thumb IP and index DIP joints. Likely involved nerve is:
a) Radial
b) Ulnar
c) AIN
d) Musculocutaneous
Explanation: This classic presentation is due to AIN palsy following trauma, causing paralysis of FPL and FDP (index). Correct answer: AIN. Distinguishing feature is pure motor deficit with preserved cutaneous sensation.
Keyword Definitions
• Extensor compartments – Six fibro-osseous dorsal compartments on wrist that guide extensor tendons beneath extensor retinaculum.
• Extensor pollicis longus (EPL) – Tendon that extends the thumb interphalangeal joint; runs in 3rd compartment around Lister’s tubercle.
• Extensor carpi radialis longus (ECRL) – Wrist extensor and radial abductor; runs in 2nd compartment (partly).
• Extensor carpi radialis brevis (ECRB) – Wrist extensor in 2nd compartment; often involved in lateral epicondylitis.
• Extensor pollicis brevis (EPB) – Short thumb extensor in 1st compartment with APL.
• Lister’s tubercle – Dorsal tubercle of radius that acts as a pulley for EPL tendon (3rd compartment).
• De Quervain’s tenosynovitis – Stenosing tenosynovitis of 1st dorsal compartment (APL, EPB).
• Intersection syndrome – Overuse tenosynovitis where 1st compartment tendons cross 2nd compartment tendons.
• Extensor retinaculum – Fibrous band holding extensor tendons in compartments at wrist.
• Clinical importance – Identifying compartment involved helps diagnose dorsal wrist pain and plan surgical release.
Chapter: Anatomy / Upper Limb
Topic: Wrist and Hand
Subtopic: Dorsal Extensor Compartments of Wrist
Lead Question – 2013
3rd extensor compartment of wrist contains tendon of ?
a) ECRL
b) ECRB
c) EPL
d) EPB
Explanation: The third dorsal compartment contains the extensor pollicis longus (EPL) tendon as it uses Lister’s tubercle as a pulley, redirecting its line of pull to extend the thumb. Correct answer: EPL. Clinically, EPL rupture may follow distal radius fractures and presents as loss of thumb IP extension.
Guessed Questions for NEET PG
1) First dorsal compartment contains tendons of:
a) APL & EPB
b) EPL only
c) ECRL & ECRB
d) Extensor digitorum
Explanation: The first dorsal compartment contains abductor pollicis longus (APL) and extensor pollicis brevis (EPB). Stenosis here causes de Quervain’s tenosynovitis with radial wrist pain. Correct answer: APL & EPB. Treatment includes splinting or compartmental release.
2) Second dorsal compartment contains which tendons?
a) ECRL & ECRB
b) EPL only
c) APL & EPB
d) Extensor digiti minimi
Explanation: The second compartment contains extensor carpi radialis longus and brevis (ECRL, ECRB). These tendons glide under the retinaculum and are implicated in intersection syndrome when irritated by crossing first-compartment tendons. Correct answer: ECRL & ECRB.
3) Lister’s tubercle is clinically significant because it:
a) Is attachment for ECU
b) Acts as pulley for EPL tendon
c) Houses radial artery
d) Is origin of APL
Explanation: Lister’s tubercle on the distal radius acts as a dorsal pulley for the EPL tendon, changing its direction toward the thumb. After distal radius fractures, EPL attrition or rupture can occur here. Correct answer: Acts as pulley for EPL tendon.
4) De Quervain’s tenosynovitis typically presents with pain at:
a) Ulnar styloid
b) Radial styloid / lateral wrist
c) Dorsal midcarpal region
d) Pisiform area
Explanation: De Quervain’s affects APL and EPB in the first dorsal compartment causing pain and tenderness at the radial styloid. Finkelstein’s test reproduces pain. Correct answer: Radial styloid / lateral wrist. Management includes splinting and steroid injection.
5) Extensor digitorum communis (EDC) tendons lie in which compartment primarily?
a) Third
b) Fourth
c) Fifth
d) Sixth
Explanation: The fourth dorsal compartment houses the extensor digitorum communis (EDC) tendons and extensor indicis. Correct answer: Fourth. Clinical: Extensor tendon injuries in this compartment affect finger extension and may require repair or tenodesis.
6) Extensor digiti minimi runs in which compartment?
a) First
b) Second
c) Fifth
d) Sixth
Explanation: The fifth dorsal compartment contains the extensor digiti minimi tendon (to little finger). Tenosynovitis here causes localized dorsal ulnar wrist pain. Correct answer: Fifth compartment. Surgical release may be needed for refractory cases.
7) Extensor carpi ulnaris (ECU) tendon lies in which compartment?
a) Third
b) Fourth
c) Fifth
d) Sixth
Explanation: The sixth dorsal compartment contains the extensor carpi ulnaris (ECU) tendon running along the ulnar side. ECU subluxation or tendinopathy causes ulnar-sided wrist pain, especially in racket sports. Correct answer: Sixth compartment.
8) Intersection syndrome involves friction where first compartment tendons cross which compartment?
a) Second compartment tendons
b) Third compartment tendons
c) Fourth compartment tendons
d) Fifth compartment tendons
Explanation: Intersection syndrome results from friction where APL/EPB (1st compartment) cross over ECRL/ECRB (2nd compartment) about 4–8 cm proximal to wrist, producing forearm pain and crepitus. Correct answer: Second compartment tendons.
9) Rupture of EPL tendon is most commonly associated with which injury?
a) Distal radius fracture
b) Scaphoid fracture
c) Hamate fracture
d) Colles’ dislocation only
Explanation: EPL rupture classically follows distal radius fractures due to attrition at Lister’s tubercle or ischemia of the tendon sheath. Patients lose active IP extension of thumb. Correct answer: Distal radius fracture. Surgical tendon transfer may be required.
10) A swollen dorsal wrist with pain on thumb extension and positive Finkelstein’s test indicates involvement of which compartment?
a) First compartment
b) Third compartment
c) Fourth compartment
d) Sixth compartment
Explanation: Positive Finkelstein’s test with radial styloid tenderness indicates first compartment stenosing tenosynovitis (APL & EPB) — de Quervain’s disease. Correct answer: First compartment. Conservative treatment includes rest, splinting, and steroid injection.
Keyword Definitions
• Supination – Outward rotation of forearm turning palm upward.
• Pronation – Inward rotation of forearm turning palm downward.
• Biceps brachii – Flexor of elbow and chief supinator when forearm flexed.
• Supinator muscle – Assists in supination, especially when elbow extended.
• Brachioradialis – Flexes elbow, acts in mid-prone position, not supination.
• FDS (Flexor digitorum superficialis) – Flexes middle phalanges, unrelated to supination.
• Anconeus – Assists in elbow extension, stabilizes joint, not in supination.
• Musculocutaneous nerve – Innervates biceps brachii.
• Radial nerve – Innervates supinator and brachioradialis.
• Clinical test – Turning a screwdriver tests supination and biceps activity.
• Spiral groove – Radial nerve and profunda brachii pass here, relevant for supinator function.
Chapter: Anatomy / Upper Limb
Topic: Muscles of Forearm and Arm
Subtopic: Supinators of Forearm
Lead Question – 2013
Muscle causing supination of forearm?
a) Biceps brachii
b) Brachioradialis
c) FDS
d) Anconeus
Explanation: Supination is mainly caused by biceps brachii when the elbow is flexed and by supinator when the elbow is extended. Other listed muscles are not supinators. Correct answer: (a) Biceps brachii. Clinical: Weak supination occurs in musculocutaneous nerve or radial nerve injury.
Guessed Questions for NEET PG
1) Supination in extended elbow is mainly by?
a) Supinator
b) Biceps brachii
c) Pronator teres
d) Anconeus
Explanation: Supinator acts strongly when elbow is extended, while biceps dominates in flexion. Correct answer: Supinator. Clinical: Radial nerve injury weakens supination.
2) Which nerve supplies the supinator muscle?
a) Median
b) Radial (deep branch)
c) Musculocutaneous
d) Ulnar
Explanation: Supinator is supplied by deep branch of radial nerve. Correct answer: Radial nerve (deep branch). Clinical: Compression in supinator leads to posterior interosseous nerve syndrome.
3) Turning a screwdriver involves mainly?
a) Pronator quadratus
b) Biceps brachii
c) Anconeus
d) Flexor carpi radialis
Explanation: Biceps brachii provides strong supination, especially in flexed forearm. Correct answer: Biceps brachii. Clinical: Fatigue in this muscle seen in repetitive screw turning.
4) A patient with musculocutaneous nerve injury will have weakness in?
a) Supination with flexed elbow
b) Supination with extended elbow
c) Pronation
d) Wrist extension
Explanation: Musculocutaneous nerve supplies biceps brachii, the chief supinator in flexion. Correct answer: Supination with flexed elbow. Clinical: Weakness plus sensory loss lateral forearm.
5) Which movement is preserved in posterior interosseous nerve injury?
a) Finger extension
b) Supination with flexed elbow
c) Wrist extension
d) Thumb abduction
Explanation: Biceps brachii (musculocutaneous) compensates supination in flexion. Correct answer: Supination with flexed elbow. Clinical: Posterior interosseous palsy spares biceps function.
6) Which muscle is a synergist in both pronation and supination, bringing forearm to mid-prone?
a) Supinator
b) Biceps brachii
c) Brachioradialis
d) Pronator teres
Explanation: Brachioradialis brings forearm into mid-prone from either side. Correct answer: Brachioradialis. Clinical: Preserved action in radial nerve palsy proximal to its branch.
7) Supination test in clinical examination mainly evaluates?
a) Median nerve
b) Musculocutaneous nerve
c) Ulnar nerve
d) Axillary nerve
Explanation: Supination in flexion depends on biceps brachii, innervated by musculocutaneous nerve. Correct answer: Musculocutaneous nerve. Clinical: Used to assess injury after trauma.
8) Which muscle is NOT involved in forearm supination?
a) Supinator
b) Biceps brachii
c) Brachioradialis
d) FDS
Explanation: FDS is a finger flexor, has no role in forearm rotation. Correct answer: FDS. Clinical: Misconception often tested in exams.
9) In radial head dislocation (nursemaid’s elbow), which movement is restricted?
a) Pronation
b) Supination
c) Flexion
d) Extension
Explanation: Dislocated radial head impairs supination since supinator attaches here. Correct answer: Supination. Clinical: Common in children lifted by hand.
10) In fracture of surgical neck of humerus sparing biceps, supination is?
a) Lost completely
b) Weak but present
c) Normal
d) Exaggerated
Explanation: Biceps brachii (musculocutaneous) remains intact; supination is preserved. Correct answer: Normal. Clinical: Differentiates between radial and musculocutaneous lesions.
Keyword Definitions
• Musculocutaneous nerve – Terminal branch of lateral cord of brachial plexus, supplies flexors of arm.
• Brachial plexus – Nerve network supplying upper limb.
• Flexor compartment of arm – Contains biceps brachii, brachialis, and coracobrachialis.
• Biceps brachii – Flexor of elbow, supinator of forearm.
• Coracobrachialis – Flexes and adducts the arm.
• Brachialis – Primary flexor of elbow.
• Radial nerve – Supplies extensor compartment of arm.
• Median nerve – Supplies forearm and hand, not arm flexors.
• Ulnar nerve – Supplies intrinsic hand muscles and part of forearm.
• Clinical test – Elbow flexion and cutaneous sensation of lateral forearm test musculocutaneous nerve.
• Upper limb injuries – Trauma, fractures, or entrapment may affect musculocutaneous nerve function.
Chapter: Anatomy / Upper Limb
Topic: Brachial Plexus
Subtopic: Musculocutaneous Nerve and Arm Flexors
Lead Question – 2013
Nerve supply to the muscles of flexor compartment of arm?
a) Radial nerve
b) Median nerve
c) Musculocutaneous nerve
d) Ulnar nerve
Explanation: The flexor compartment of the arm (biceps brachii, brachialis, coracobrachialis) is innervated by the musculocutaneous nerve, a branch of the lateral cord of the brachial plexus. Correct answer: (c) Musculocutaneous nerve. Clinical: Injury leads to weak elbow flexion and sensory loss over lateral forearm.
Guessed Questions for NEET PG
1) Which nerve continues as the lateral cutaneous nerve of forearm?
a) Radial
b) Median
c) Musculocutaneous
d) Ulnar
Explanation: Musculocutaneous nerve ends as lateral cutaneous nerve of forearm. Correct answer: Musculocutaneous nerve. Clinical: Injury causes sensory loss in lateral forearm.
2) Which muscle is pierced by musculocutaneous nerve?
a) Biceps brachii
b) Coracobrachialis
c) Brachialis
d) Deltoid
Explanation: Musculocutaneous nerve pierces coracobrachialis before supplying flexor compartment. Correct answer: Coracobrachialis. Clinical: Landmark for nerve tracing.
3) Which muscle in flexor compartment also receives supply from radial nerve?
a) Biceps brachii
b) Coracobrachialis
c) Brachialis
d) None
Explanation: Brachialis is mainly supplied by musculocutaneous nerve, but radial nerve gives additional innervation. Correct answer: Brachialis. Clinical: Explains preserved flexion in musculocutaneous injury.
4) Elbow flexion against resistance tests mainly?
a) Median nerve
b) Musculocutaneous nerve
c) Ulnar nerve
d) Axillary nerve
Explanation: Biceps brachii and brachialis, innervated by musculocutaneous nerve, are prime elbow flexors. Correct answer: Musculocutaneous nerve. Clinical: Used to check function.
5) Which branch of brachial plexus gives rise to musculocutaneous nerve?
a) Lateral cord
b) Posterior cord
c) Medial cord
d) Upper trunk
Explanation: Musculocutaneous nerve arises from the lateral cord (C5–C7 roots). Correct answer: Lateral cord. Clinical: Knowledge useful in brachial plexus blocks.
6) Sensory loss over lateral forearm is due to injury of?
a) Radial nerve
b) Musculocutaneous nerve
c) Median nerve
d) Ulnar nerve
Explanation: Lateral cutaneous nerve of forearm (continuation of musculocutaneous) supplies skin here. Correct answer: Musculocutaneous nerve. Clinical: Sensory deficit confirms diagnosis.
7) Which movement is most affected in musculocutaneous nerve injury?
a) Shoulder abduction
b) Elbow flexion
c) Wrist extension
d) Finger flexion
Explanation: Musculocutaneous injury impairs elbow flexion due to paralysis of biceps and brachialis. Correct answer: Elbow flexion. Clinical: Weak supination also observed.
8) A patient with injury to musculocutaneous nerve will show weakness of?
a) Forearm pronation
b) Elbow flexion
c) Finger extension
d) Thumb opposition
Explanation: Injury leads to loss of flexors of arm, causing weak elbow flexion. Correct answer: Elbow flexion. Clinical: Supination also affected due to biceps involvement.
9) Which muscle is absent in flexor compartment if musculocutaneous nerve is injured?
a) Deltoid
b) Coracobrachialis
c) Triceps
d) Supinator
Explanation: Coracobrachialis is supplied by musculocutaneous nerve and loses function in its injury. Correct answer: Coracobrachialis. Clinical: Shoulder adduction and flexion weakened.
10) Which of the following is NOT supplied by musculocutaneous nerve?
a) Biceps brachii
b) Brachialis
c) Coracobrachialis
d) Triceps brachii
Explanation: Triceps brachii belongs to extensor compartment, supplied by radial nerve. Correct answer: Triceps brachii. Clinical: Preserved triceps function rules out musculocutaneous injury.
Keyword Definitions
• Axillary nerve – Branch of posterior cord of brachial plexus, supplies deltoid and teres minor.
• Quadrangular space – Anatomical space transmitting axillary nerve and posterior circumflex humeral artery.
• Deltoid paralysis – Clinical feature of axillary nerve injury, causing loss of shoulder abduction.
• Humeral surgical neck – Common fracture site leading to axillary nerve damage.
• Circumflex humeral arteries – Branches of axillary artery encircling humerus.
• Teres minor – Rotator cuff muscle innervated by axillary nerve.
• Shoulder dislocation – Can injure axillary nerve.
• Posterior circumflex humeral artery – Runs with axillary nerve in quadrangular space.
• Clinical test – Abduction and sensation over regimental badge area for axillary nerve integrity.
• Brachial plexus – Nerve network supplying upper limb.
• Surgical relevance – Axillary nerve at risk during deltoid intramuscular injections.
Chapter: Anatomy / Upper Limb
Topic: Brachial Plexus
Subtopic: Axillary Nerve and Vessels
Lead Question – 2013
Axillary nerve is accompanied by which artery?
a) Axillary
b) Subscapular
c) Anterior circumflex humeral
d) Posterior circumflex humeral
Explanation: Axillary nerve passes through the quadrangular space along with the posterior circumflex humeral artery. This anatomical relationship is clinically important during humeral neck fractures or shoulder dislocations. Correct answer: (d) Posterior circumflex humeral artery. Clinical: Injury leads to deltoid weakness and sensory loss over regimental badge area.
Guessed Questions for NEET PG
1) Which space transmits axillary nerve and posterior circumflex humeral artery?
a) Triangular space
b) Quadrangular space
c) Cubital fossa
d) Axilla
Explanation: Axillary nerve and posterior circumflex humeral artery pass through quadrangular space. Correct answer: Quadrangular space. Clinical: Compression here can cause axillary neuropathy.
2) Fracture of surgical neck of humerus most likely injures?
a) Radial nerve
b) Axillary nerve
c) Median nerve
d) Ulnar nerve
Explanation: Surgical neck fracture endangers axillary nerve and posterior circumflex humeral artery. Correct answer: Axillary nerve. Clinical: Presents with deltoid atrophy and shoulder abduction weakness.
3) Which muscle is NOT supplied by axillary nerve?
a) Deltoid
b) Teres minor
c) Teres major
d) Skin over regimental badge
Explanation: Teres major is supplied by subscapular nerve, not axillary nerve. Correct answer: Teres major. Clinical: Differentiates axillary nerve palsy from broader plexus injury.
4) Loss of sensation over regimental badge area indicates injury to?
a) Radial nerve
b) Axillary nerve
c) Suprascapular nerve
d) Musculocutaneous nerve
Explanation: Axillary nerve injury causes sensory deficit over regimental badge area. Correct answer: Axillary nerve. Clinical: Pathognomonic for axillary neuropathy.
5) Which rotator cuff muscle is innervated by axillary nerve?
a) Supraspinatus
b) Infraspinatus
c) Teres minor
d) Subscapularis
Explanation: Teres minor is the only rotator cuff muscle supplied by axillary nerve. Correct answer: Teres minor. Clinical: Weakness in external rotation occurs in axillary nerve injury.
6) During deltoid intramuscular injection, which nerve is at risk?
a) Radial nerve
b) Median nerve
c) Axillary nerve
d) Musculocutaneous nerve
Explanation: Axillary nerve runs deep to deltoid; incorrect needle placement may injure it. Correct answer: Axillary nerve. Clinical: Presents with deltoid weakness.
7) Posterior circumflex humeral artery is a branch of?
a) Brachial artery
b) Axillary artery
c) Subclavian artery
d) Radial artery
Explanation: Posterior circumflex humeral artery arises from the 3rd part of axillary artery. Correct answer: Axillary artery. Clinical: Injured in humeral neck fractures.
8) Which movement is most affected in axillary nerve injury?
a) Elbow flexion
b) Shoulder abduction
c) Wrist extension
d) Thumb opposition
Explanation: Deltoid paralysis impairs shoulder abduction beyond 15 degrees. Correct answer: Shoulder abduction. Clinical: Differentiates from supraspinatus injury which initiates abduction.
9) Which clinical test best evaluates axillary nerve function?
a) Flexion of elbow
b) Abduction of shoulder against resistance
c) Extension of wrist
d) Pronation of forearm
Explanation: Abduction of shoulder against resistance tests deltoid function supplied by axillary nerve. Correct answer: Shoulder abduction against resistance. Clinical: Standard examination method.
10) Anterior dislocation of shoulder commonly injures?
a) Radial nerve
b) Axillary nerve
c) Median nerve
d) Ulnar nerve
Explanation: Axillary nerve lies close to shoulder joint and is frequently injured in anterior dislocation. Correct answer: Axillary nerve. Clinical: Presents with deltoid atrophy and regimental badge anesthesia.
Keyword Definitions
• Midpalmar space – Deep fascial space of hand located beneath central compartment, communicates with forearm via carpal tunnel.
• Lumbricals – Four small intrinsic hand muscles arising from flexor digitorum profundus tendons.
• FDP – Flexor digitorum profundus, flexes distal interphalangeal joints.
• Thenar space – Fascial space near thumb, separated from midpalmar space by septum.
• Hand compartments – Thenar, hypothenar, adductor, central, interosseous compartments.
• Clinical correlation – Midpalmar abscess can spread to forearm through carpal tunnel.
• Carpal tunnel – Passage for FDP, FDS tendons, FPL tendon, and median nerve.
• Infection spread – From finger pulp to midpalmar space via lumbrical canals.
• Surgical drainage – Important in treating deep palmar space infections.
• Lumbrical canal – Interval through which lumbrical muscles enter palm from FDP.
• Interossei – Intrinsic hand muscles not part of midpalmar space content.
Chapter: Anatomy / Upper Limb
Topic: Hand
Subtopic: Midpalmar Space
Lead Question – 2013
Contents of midpalmar space are all except
a) 2nd lumbrical
b) FDP of 3rd finger
c) 1st lumbrical
d) FDP of 4th finger
Explanation: Midpalmar space contains medial three lumbricals and flexor digitorum profundus tendons of middle, ring, and little fingers. The 1st lumbrical belongs to thenar space, not midpalmar. Correct answer: (c) 1st lumbrical. Clinical: Midpalmar abscess can cause swelling in central palm and requires careful surgical drainage.
Guessed Questions for NEET PG
1) Which lumbrical muscle lies in thenar space?
a) 1st lumbrical
b) 2nd lumbrical
c) 3rd lumbrical
d) 4th lumbrical
Explanation: The 1st lumbrical lies in thenar space along with FPL tendon. Others lie in midpalmar space. Correct answer: 1st lumbrical. Clinical: Swelling of thenar space may compromise thumb movements.
2) Infection from the index finger pulp may spread to?
a) Thenar space
b) Midpalmar space
c) Hypothenar space
d) Dorsum of hand
Explanation: Lumbrical canal of index finger connects pulp with thenar space. Correct answer: Thenar space. Clinical: Early drainage is essential to preserve thumb function.
3) Which tendon passes through carpal tunnel and continues into midpalmar space?
a) FPL
b) FDP
c) Extensor digitorum
d) Palmaris longus
Explanation: Flexor digitorum profundus tendons pass through carpal tunnel and form part of midpalmar space contents. Correct answer: FDP. Clinical: Infections may spread from palm to forearm via carpal tunnel.
4) Midpalmar space infection presents with swelling in which region?
a) Thenar eminence
b) Hypothenar eminence
c) Central palm
d) Dorsal web space
Explanation: Midpalmar space infections typically cause fullness in central palm. Correct answer: Central palm. Clinical: Deep abscess requires surgical drainage through palmar incision.
5) Which of the following is not a boundary of midpalmar space?
a) Palmar aponeurosis
b) Metacarpals of index finger
c) Interossei muscles
d) Flexor pollicis longus
Explanation: FPL belongs to thenar space, not boundary of midpalmar space. Correct answer: FPL. Clinical: Differentiating compartments is crucial in infection management.
6) Which lumbricals are supplied by ulnar nerve?
a) 1st and 2nd
b) 3rd and 4th
c) All four
d) None
Explanation: The 3rd and 4th lumbricals are supplied by the deep branch of ulnar nerve. Correct answer: 3rd and 4th. Clinical: Ulnar nerve injury affects fine grip due to lumbrical paralysis.
7) Patient with midpalmar space infection is unable to flex distal phalanx of ring finger. Which tendon is affected?
a) FDS
b) FDP
c) FPL
d) Extensor indicis
Explanation: FDP of ring finger passes through midpalmar space, infection can impair its function. Correct answer: FDP. Clinical: This indicates deep involvement requiring urgent drainage.
8) Which muscle group borders the midpalmar space dorsally?
a) Palmaris brevis
b) Interossei
c) Thenar muscles
d) Hypothenar muscles
Explanation: Interossei muscles form dorsal boundary of midpalmar space. Correct answer: Interossei. Clinical: Infection here can spread to intermetacarpal spaces.
9) Midpalmar space communicates with forearm via?
a) Anatomical snuffbox
b) Carpal tunnel
c) Guyon’s canal
d) Radial bursa
Explanation: Midpalmar space communicates proximally through carpal tunnel with forearm. Correct answer: Carpal tunnel. Clinical: Explains spread of deep hand infections to forearm flexor sheath.
10) Which space is affected in a patient unable to oppose thumb with swelling near 1st web space?
a) Midpalmar space
b) Thenar space
c) Hypothenar space
d) Dorsal space
Explanation: Thenar space infection affects thumb opposition and causes swelling near first web space. Correct answer: Thenar space. Clinical: Misdiagnosis may lead to permanent disability of thumb.
Keyword Definitions
• Superficial anterior compartment of forearm – Contains five muscles: pronator teres, flexor carpi radialis, palmaris longus, flexor carpi ulnaris, flexor digitorum superficialis.
• FDS – Flexor digitorum superficialis, a superficial flexor of fingers.
• FCR – Flexor carpi radialis, wrist flexor and abductor.
• Palmaris longus – Weak wrist flexor, absent in 10-15% individuals.
• FPL – Flexor pollicis longus, belongs to deep compartment, flexes thumb.
• Median nerve – Supplies most superficial anterior forearm muscles.
• Ulnar nerve – Supplies flexor carpi ulnaris and medial half of FDP.
• Brachial artery – Main arterial supply of forearm, divides into radial and ulnar.
• Clinical correlation – Superficial muscles are commonly involved in tendinitis and occupational overuse syndromes.
• Compartment syndrome – Increased pressure in forearm compartments can damage muscles and nerves.
• Pronator teres syndrome – Median nerve entrapment by pronator teres in superficial compartment.
Chapter: Anatomy / Upper Limb
Topic: Forearm Muscles
Subtopic: Superficial anterior compartment of forearm
Lead Question – 2013
Which of the following is not the muscle of superficial anterior compartment of forearm?
a) FDS
b) FPL
c) FCR
d) Palmaris longus
Explanation: The superficial anterior compartment includes pronator teres, flexor carpi radialis, palmaris longus, flexor carpi ulnaris, and flexor digitorum superficialis. Flexor pollicis longus is a deep compartment muscle. Correct answer: (b) FPL. Clinical: FPL is important in thumb flexion and tested in anterior interosseous nerve palsy.
Guessed Questions for NEET PG
1) Which of the following is a superficial flexor of forearm?
a) Pronator teres
b) Flexor pollicis longus
c) Flexor digitorum profundus
d) Supinator
Explanation: Pronator teres belongs to superficial anterior compartment. FPL and FDP are deep flexors, supinator belongs to posterior compartment. Correct answer: Pronator teres. Clinical: Median nerve may be compressed between its two heads.
2) Which superficial anterior forearm muscle is most frequently absent in population?
a) FCR
b) FCU
c) Palmaris longus
d) Pronator teres
Explanation: Palmaris longus is absent in about 10–15% of individuals. Correct answer: Palmaris longus. Clinical: Its tendon is used in reconstructive tendon graft surgeries without functional deficit.
3) Which muscle is supplied by ulnar nerve among superficial flexors?
a) FCR
b) FCU
c) Palmaris longus
d) Pronator teres
Explanation: Flexor carpi ulnaris is the only superficial flexor supplied by the ulnar nerve. Others are supplied by median nerve. Correct answer: FCU. Clinical: Weak wrist flexion and ulnar deviation occur in lesions.
4) A patient unable to flex PIP joints of fingers likely has paralysis of?
a) FDP
b) FDS
c) FPL
d) FCU
Explanation: Flexor digitorum superficialis flexes proximal interphalangeal joints. Correct answer: FDS. Clinical: Median nerve lesions at elbow impair this function.
5) Which muscle originates from common flexor origin on medial epicondyle?
a) Palmaris longus
b) Pronator teres
c) FCR
d) All of the above
Explanation: All superficial flexors except FDS deep head arise from medial epicondyle via common flexor origin. Correct answer: All of the above. Clinical: Overuse may cause medial epicondylitis (golfer’s elbow).
6) Flexor pollicis longus is supplied by?
a) Median nerve (anterior interosseous branch)
b) Ulnar nerve
c) Radial nerve
d) Posterior interosseous nerve
Explanation: FPL is innervated by anterior interosseous branch of median nerve. Correct answer: Median nerve (AIN). Clinical: AIN palsy causes inability to make “OK” sign due to FPL weakness.
7) Which superficial flexor muscle is involved in carpal tunnel syndrome symptoms due to tendinopathy?
a) FDS
b) FCR
c) FCU
d) Pronator teres
Explanation: Flexor digitorum superficialis tendons pass through carpal tunnel and may contribute to compression. Correct answer: FDS. Clinical: CTS presents with numbness, tingling in lateral fingers.
8) Median nerve lies deep to which superficial muscle at wrist?
a) FCU
b) FCR
c) Palmaris longus
d) Pronator teres
Explanation: Palmaris longus tendon lies superficial to median nerve at wrist. Correct answer: Palmaris longus. Clinical: Median nerve blocks may be guided by this relation.
9) Which superficial forearm muscle flexes and abducts the wrist?
a) FCR
b) FCU
c) Palmaris longus
d) FDS
Explanation: Flexor carpi radialis flexes wrist and abducts it towards radial side. Correct answer: FCR. Clinical: FCR tendon is palpable in radial wrist and used for arterial cannulation landmark.
10) In anterior compartment syndrome, which superficial muscle is most vulnerable due to location?
a) Pronator teres
b) FCR
c) FCU
d) FDS
Explanation: Flexor digitorum superficialis, being central and bulky, is commonly affected in increased compartment pressure. Correct answer: FDS. Clinical: Early decompression prevents ischemic contracture (Volkmann’s).
Keyword Definitions
• Dual nerve supply – Muscle receiving motor innervation from two different nerves.
• Subscapularis – Supplied by upper and lower subscapular nerves.
• Pectoralis major – Supplied by medial and lateral pectoral nerves.
• Pronator teres – Supplied by median nerve only.
• Flexor digitorum profundus – Medial half by ulnar nerve, lateral half by anterior interosseous branch of median nerve.
• Brachial plexus – Network of nerves supplying upper limb, roots C5–T1.
• Median nerve – Formed by medial and lateral cords, supplies most forearm flexors.
• Ulnar nerve – Arises from medial cord, supplies intrinsic hand muscles and medial FDP.
• Subscapular nerves – Branches of posterior cord, innervate subscapularis.
• Clinical correlation – Knowledge of dual supply important in nerve lesions and recovery.
• Muscle palsy – Weakness pattern helps localize lesion to specific nerve or part of plexus.
Chapter: Anatomy / Upper Limb
Topic: Brachial Plexus and Muscle Innervation
Subtopic: Dual nerve supply of upper limb muscles
Lead Question – 2013
All of the following muscles have dual nerve supply except?
a) Subscapularis
b) Pectoralis major
c) Pronator teres
d) Flexor digitorum profundus
Explanation: Subscapularis has dual supply (upper and lower subscapular nerves). Pectoralis major has dual supply (medial and lateral pectoral nerves). FDP has dual supply (median and ulnar nerves). Pronator teres has single supply (median nerve). Correct answer: (c) Pronator teres. Clinical: Isolated median injury can paralyze pronator teres completely.
Guessed Questions for NEET PG
1) Which muscle among the following is supplied by both ulnar and median nerves?
a) Flexor pollicis longus
b) Flexor carpi radialis
c) Flexor digitorum profundus
d) Pronator quadratus
Explanation: Flexor digitorum profundus has dual supply – medial half by ulnar, lateral half by anterior interosseous (median). Correct answer: FDP. Clinical: Explains partial preservation in isolated lesions.
2) Subscapularis is supplied by?
a) Upper and lower subscapular nerves
b) Thoracodorsal nerve
c) Lateral pectoral nerve
d) Axillary nerve
Explanation: Subscapularis is innervated by both upper and lower subscapular nerves from posterior cord. Correct answer: Upper and lower subscapular nerves. Clinical: Injury leads to weak internal rotation.
3) Which muscle receives innervation from both medial and lateral pectoral nerves?
a) Pectoralis major
b) Pectoralis minor
c) Subclavius
d) Serratus anterior
Explanation: Pectoralis major is supplied by medial and lateral pectoral nerves. Correct answer: Pectoralis major. Clinical: Paralysis leads to weak adduction and internal rotation.
4) A patient with ulnar nerve lesion at wrist retains partial flexion of DIP of ring finger due to?
a) Median nerve supply
b) Radial nerve supply
c) Musculocutaneous nerve supply
d) Axillary nerve supply
Explanation: Lateral half of FDP (index and middle fingers) supplied by median, medial half (ring and little fingers) by ulnar. Correct answer: Median nerve supply. Clinical: Explains incomplete loss in ulnar palsy.
5) Which of the following has single nerve supply?
a) FDP
b) Pectoralis major
c) Pronator teres
d) Subscapularis
Explanation: Pronator teres is solely supplied by median nerve. Others have dual innervation. Correct answer: Pronator teres. Clinical: Useful in lesion localization.
6) Which nerve supplies medial half of flexor digitorum profundus?
a) Ulnar
b) Median
c) Radial
d) Musculocutaneous
Explanation: Medial half (ring and little fingers) of FDP is innervated by ulnar nerve. Correct answer: Ulnar nerve. Clinical: Explains weakness of DIP flexion in ulnar palsy.
7) Damage to lateral pectoral nerve causes weakness in?
a) Shoulder abduction
b) Arm adduction
c) Elbow flexion
d) Wrist extension
Explanation: Lateral pectoral nerve innervates pectoralis major, main action is adduction and internal rotation of arm. Correct answer: Arm adduction. Clinical: Loss of powerful adduction in lesion.
8) A patient with lesion of posterior cord affecting both upper and lower subscapular nerves shows weakness in?
a) Internal rotation
b) External rotation
c) Abduction
d) Supination
Explanation: Subscapularis performs internal rotation of humerus, supplied by both upper and lower subscapular nerves. Correct answer: Internal rotation. Clinical: Shoulder stability is also reduced.
9) Median nerve injury at elbow spares which of the following?
a) Pronator teres
b) Flexor digitorum profundus (medial half)
c) Flexor digitorum superficialis
d) Flexor pollicis longus
Explanation: Medial half of FDP is supplied by ulnar nerve, hence spared in median nerve injury at elbow. Correct answer: FDP (medial half). Clinical: Explains partial preservation of finger flexion.
10) Which of the following combinations represent dual innervation correctly?
a) FDP – Median & Ulnar
b) Pectoralis major – Medial & Lateral pectoral
c) Subscapularis – Upper & Lower subscapular
d) All of the above
Explanation: All mentioned muscles are examples of dual innervation. Correct answer: All of the above. Clinical: Important for understanding muscle function in partial nerve injuries.
Keyword Definitions
• Profunda brachii artery – Deep artery of arm, branch of brachial artery, runs in spiral groove.
• Spiral groove – Shallow groove on posterior humerus, occupied by radial nerve and profunda brachii artery.
• Radial nerve – Continuation of posterior cord of brachial plexus, supplies extensor compartment.
• Ulnar nerve – Arises from medial cord, passes behind medial epicondyle, supplies intrinsic hand muscles.
• Median nerve – Formed from medial and lateral cords, passes through carpal tunnel, major flexor nerve.
• Humeral shaft fracture – Common injury damaging radial nerve in spiral groove.
• Wrist drop – Clinical sign of radial nerve injury, due to loss of extensor muscle function.
• Saturday night palsy – Radial nerve compression neuropathy in spiral groove.
• Deep brachial artery – Synonym for profunda brachii artery, accompanies radial nerve.
• Extensor compartment – Muscles of posterior arm and forearm controlled by radial nerve.
• Clinical correlation – Spiral groove relation important in fractures and compressive neuropathies.
Chapter: Anatomy / Upper Limb
Topic: Arm and Brachial Plexus
Subtopic: Radial nerve and profunda brachii artery in spiral groove
Lead Question – 2013
Nerve running along with profunda brachii artery, in spiral groove?
a) Ulnar
b) Median
c) Radial
d) None
Explanation: The radial nerve runs along with the profunda brachii artery in the spiral groove of the humerus. This relationship is clinically significant as humeral shaft fractures can injure both structures. Correct answer: (c) Radial. Clinical: Injury causes wrist drop and sensory loss over dorsum of hand.
Guessed Questions for NEET PG
1) A mid-shaft fracture of humerus most commonly injures?
a) Median nerve
b) Radial nerve
c) Ulnar nerve
d) Musculocutaneous nerve
Explanation: Mid-shaft humeral fractures frequently damage the radial nerve as it lies in the spiral groove. Correct answer: Radial nerve. Clinical: Presents with wrist drop and loss of finger extension.
2) Which muscle is first affected in radial nerve palsy at spiral groove?
a) Triceps
b) Anconeus
c) Brachioradialis
d) Extensor carpi radialis longus
Explanation: Triceps is spared in spiral groove lesions. Brachioradialis and wrist extensors are first affected. Correct answer: Brachioradialis. Clinical: Weak elbow flexion in mid-pronation position.
3) Saturday night palsy refers to?
a) Ulnar nerve compression
b) Radial nerve compression
c) Median nerve compression
d) Axillary nerve compression
Explanation: Saturday night palsy occurs when prolonged compression damages the radial nerve in spiral groove during deep sleep or intoxication. Correct answer: Radial nerve compression. Clinical: Wrist drop with sensory loss.
4) Sensory loss in radial nerve injury at spiral groove involves?
a) Thenar eminence
b) Dorsum of first web space
c) Medial forearm
d) Palmar little finger
Explanation: Spiral groove injury spares triceps but causes sensory loss over dorsum of hand, particularly first web space. Correct answer: Dorsum of first web space. Clinical: Important diagnostic clue.
5) Which branch of radial nerve supplies triceps?
a) Posterior cutaneous nerve
b) Muscular branches
c) Deep branch
d) Superficial branch
Explanation: Muscular branches of radial nerve supply triceps before entering spiral groove. Correct answer: Muscular branches. Clinical: Triceps preserved in spiral groove lesions.
6) Which artery is at risk with humeral shaft fracture along with radial nerve?
a) Brachial artery
b) Profunda brachii artery
c) Radial artery
d) Ulnar artery
Explanation: Profunda brachii artery accompanies radial nerve in spiral groove, making it vulnerable in shaft fractures. Correct answer: Profunda brachii artery. Clinical: Bleeding complicates fracture management.
7) Which test best detects radial nerve palsy?
a) Asking patient to oppose thumb
b) Asking patient to extend wrist
c) Asking patient to flex DIP of index
d) Asking patient to abduct little finger
Explanation: Wrist extension is controlled by radial nerve. In palsy, patient cannot extend wrist, producing wrist drop. Correct answer: Wrist extension test. Clinical: Pathognomonic finding.
8) Wrist drop occurs due to paralysis of?
a) Flexor muscles
b) Extensor muscles
c) Pronator muscles
d) Intrinsic hand muscles
Explanation: Radial nerve injury paralyzes extensor muscles of forearm, causing wrist drop. Correct answer: Extensor muscles. Clinical: Patient presents with inability to extend wrist and fingers.
9) Which part of triceps is usually spared in spiral groove lesion?
a) Long head
b) Lateral head
c) Medial head
d) All heads affected
Explanation: Radial nerve supplies long and lateral heads of triceps before entering spiral groove, hence spared. Medial head may be affected. Correct answer: Long and lateral heads spared. Clinical: Partial triceps weakness only.
10) In radial nerve injury, supination is preserved due to action of?
a) Biceps brachii
b) Supinator
c) Pronator teres
d) Brachialis
Explanation: Supination is performed by supinator (radial nerve) and biceps brachii (musculocutaneous nerve). Even if radial nerve is injured, biceps maintains supination. Correct answer: Biceps brachii. Clinical: Supination relatively preserved in palsy.
Keyword Definitions
• Pectoral region – The area on the anterior chest wall related to pectoral muscles.
• Pectoralis major – Large superficial chest muscle aiding arm adduction and medial rotation.
• Pectoralis minor – Lies deep to pectoralis major, attaches coracoid process to ribs.
• Subclavius – Small muscle beneath clavicle, stabilizes clavicle.
• Infraspinatus – Rotator cuff muscle of scapula, not part of pectoral region.
• Axilla – Space beneath pectoral region transmitting vessels and nerves.
• Thoracoacromial artery – Main arterial supply of pectoral region.
• Medial and lateral pectoral nerves – Innervation of pectoralis muscles.
• Clinical correlation – Trauma or surgery of chest may injure pectoral nerves.
• Rotator cuff – Group of muscles stabilizing shoulder joint, includes infraspinatus.
• Breast surgery relevance – Pectoralis muscles form bed of breast and are key in mastectomy.
Chapter: Anatomy / Upper Limb
Topic: Pectoral Region and Axilla
Subtopic: Muscles of pectoral region
Lead Question – 2013
Which of the following muscle is not in the pectoral region?
a) Pectoralis major
b) Infraspinatus
c) Pectoralis minor
d) Subclavius
Explanation: The pectoral region consists of pectoralis major, pectoralis minor, and subclavius. Infraspinatus is located on posterior aspect of scapula, part of rotator cuff, not pectoral region. Correct answer: (b) Infraspinatus. Clinical: Important to distinguish anterior chest muscles from posterior scapular muscles.
Guessed Questions for NEET PG
1) Which nerve supplies pectoralis major?
a) Thoracodorsal nerve
b) Medial and lateral pectoral nerves
c) Axillary nerve
d) Suprascapular nerve
Explanation: Pectoralis major is supplied by both medial and lateral pectoral nerves. Correct answer: Medial and lateral pectoral nerves. Clinical: Nerve injury may weaken adduction and medial rotation of arm.
2) Which structure lies deep to pectoralis minor?
a) Brachial plexus cords
b) Cephalic vein
c) Basilic vein
d) Ulnar nerve
Explanation: Pectoralis minor is a landmark for cords of brachial plexus and axillary vessels. Correct answer: Brachial plexus cords. Clinical: Used as a guide in axillary dissections.
3) Subclavius muscle function is?
a) Elevates scapula
b) Depresses clavicle
c) Flexes humerus
d) Extends arm
Explanation: Subclavius depresses and stabilizes the clavicle during shoulder movements. Correct answer: Depresses clavicle. Clinical: Provides protection to subclavian vessels during clavicular fracture.
4) Which artery mainly supplies pectoralis major?
a) Subclavian artery
b) Thoracoacromial artery
c) Radial artery
d) Subscapular artery
Explanation: The thoracoacromial artery, a branch of axillary artery, supplies pectoralis major. Correct answer: Thoracoacromial artery. Clinical: Important during reconstructive flap surgeries.
5) A breast carcinoma infiltrating deep fascia can involve which muscle first?
a) Serratus anterior
b) Pectoralis major
c) Infraspinatus
d) Latissimus dorsi
Explanation: Pectoralis major forms the bed of breast, hence infiltrated in advanced carcinoma. Correct answer: Pectoralis major. Clinical: Explains fixation of breast mass to chest wall.
6) Which muscle is part of rotator cuff but not pectoral region?
a) Subscapularis
b) Infraspinatus
c) Supraspinatus
d) Teres minor
Explanation: Infraspinatus is part of rotator cuff, not pectoral region. Correct answer: Infraspinatus. Clinical: Weakness causes loss of external rotation at shoulder.
7) Injury to medial pectoral nerve causes weakness of?
a) Deltoid
b) Pectoralis minor
c) Latissimus dorsi
d) Teres major
Explanation: Medial pectoral nerve supplies pectoralis minor and part of pectoralis major. Correct answer: Pectoralis minor. Clinical: May weaken scapular protraction.
8) Which lymph nodes lie deep to pectoralis minor?
a) Apical
b) Central
c) Lateral
d) Subscapular
Explanation: Apical group of axillary lymph nodes lie deep to pectoralis minor. Correct answer: Apical. Clinical: Important in breast cancer spread and axillary clearance surgeries.
9) A patient with winging of scapula likely has injury to?
a) Medial pectoral nerve
b) Long thoracic nerve
c) Axillary nerve
d) Suprascapular nerve
Explanation: Winging of scapula occurs due to serratus anterior paralysis from long thoracic nerve injury. Correct answer: Long thoracic nerve. Clinical: Seen in radical mastectomy complications.
10) Which of the following is not a muscle of anterior axillary fold?
a) Pectoralis major
b) Pectoralis minor
c) Subclavius
d) Latissimus dorsi
Explanation: Anterior axillary fold is formed by lower border of pectoralis major. Latissimus dorsi forms posterior fold. Correct answer: Latissimus dorsi. Clinical: Used in surgical identification of axillary folds.
Keyword Definitions
• Bursa – Fluid-filled sac reducing friction between tendon and bone.
• Synovial sheath – Tubular bursa surrounding a tendon for smooth gliding.
• Radial bursa – Synovial sheath enclosing flexor pollicis longus tendon.
• Ulnar bursa – Common flexor sheath for FDP and FDS tendons.
• Flexor pollicis longus (FPL) – Muscle flexing thumb distal phalanx.
• Flexor digitorum profundus (FDP) – Muscle flexing distal phalanges of fingers.
• Flexor digitorum superficialis (FDS) – Muscle flexing middle phalanges.
• Flexor carpi radialis (FCR) – Wrist flexor inserting into 2nd metacarpal.
• Thenar space – Potential space in palm communicating with radial bursa.
• Midpalmar space – Potential space in palm communicating with ulnar bursa.
• Clinical relevance – Infections of synovial sheaths may spread rapidly to palm and forearm.
Chapter: Anatomy / Upper Limb
Topic: Hand and Forearm Structures
Subtopic: Synovial Sheaths and Bursae of Hand
Lead Question – 2013
Radial bursa is the synovial sheath covering the tendon of ?
a) FDS
b) FDP
c) FPL
d) FCR
Explanation: The radial bursa is the synovial sheath of flexor pollicis longus (FPL) tendon. It extends from wrist into the thumb. Correct answer: FPL. Clinical: infection here (tenosynovitis) may spread into the forearm and cause “horseshoe abscess” by communicating with the ulnar bursa.
Guessed Questions for NEET PG
1) Ulnar bursa covers tendons of:
a) FPL
b) FDP and FDS
c) FCR
d) EPL
Explanation: Ulnar bursa is the common flexor sheath enclosing tendons of FDP and FDS to fingers. Correct answer: FDP and FDS. Clinical: infections here can spread into midpalmar space, causing swelling and impaired finger movements.
2) Horseshoe abscess occurs due to communication between:
a) Radial and ulnar bursa
b) Ulnar bursa and carpal tunnel
c) Radial bursa and midpalmar space
d) Thenar and hypothenar spaces
Explanation: Radial bursa of thumb communicates with ulnar bursa of little finger, producing a characteristic “horseshoe-shaped abscess.” Correct answer: Radial and ulnar bursa. Clinical: requires early drainage to prevent spread to forearm.
3) Infection of thumb flexor tendon sheath may spread into:
a) Thenar space
b) Midpalmar space
c) Parona’s space
d) Dorsum of hand
Explanation: FPL tendon sheath infection spreads through radial bursa into Parona’s space (forearm). Correct answer: Parona’s space. Clinical: severe swelling of forearm seen in advanced tenosynovitis.
4) Which tendon passes separately in its own sheath within carpal tunnel?
a) FPL
b) FDP
c) FDS
d) Palmaris longus
Explanation: FPL passes in its own synovial sheath (radial bursa) through the carpal tunnel. Correct answer: FPL. Clinical: inflammation here may cause isolated thumb pain in carpal tunnel syndrome.
5) Parona’s space is located:
a) Between palmar aponeurosis and flexor tendons
b) Between pronator quadratus and flexor tendons
c) In dorsal hand
d) In thenar eminence
Explanation: Parona’s space is between pronator quadratus and flexor tendons in distal forearm. Correct answer: Between pronator quadratus and flexor tendons. Clinical: serves as pathway for spread of infection from radial or ulnar bursa.
6) Which of the following muscles inserts into the distal phalanx of thumb?
a) FPL
b) FDS
c) FDP
d) EPL
Explanation: Flexor pollicis longus (FPL) inserts into the base of distal phalanx of thumb, flexing IP joint. Correct answer: FPL. Clinical: important in pinch grip strength, loss indicates anterior interosseous nerve palsy.
7) Ulnar bursa commonly extends up to which finger?
a) Index
b) Middle
c) Ring
d) Little
Explanation: The ulnar bursa extends into the little finger flexor sheath. Correct answer: Little finger. Clinical: explains why infections of little finger flexor sheath can spread to common flexor sheath and palm.
8) Which structure is enclosed within both radial bursa and carpal tunnel?
a) FCR
b) FPL
c) EPL
d) Lumbricals
Explanation: FPL tendon passes through carpal tunnel inside its radial bursa sheath. Correct answer: FPL. Clinical: tenosynovitis here may mimic carpal tunnel syndrome with isolated thumb symptoms.
9) A 25-year-old presents with swelling of thumb and little finger tendon sheaths with forearm spread. Most likely condition?
a) Thenar abscess
b) Midpalmar abscess
c) Horseshoe abscess
d) Carpal tunnel syndrome
Explanation: Simultaneous infection of radial and ulnar bursae produces characteristic horseshoe abscess. Correct answer: Horseshoe abscess. Clinical: requires surgical drainage through palmar incisions.
10) Which flexor tendon does not pass through the carpal tunnel?
a) FCR
b) FDP
c) FDS
d) FPL
Explanation: Flexor carpi radialis (FCR) passes in its own canal, not inside carpal tunnel. Correct answer: FCR. Clinical: helps distinguish isolated FCR tenosynovitis from carpal tunnel pathologies.
Keyword Definitions
• Cardiac muscle – Specialized involuntary muscle forming the myocardium; responsible for heart contractions.
• Myocardium – Thick middle layer of heart wall made of cardiac muscle; contracts to pump blood.
• Intercalated discs – Specialized junctions connecting cardiac muscle cells; contain desmosomes and gap junctions.
• Gap junctions – Channels allowing electrical coupling between cardiac myocytes for synchronized contraction.
• Nucleus – Cardiac muscle cells typically have one centrally located nucleus; occasionally two.
• Striations – Alternating light and dark bands in cardiac muscle due to organized sarcomeres.
• Sheet arrangement – Cardiac muscle fibers arranged in branching sheets for efficient contraction.
• Spindle-shaped – Characteristic of smooth muscle, not cardiac muscle.
• Clinical relevance – Dysfunction of cardiac muscle leads to arrhythmias, heart failure, or cardiomyopathies.
• Histology – Cardiac muscle cells are short, branched, striated, with intercalated discs and central nuclei.
Chapter: Histology / Muscular System
Topic: Cardiac Muscle
Subtopic: Structure, Histology, and Clinical Relevance
Lead Question – 2013
True about cardiac muscle is?
a) Spindle shaped
b) Large central nucleus
c) No gap junctions
d) Arranged in sheets
Explanation: Cardiac muscle fibers are arranged in branching sheets, enabling coordinated contractions. Correct answer: Arranged in sheets. They are striated, short, branched, with one central nucleus and intercalated discs containing gap junctions. Spindle-shaped is smooth muscle. Clinically, the sheet arrangement ensures synchronized myocardial contraction and efficient blood pumping.
Guessed Questions for NEET PG
1) Cardiac muscle fibers are:
a) Striated
b) Non-striated
c) Voluntary
d) Spindle-shaped
Explanation: Cardiac muscle is striated due to sarcomere organization. Correct answer: Striated. Clinical: Striations are essential for contraction; histological changes indicate cardiomyopathies.
2) Intercalated discs contain:
a) Desmosomes and gap junctions
b) Tight junctions only
c) Hemidesmosomes only
d) None
Explanation: Intercalated discs connect cardiac myocytes via desmosomes for mechanical strength and gap junctions for electrical coupling. Correct answer: Desmosomes and gap junctions. Clinical: defects cause arrhythmias.
3) Nucleus in cardiac myocytes is typically:
a) Single and central
b) Multiple and peripheral
c) Absent
d) Peripheral only
Explanation: Cardiac myocytes usually have a single centrally located nucleus. Correct answer: Single and central. Clinical: nuclear abnormalities may indicate hypertrophy or cardiomyopathy.
4) Cardiac muscle is controlled by:
a) Autonomic nervous system
b) Somatic nervous system
c) Hormones only
d) Voluntary control
Explanation: Cardiac muscle contracts involuntarily under autonomic nervous system regulation. Correct answer: Autonomic nervous system. Clinical: autonomic dysfunction affects heart rate and rhythm.
5) Branching of cardiac fibers allows:
a) Efficient force distribution
b) Limited contraction
c) No contraction
d) Smooth movement only
Explanation: Branching ensures synchronized contraction and effective pumping. Correct answer: Efficient force distribution. Clinical: branching abnormalities reduce cardiac efficiency.
6) Gap junctions allow:
a) Electrical coupling
b) Structural support only
c) Nutrient diffusion only
d) No function
Explanation: Gap junctions permit ion flow between cardiac cells for coordinated contraction. Correct answer: Electrical coupling. Clinical: defective gap junctions cause arrhythmias.
7) Spindle-shaped cells are characteristic of:
a) Smooth muscle
b) Cardiac muscle
c) Skeletal muscle
d) Connective tissue
Explanation: Spindle-shaped cells belong to smooth muscle. Correct answer: Smooth muscle. Cardiac cells are branched. Clinical: smooth muscle pathology affects vessel tone.
8) Clinical significance of cardiac muscle sheets:
a) Coordinated contraction of myocardium
b) Voluntary movements
c) Endocrine secretion
d) Filtration
Explanation: Sheet arrangement allows synchronized myocardial contraction. Correct answer: Coordinated contraction of myocardium. Clinical: disruption leads to inefficient pumping and heart failure.
9) Cardiac muscle striations are due to:
a) Sarcomeres
b) Fibrocartilage
c) Elastic fibers
d) Gap junctions
Explanation: Striations arise from organized sarcomeres with actin and myosin filaments. Correct answer: Sarcomeres. Clinical: sarcomere disruption occurs in cardiomyopathy.
10) Cardiac muscle differs from skeletal muscle in:
a) Branching and intercalated discs
b) Voluntary control
c) Peripheral nuclei
d) Non-striated appearance
Explanation: Cardiac muscle is branched, striated, and has intercalated discs, unlike skeletal muscle which is unbranched, striated, and multinucleated. Correct answer: Branching and intercalated discs. Clinical: these features enable synchronized contractions and resistance to mechanical stress.
Keyword Definitions
• Pectoral region – The area on the anterior chest wall related to pectoral muscles.
• Pectoralis major – Large superficial chest muscle aiding arm adduction and medial rotation.
• Pectoralis minor – Lies deep to pectoralis major, attaches coracoid process to ribs.
• Subclavius – Small muscle beneath clavicle, stabilizes clavicle.
• Infraspinatus – Rotator cuff muscle of scapula, not part of pectoral region.
• Axilla – Space beneath pectoral region transmitting vessels and nerves.
• Thoracoacromial artery – Main arterial supply of pectoral region.
• Medial and lateral pectoral nerves – Innervation of pectoralis muscles.
• Clinical correlation – Trauma or surgery of chest may injure pectoral nerves.
• Rotator cuff – Group of muscles stabilizing shoulder joint, includes infraspinatus.
• Breast surgery relevance – Pectoralis muscles form bed of breast and are key in mastectomy.
Chapter: Anatomy / Upper Limb
Topic: Pectoral Region and Axilla
Subtopic: Muscles of pectoral region
Lead Question – 2013
Which of the following muscle is not in the pectoral region?
a) Pectoralis major
b) Infraspinatus
c) Pectoralis minor
d) Subclavius
Explanation: The pectoral region consists of pectoralis major, pectoralis minor, and subclavius. Infraspinatus is located on posterior aspect of scapula, part of rotator cuff, not pectoral region. Correct answer: (b) Infraspinatus. Clinical: Important to distinguish anterior chest muscles from posterior scapular muscles.
Guessed Questions for NEET PG
1) Which nerve supplies pectoralis major?
a) Thoracodorsal nerve
b) Medial and lateral pectoral nerves
c) Axillary nerve
d) Suprascapular nerve
Explanation: Pectoralis major is supplied by both medial and lateral pectoral nerves. Correct answer: Medial and lateral pectoral nerves. Clinical: Nerve injury may weaken adduction and medial rotation of arm.
2) Which structure lies deep to pectoralis minor?
a) Brachial plexus cords
b) Cephalic vein
c) Basilic vein
d) Ulnar nerve
Explanation: Pectoralis minor is a landmark for cords of brachial plexus and axillary vessels. Correct answer: Brachial plexus cords. Clinical: Used as a guide in axillary dissections.
3) Subclavius muscle function is?
a) Elevates scapula
b) Depresses clavicle
c) Flexes humerus
d) Extends arm
Explanation: Subclavius depresses and stabilizes the clavicle during shoulder movements. Correct answer: Depresses clavicle. Clinical: Provides protection to subclavian vessels during clavicular fracture.
4) Which artery mainly supplies pectoralis major?
a) Subclavian artery
b) Thoracoacromial artery
c) Radial artery
d) Subscapular artery
Explanation: The thoracoacromial artery, a branch of axillary artery, supplies pectoralis major. Correct answer: Thoracoacromial artery. Clinical: Important during reconstructive flap surgeries.
5) A breast carcinoma infiltrating deep fascia can involve which muscle first?
a) Serratus anterior
b) Pectoralis major
c) Infraspinatus
d) Latissimus dorsi
Explanation: Pectoralis major forms the bed of breast, hence infiltrated in advanced carcinoma. Correct answer: Pectoralis major. Clinical: Explains fixation of breast mass to chest wall.
6) Which muscle is part of rotator cuff but not pectoral region?
a) Subscapularis
b) Infraspinatus
c) Supraspinatus
d) Teres minor
Explanation: Infraspinatus is part of rotator cuff, not pectoral region. Correct answer: Infraspinatus. Clinical: Weakness causes loss of external rotation at shoulder.
7) Injury to medial pectoral nerve causes weakness of?
a) Deltoid
b) Pectoralis minor
c) Latissimus dorsi
d) Teres major
Explanation: Medial pectoral nerve supplies pectoralis minor and part of pectoralis major. Correct answer: Pectoralis minor. Clinical: May weaken scapular protraction.
8) Which lymph nodes lie deep to pectoralis minor?
a) Apical
b) Central
c) Lateral
d) Subscapular
Explanation: Apical group of axillary lymph nodes lie deep to pectoralis minor. Correct answer: Apical. Clinical: Important in breast cancer spread and axillary clearance surgeries.
9) A patient with winging of scapula likely has injury to?
a) Medial pectoral nerve
b) Long thoracic nerve
c) Axillary nerve
d) Suprascapular nerve
Explanation: Winging of scapula occurs due to serratus anterior paralysis from long thoracic nerve injury. Correct answer: Long thoracic nerve. Clinical: Seen in radical mastectomy complications.
10) Which of the following is not a muscle of anterior axillary fold?
a) Pectoralis major
b) Pectoralis minor
c) Subclavius
d) Latissimus dorsi
Explanation: Anterior axillary fold is formed by lower border of pectoralis major. Latissimus dorsi forms posterior fold. Correct answer: Latissimus dorsi. Clinical: Used in surgical identification of axillary folds.
Keyword Definitions
• Anatomical snuff box – Triangular depression on lateral wrist, important surface landmark.
• Abductor pollicis longus (APL) – Forms lateral boundary of snuff box.
• Extensor pollicis brevis (EPB) – Lateral boundary with APL.
• Extensor pollicis longus (EPL) – Forms medial boundary.
• Extensor carpi ulnaris (ECU) – Not a boundary of snuff box, lies more medially.
• Radial artery – Runs through floor of snuff box, pulse palpable.
• Scaphoid bone – Floor of snuff box, common fracture site.
• Cephalic vein – Originates near snuff box region.
• Superficial branch of radial nerve – Crosses over snuff box, provides cutaneous innervation.
• Clinical importance – Site for palpating scaphoid fracture tenderness.
• Wrist injuries – Tenderness in snuff box suggests scaphoid fracture.
Chapter: Anatomy / Upper Limb
Topic: Wrist and Hand
Subtopic: Anatomical snuff box and relations
Lead Question – 2013
Boundaries of anatomical snuff box are all except
a) APL
b) EPL
c) EPB
d) ECU
Explanation: The anatomical snuff box is bounded laterally by abductor pollicis longus and extensor pollicis brevis, medially by extensor pollicis longus. Extensor carpi ulnaris is not a boundary. Correct answer: (d) ECU. Clinical: Snuff box tenderness is diagnostic of scaphoid fracture.
Guessed Questions for NEET PG
1) Which structure forms the floor of anatomical snuff box?
a) Capitate
b) Lunate
c) Scaphoid
d) Pisiform
Explanation: The scaphoid forms the main floor of the anatomical snuff box, along with trapezium. Correct answer: Scaphoid. Clinical: Scaphoid fractures are suspected if snuff box tenderness is present after a fall on an outstretched hand.
2) Which artery passes through anatomical snuff box?
a) Ulnar artery
b) Radial artery
c) Brachial artery
d) Interosseous artery
Explanation: The radial artery runs through the floor of the snuff box before entering the palm. Correct answer: Radial artery. Clinical: Radial pulse can be palpated here in lean individuals.
3) Injury to scaphoid bone presents with?
a) Swelling of thenar eminence
b) Pain in anatomical snuff box
c) Loss of thumb extension
d) Tingling of little finger
Explanation: Scaphoid fracture commonly presents with pain and tenderness in anatomical snuff box. Correct answer: Pain in anatomical snuff box. Clinical: Delayed diagnosis risks avascular necrosis of proximal scaphoid.
4) Which tendon crosses the floor of snuff box?
a) Flexor carpi radialis
b) Extensor carpi radialis longus
c) Palmaris longus
d) Flexor digitorum profundus
Explanation: Extensor carpi radialis longus and brevis tendons form part of floor of anatomical snuff box. Correct answer: Extensor carpi radialis longus. Clinical: Palpable tendon aids in anatomical landmarking.
5) Which nerve crosses superficial to anatomical snuff box?
a) Ulnar nerve
b) Median nerve
c) Superficial radial nerve
d) Deep radial nerve
Explanation: The superficial branch of radial nerve crosses superficial to snuff box, supplying cutaneous sensation. Correct answer: Superficial radial nerve. Clinical: Injury causes numbness over dorsum of hand near thumb.
6) Which muscle tendon forms medial boundary of snuff box?
a) EPL
b) EPB
c) APL
d) ECU
Explanation: The medial boundary of the snuff box is formed by extensor pollicis longus tendon. Correct answer: EPL. Clinical: Prominent during thumb extension testing.
7) A patient with fall on outstretched hand and tenderness in snuff box most likely has?
a) Colles fracture
b) Scaphoid fracture
c) Hamate fracture
d) Lunate dislocation
Explanation: Snuff box tenderness is classic for scaphoid fracture. Correct answer: Scaphoid fracture. Clinical: Requires urgent immobilization to prevent avascular necrosis.
8) Which carpal bone is most prone to avascular necrosis after fracture?
a) Lunate
b) Pisiform
c) Scaphoid
d) Capitate
Explanation: The scaphoid bone is prone to avascular necrosis due to retrograde blood supply. Correct answer: Scaphoid. Clinical: Missed fractures can cause chronic wrist pain.
9) In wrist examination, tenderness in anatomical snuff box is tested to rule out?
a) Radial head fracture
b) Scaphoid fracture
c) Ulna styloid fracture
d) Capitate fracture
Explanation: Snuff box tenderness specifically indicates scaphoid fracture. Correct answer: Scaphoid fracture. Clinical: Common in young adults after fall on outstretched hand.
10) Which vein originates near anatomical snuff box?
a) Basilic vein
b) Cephalic vein
c) Median cubital vein
d) Radial vein
Explanation: The cephalic vein begins from venous plexus near the anatomical snuff box. Correct answer: Cephalic vein. Clinical: Important for venous access in upper limb procedures.
Keyword Definitions
• Median nerve – Major nerve of forearm and hand, supplies palmar aspect of lateral 3½ fingers.
• Ulnar nerve – Supplies medial 1½ fingers and most intrinsic hand muscles.
• Radial nerve – Provides sensation to dorsum of hand and motor supply to extensor compartment.
• Digital nerves – Terminal branches of median and ulnar nerves, supplying fingers and nail beds.
• Nail bed – Specialized skin beneath nail plate, richly innervated for fine sensation.
• Palmar digital branches – Arise from median nerve, innervate palmar surfaces of lateral fingers.
• Dorsal digital branches – Arise from radial and ulnar nerves, supply dorsum of fingers.
• Clinical surface anatomy – Nail bed sensation is a key test in digital nerve injuries.
• Carpal tunnel – Narrow passage in wrist transmitting median nerve and tendons, site of compression.
• Sensory testing – Performed with pinprick or light touch to assess nerve integrity in trauma.
• Hand dominance – Important in recovery and surgical repair of nerve injuries.
Chapter: Anatomy / Upper Limb
Topic: Hand
Subtopic: Nerve supply of digits and nail bed
Lead Question – 2013
The nerve supply of nail bed of index finger is?
a) Superficial br of radial nerve
b) Deep br of radial nerve
c) Median nerve
d) Ulnar nerve
Explanation: The nail bed of index finger is supplied by the palmar digital branches of the median nerve. The superficial branch of radial nerve supplies dorsum of hand but not the nail bed of index. Correct answer: (c) Median nerve. Clinical: Sensory loss here suggests median nerve injury.
Guessed Questions for NEET PG
1) Sensory loss over nail bed of middle finger indicates injury to?
a) Ulnar nerve
b) Median nerve
c) Radial nerve
d) Musculocutaneous nerve
Explanation: The nail bed of middle finger, like index, is innervated by the median nerve. Injury to the nerve proximal to wrist causes loss of sensation here. Correct answer: Median nerve. Clinical: Important in diagnosing carpal tunnel syndrome.
2) Which nerve supplies nail bed of little finger?
a) Radial nerve
b) Median nerve
c) Ulnar nerve
d) Musculocutaneous nerve
Explanation: The little finger is supplied by the palmar digital branches of the ulnar nerve. Correct answer: Ulnar nerve. Clinical: Injury to ulnar nerve in Guyon’s canal affects sensation of little finger.
3) Which nerve injury is suspected when thumb, index, and middle finger nail beds lose sensation?
a) Radial nerve
b) Median nerve
c) Ulnar nerve
d) Axillary nerve
Explanation: Loss of sensation in thumb, index, and middle finger nail beds indicates median nerve injury. Correct answer: Median nerve. Clinical: Often seen in carpal tunnel syndrome and supracondylar fractures.
4) The superficial branch of radial nerve supplies?
a) Palmar surface of index finger
b) Dorsum of thumb
c) Nail bed of index finger
d) Palmaris brevis muscle
Explanation: The superficial branch of radial nerve supplies dorsum of thumb and hand but does not reach nail beds of index finger. Correct answer: Dorsum of thumb. Clinical: Injury leads to sensory deficit in dorsum of hand.
5) Injury at wrist producing loss of sensation in nail bed of ring finger (lateral half) indicates?
a) Ulnar nerve
b) Median nerve
c) Radial nerve
d) Posterior interosseous nerve
Explanation: The lateral half of ring finger is supplied by median nerve digital branches. Correct answer: Median nerve. Clinical: Important in mixed finger innervation assessment.
6) Which nerve supplies motor innervation to thenar muscles along with nail bed sensation of index finger?
a) Ulnar nerve
b) Median nerve
c) Radial nerve
d) Axillary nerve
Explanation: The median nerve supplies both thenar muscles (except adductor pollicis and deep head of FPB) and sensation of index finger nail bed. Correct answer: Median nerve. Clinical: Injury causes thenar atrophy and sensory loss.
7) A patient with carpal tunnel syndrome will typically complain of numbness over?
a) Little finger nail bed
b) Index and middle finger nail beds
c) Medial palm
d) Dorsum of hand
Explanation: Carpal tunnel compression of median nerve affects sensation over nail beds of thumb, index, middle, and radial half of ring finger. Correct answer: Index and middle finger nail beds. Clinical: Classic diagnostic sign.
8) Which nerve injury is tested by checking sensation at tip of little finger?
a) Median nerve
b) Radial nerve
c) Ulnar nerve
d) Axillary nerve
Explanation: Sensation at tip of little finger is supplied by ulnar nerve digital branches. Correct answer: Ulnar nerve. Clinical: Simple bedside test to isolate ulnar nerve damage.
9) Which nerve provides sensory supply to dorsum of index finger proximal phalanx?
a) Ulnar nerve
b) Median nerve
c) Superficial radial nerve
d) Musculocutaneous nerve
Explanation: The dorsum of proximal index finger is supplied by superficial branch of radial nerve. Correct answer: Superficial radial nerve. Clinical: Differentiate radial vs median injury.
10) Following supracondylar fracture, patient develops loss of sensation in nail bed of index finger. Which nerve is likely injured?
a) Median nerve
b) Ulnar nerve
c) Radial nerve
d) Musculocutaneous nerve
Explanation: Supracondylar fracture often injures median nerve, causing sensory loss in index nail bed. Correct answer: Median nerve. Clinical: Needs urgent assessment due to risk of Volkmann’s ischemic contracture.
Keyword Definitions
• Mammary gland – Modified sweat gland, present in superficial fascia of chest wall.
• Axillary tail of Spence – Extension of breast into axilla, important in carcinoma spread.
• Cooper’s ligaments – Fibrous septa supporting breast tissue, tethered in malignancy.
• Retromammary space – Loose areolar plane between breast and pectoral fascia, allows mobility.
• Internal mammary artery – Provides medial blood supply to breast.
• Lateral thoracic artery – Provides lateral blood supply to breast.
• Nipple – Central pigmented projection, sensory innervation mainly from 4th intercostal nerve.
• Lobules – Functional units of breast, site of lactation.
• Lactiferous ducts – Drain lobules, open at nipple.
• Areola – Pigmented skin surrounding nipple, with Montgomery’s glands.
• Carcinoma breast – Malignancy spreading via lymphatics, often to axillary nodes.
Chapter: Anatomy / Thorax
Topic: Breast
Subtopic: Structure, Supply and Clinical Anatomy
Lead Question – 2013
All are true about mammary gland, except?
a) Is a modified sweat gland
b) Extends from 2nd to 6th rib vertically
c) Supplied by internal mammary artery
d) Nipple is supplied by 6th intercostal nerve
Explanation: The nipple is supplied by the 4th intercostal nerve, not the 6th. The breast is a modified sweat gland extending vertically from the 2nd to 6th rib. Blood supply is mainly by internal mammary and lateral thoracic arteries. Correct answer: (d). Clinical: Nipple sensation is important in surgeries.
Guessed Questions for NEET PG
1) Which nerve supplies the nipple?
a) 2nd intercostal nerve
b) 4th intercostal nerve
c) 6th intercostal nerve
d) Supraclavicular nerve
Explanation: The nipple is supplied by the 4th intercostal nerve, forming a key surface landmark. Correct answer: 4th intercostal nerve. Clinical: This landmark helps locate breast tissue during surgical incisions and reconstructive procedures.
2) Primary blood supply to the lateral portion of breast is from?
a) Internal mammary artery
b) Lateral thoracic artery
c) Subscapular artery
d) Thoracoacromial artery
Explanation: The lateral thoracic artery, a branch of the axillary artery, supplies the lateral breast. Correct answer: Lateral thoracic artery. Clinical: Preserved during mastectomy to maintain vascular supply for flaps.
3) Which statement about Cooper’s ligaments is true?
a) Are vascular structures
b) Are fibrous septa supporting breast
c) Connect nipple to areola only
d) Present only in lactating women
Explanation: Cooper’s ligaments are fibrous septa extending from skin to deep fascia, providing support. Correct answer: Fibrous septa. Clinical: In carcinoma, their contraction causes skin dimpling, a classic sign.
4) The axillary tail of Spence:
a) Lies in inframammary fold
b) Extends into axilla
c) Is absent in males
d) Has no clinical significance
Explanation: The axillary tail extends into the axilla and is clinically important as carcinoma often involves it. Correct answer: Extends into axilla. Clinical: Careful examination is essential in breast cancer screening.
5) Retromammary space allows:
a) Venous drainage
b) Lymphatic drainage
c) Free movement of breast
d) Attachment to skin
Explanation: Retromammary space is a plane of loose areolar tissue permitting free mobility of the breast over pectoral fascia. Correct answer: Free movement of breast. Clinical: Fixation of breast to chest wall indicates advanced carcinoma.
6) Which artery provides medial breast supply?
a) Axillary artery
b) Internal mammary artery
c) Subscapular artery
d) Thoracoacromial artery
Explanation: The internal mammary (internal thoracic) artery provides medial supply via perforating branches. Correct answer: Internal mammary artery. Clinical: In CABG, this artery is often harvested, affecting breast circulation.
7) Which lymph nodes are primarily involved in breast carcinoma spread?
a) Cervical
b) Axillary
c) Mediastinal
d) Inguinal
Explanation: Axillary lymph nodes are primary sites for metastatic spread from breast carcinoma. Correct answer: Axillary nodes. Clinical: Sentinel lymph node biopsy helps identify early spread and guides surgical treatment.
8) Which hormone is mainly responsible for breast milk ejection?
a) Prolactin
b) Estrogen
c) Oxytocin
d) Progesterone
Explanation: Oxytocin, released from posterior pituitary, causes contraction of myoepithelial cells for milk ejection. Correct answer: Oxytocin. Clinical: Suckling reflex stimulates oxytocin release, essential in lactation.
9) In carcinoma of breast, peau d’orange appearance is due to?
a) Blockage of veins
b) Blockage of lymphatics
c) Nerve involvement
d) Arterial spasm
Explanation: Obstruction of dermal lymphatics by carcinoma causes edema, producing skin thickening with dimpled “peau d’orange” appearance. Correct answer: Blockage of lymphatics. Clinical: Indicates locally advanced carcinoma.
10) A lactating mother develops mastitis, the most common causative organism is?
a) E. coli
b) Streptococcus
c) Staphylococcus aureus
d) Klebsiella
Explanation: Staphylococcus aureus is the commonest organism causing lactational mastitis through cracks in nipple. Correct answer: Staphylococcus aureus. Clinical: Treated with antibiotics and continued breastfeeding to prevent abscess formation.
Keyword Definitions
• Bursa – Fluid-filled sac reducing friction between tendon and bone.
• Synovial sheath – Tubular bursa surrounding a tendon for smooth gliding.
• Radial bursa – Synovial sheath enclosing flexor pollicis longus tendon.
• Ulnar bursa – Common flexor sheath for FDP and FDS tendons.
• Flexor pollicis longus (FPL) – Muscle flexing thumb distal phalanx.
• Flexor digitorum profundus (FDP) – Muscle flexing distal phalanges of fingers.
• Flexor digitorum superficialis (FDS) – Muscle flexing middle phalanges.
• Flexor carpi radialis (FCR) – Wrist flexor inserting into 2nd metacarpal.
• Thenar space – Potential space in palm communicating with radial bursa.
• Midpalmar space – Potential space in palm communicating with ulnar bursa.
• Clinical relevance – Infections of synovial sheaths may spread rapidly to palm and forearm.
Chapter: Anatomy / Upper Limb
Topic: Hand and Forearm Structures
Subtopic: Synovial Sheaths and Bursae of Hand
Lead Question – 2013
Radial bursa is the synovial sheath covering the tendon of ?
a) FDS
b) FDP
c) FPL
d) FCR
Explanation: The radial bursa is the synovial sheath of flexor pollicis longus (FPL) tendon. It extends from wrist into the thumb. Correct answer: FPL. Clinical: infection here (tenosynovitis) may spread into the forearm and cause “horseshoe abscess” by communicating with the ulnar bursa.
Guessed Questions for NEET PG
1) Ulnar bursa covers tendons of:
a) FPL
b) FDP and FDS
c) FCR
d) EPL
Explanation: Ulnar bursa is the common flexor sheath enclosing tendons of FDP and FDS to fingers. Correct answer: FDP and FDS. Clinical: infections here can spread into midpalmar space, causing swelling and impaired finger movements.
2) Horseshoe abscess occurs due to communication between:
a) Radial and ulnar bursa
b) Ulnar bursa and carpal tunnel
c) Radial bursa and midpalmar space
d) Thenar and hypothenar spaces
Explanation: Radial bursa of thumb communicates with ulnar bursa of little finger, producing a characteristic “horseshoe-shaped abscess.” Correct answer: Radial and ulnar bursa. Clinical: requires early drainage to prevent spread to forearm.
3) Infection of thumb flexor tendon sheath may spread into:
a) Thenar space
b) Midpalmar space
c) Parona’s space
d) Dorsum of hand
Explanation: FPL tendon sheath infection spreads through radial bursa into Parona’s space (forearm). Correct answer: Parona’s space. Clinical: severe swelling of forearm seen in advanced tenosynovitis.
4) Which tendon passes separately in its own sheath within carpal tunnel?
a) FPL
b) FDP
c) FDS
d) Palmaris longus
Explanation: FPL passes in its own synovial sheath (radial bursa) through the carpal tunnel. Correct answer: FPL. Clinical: inflammation here may cause isolated thumb pain in carpal tunnel syndrome.
5) Parona’s space is located:
a) Between palmar aponeurosis and flexor tendons
b) Between pronator quadratus and flexor tendons
c) In dorsal hand
d) In thenar eminence
Explanation: Parona’s space is between pronator quadratus and flexor tendons in distal forearm. Correct answer: Between pronator quadratus and flexor tendons. Clinical: serves as pathway for spread of infection from radial or ulnar bursa.
6) Which of the following muscles inserts into the distal phalanx of thumb?
a) FPL
b) FDS
c) FDP
d) EPL
Explanation: Flexor pollicis longus (FPL) inserts into the base of distal phalanx of thumb, flexing IP joint. Correct answer: FPL. Clinical: important in pinch grip strength, loss indicates anterior interosseous nerve palsy.
7) Ulnar bursa commonly extends up to which finger?
a) Index
b) Middle
c) Ring
d) Little
Explanation: The ulnar bursa extends into the little finger flexor sheath. Correct answer: Little finger. Clinical: explains why infections of little finger flexor sheath can spread to common flexor sheath and palm.
8) Which structure is enclosed within both radial bursa and carpal tunnel?
a) FCR
b) FPL
c) EPL
d) Lumbricals
Explanation: FPL tendon passes through carpal tunnel inside its radial bursa sheath. Correct answer: FPL. Clinical: tenosynovitis here may mimic carpal tunnel syndrome with isolated thumb symptoms.
9) A 25-year-old presents with swelling of thumb and little finger tendon sheaths with forearm spread. Most likely condition?
a) Thenar abscess
b) Midpalmar abscess
c) Horseshoe abscess
d) Carpal tunnel syndrome
Explanation: Simultaneous infection of radial and ulnar bursae produces characteristic horseshoe abscess. Correct answer: Horseshoe abscess. Clinical: requires surgical drainage through palmar incisions.
10) Which flexor tendon does not pass through the carpal tunnel?
a) FCR
b) FDP
c) FDS
d) FPL
Explanation: Flexor carpi radialis (FCR) passes in its own canal, not inside carpal tunnel. Correct answer: FCR. Clinical: helps distinguish isolated FCR tenosynovitis from carpal tunnel pathologies.
Keyword Definitions
• Humerus – Long bone of upper limb extending from shoulder to elbow.
• Ossification center – Site where bone formation begins during development.
• Primary center – First ossification site, usually diaphysis of long bone.
• Secondary center – Ossification sites in epiphysis, appear later.
• Distal humerus – Lower end of humerus forming part of elbow joint.
• Capitulum – Lateral articular surface of distal humerus, articulates with radius.
• Trochlea – Medial articular surface of distal humerus, articulates with ulna.
• Epicondyle – Bony prominence above condyles for muscle attachment.
• Epiphysis – End part of long bone formed from secondary centers.
• Physis – Growth plate between diaphysis and epiphysis.
• Clinical relevance – Helps in diagnosing pediatric fractures and growth disturbances.
Chapter: Anatomy / Upper Limb
Topic: Osteology of Humerus
Subtopic: Ossification Centers of Distal Humerus
Lead Question – 2013
Distal end of humerus develops from how many centres ?
a) 2
b) 5
c) 3
d) 4
Explanation: The distal end of humerus has multiple secondary ossification centers – capitulum, trochlea, lateral epicondyle, medial epicondyle. Together, there are 4 centers. Correct answer: 4. Clinical: knowledge of ossification sequence (CRITOE rule) is important in interpreting pediatric elbow X-rays.
Guessed Questions for NEET PG
1) First ossification center to appear at distal humerus?
a) Capitulum
b) Trochlea
c) Medial epicondyle
d) Lateral epicondyle
Explanation: The capitulum is the first secondary ossification center to appear, around 1 year of age. Others follow in a predictable CRITOE sequence. Correct answer: Capitulum. Clinical: helps differentiate normal ossification centers from fracture fragments in pediatric radiographs.
2) Medial epicondyle ossification center appears at which age?
a) 2 years
b) 5 years
c) 7 years
d) 9 years
Explanation: The medial epicondyle ossification center appears around 5 years of age and fuses late, around puberty. Correct answer: 5 years. Clinical: it is the most common site of avulsion fracture in children due to throwing injuries.
3) Which ossification center fuses last in distal humerus?
a) Capitulum
b) Trochlea
c) Medial epicondyle
d) Lateral epicondyle
Explanation: Medial epicondyle is the last to fuse, around 18–20 years. Correct answer: Medial epicondyle. Clinical: helps identify skeletal maturity and growth potential in radiographs.
4) Sequence of ossification centers in elbow joint is remembered by acronym:
a) CRITOE
b) SALTER
c) ABCDEF
d) PRISME
Explanation: CRITOE – Capitulum, Radial head, Internal epicondyle (medial), Trochlea, Olecranon, External epicondyle (lateral). Correct answer: CRITOE. Clinical: used to read pediatric elbow X-rays and prevent misdiagnosis of normal centers as fractures.
5) At what age does trochlea ossification center appear?
a) 2 years
b) 7 years
c) 9 years
d) 12 years
Explanation: Trochlea ossification center appears around 9 years. Correct answer: 9 years. Clinical: irregular ossification pattern here can mimic fracture in children, so proper sequence knowledge is vital.
6) Failure of medial epicondyle fusion leads to:
a) Cubitus varus
b) Cubitus valgus
c) Recurvatum
d) Radial head dislocation
Explanation: Nonunion of medial epicondyle can cause valgus deformity due to loss of medial support. Correct answer: Cubitus valgus. Clinical: may lead to tardy ulnar nerve palsy in adults.
7) Which nerve is most endangered in medial epicondyle fractures?
a) Radial
b) Median
c) Ulnar
d) Musculocutaneous
Explanation: Ulnar nerve runs in groove behind medial epicondyle, making it vulnerable in avulsion fractures. Correct answer: Ulnar nerve. Clinical: may present with tingling in medial 1½ fingers.
8) In children, supracondylar fracture of humerus occurs commonly due to:
a) Direct trauma
b) Fall on flexed elbow
c) Fall on outstretched hand
d) Rotational injury
Explanation: Supracondylar fractures are common pediatric injuries due to fall on outstretched hand, leading to distal humerus fracture above condyles. Correct answer: Fall on outstretched hand. Clinical: may damage brachial artery and median nerve.
9) Which structure is at risk in supracondylar fracture of humerus?
a) Ulnar nerve
b) Radial artery
c) Brachial artery
d) Axillary nerve
Explanation: The brachial artery is closely related anteriorly and is most commonly injured in supracondylar fractures. Correct answer: Brachial artery. Clinical: leads to Volkmann’s ischemic contracture if untreated.
10) A 7-year-old presents with elbow swelling. X-ray shows separate ossification center at medial side. Most likely structure?
a) Capitulum
b) Trochlea
c) Medial epicondyle
d) Olecranon
Explanation: At 7 years, medial epicondyle ossification center is visible. Correct answer: Medial epicondyle. Clinical: distinguishing this from fracture fragment is crucial in pediatric practice.
Keyword Definitions
• Ulnar nerve – Terminal branch of medial cord of brachial plexus, supplies intrinsic hand muscles and some forearm flexors.
• Flexor carpi ulnaris (FCU) – Forearm muscle, flexes and adducts wrist, supplied by ulnar nerve.
• Flexor digitorum profundus (FDP) – Deep flexor of fingers, medial half supplied by ulnar nerve, lateral half by median nerve.
• Forearm flexors – Muscles anterior to radius/ulna, flex wrist and fingers.
• Medial cord – Branch of brachial plexus giving rise to ulnar nerve.
• Cubital tunnel – Anatomical passage for ulnar nerve at elbow, common entrapment site.
• Claw hand – Deformity caused by ulnar nerve injury, hyperextension at MCP, flexion at IP joints.
• Guyon’s canal – Ulnar nerve compression site at wrist.
• Sensory supply – Ulnar nerve supplies medial 1½ fingers and corresponding palm/dorsum.
• Motor supply – Ulnar nerve supplies FCU, medial FDP, and most intrinsic hand muscles.
• Clinical localization – Identifying site of nerve injury based on selective muscle/sensory involvement.
Chapter: Anatomy / Upper Limb
Topic: Brachial Plexus
Subtopic: Ulnar Nerve in Arm and Forearm
Lead Question – 2013
In arm ulnar nerve gives muscular branch to which muscle ?
a) FCU
b) FDP
c) Both
d) None
Explanation: In the arm, the ulnar nerve does not supply any muscle. It simply travels down medially without branches. FCU and FDP receive branches in the forearm, not arm. Correct answer: None. Clinical: important in localizing lesions since proximal arm injuries do not affect muscle action directly.
Guessed Questions for NEET PG
1) Which muscle is supplied by ulnar nerve in the forearm?
a) Pronator teres
b) Flexor carpi ulnaris
c) Flexor pollicis longus
d) Palmaris longus
Explanation: Ulnar nerve supplies FCU and medial half of FDP in the forearm. Flexor carpi ulnaris is a key muscle supplied by it. Correct answer: Flexor carpi ulnaris. Clinical: tested by resisted wrist flexion and adduction.
2) A patient with ulnar nerve lesion at elbow will have weakness of:
a) Pronation
b) Wrist flexion and adduction
c) Wrist extension
d) Supination
Explanation: Elbow lesion of ulnar nerve affects FCU and medial FDP. This weakens wrist flexion/adduction and finger flexion. Correct answer: Wrist flexion and adduction. Clinical: combined with sensory loss over medial hand.
3) Which deformity is caused by distal ulnar nerve lesion at wrist?
a) Wrist drop
b) Claw hand
c) Ape thumb
d) Benediction sign
Explanation: Distal ulnar nerve lesion causes paralysis of lumbricals/interossei leading to claw hand deformity. Correct answer: Claw hand. Clinical: more severe when lesion is distal because FDP is spared.
4) Which nerve is compressed in Guyon’s canal syndrome?
a) Radial
b) Median
c) Ulnar
d) Musculocutaneous
Explanation: Ulnar nerve passes through Guyon’s canal near wrist. Compression here produces sensory and motor loss in ulnar distribution without affecting forearm muscles. Correct answer: Ulnar nerve. Clinical: common in cyclists ("handlebar palsy").
5) Which intrinsic hand muscle is not supplied by ulnar nerve?
a) Adductor pollicis
b) First dorsal interosseous
c) Lateral two lumbricals
d) Palmar interossei
Explanation: Median nerve supplies lateral two lumbricals and thenar muscles (except adductor pollicis). Ulnar supplies all others. Correct answer: Lateral two lumbricals. Clinical: important for fine finger movements.
6) Injury to ulnar nerve at elbow spares which muscle?
a) FCU
b) FDP (medial half)
c) FDP (lateral half)
d) Palmar interossei
Explanation: Lateral half of FDP is supplied by median nerve, not ulnar nerve. Hence spared in elbow lesion. Correct answer: FDP (lateral half). Clinical: helps differentiate median vs ulnar nerve contributions.
7) Sensory supply of ulnar nerve includes:
a) Lateral 3½ fingers
b) Medial 1½ fingers
c) Entire palm
d) Thenar eminence
Explanation: Ulnar nerve supplies skin of medial 1½ fingers and adjacent palm/dorsum. Correct answer: Medial 1½ fingers. Clinical: loss of sensation here is diagnostic.
8) Froment’s sign is positive in lesion of:
a) Radial nerve
b) Median nerve
c) Ulnar nerve
d) Axillary nerve
Explanation: Froment’s sign indicates weakness of adductor pollicis supplied by ulnar nerve. Thumb flexion occurs due to compensation by flexor pollicis longus. Correct answer: Ulnar nerve. Clinical: classic bedside test.
9) A patient with difficulty in finger abduction most likely has injury to:
a) Median nerve
b) Ulnar nerve
c) Radial nerve
d) Musculocutaneous nerve
Explanation: Interossei supplied by ulnar nerve abduct/adduct fingers. Injury impairs abduction. Correct answer: Ulnar nerve. Clinical: seen in claw hand cases.
10) Which test checks for integrity of ulnar nerve?
a) Phalen’s test
b) Card test
c) Tinel’s sign
d) O’Brien’s test
Explanation: Card test involves holding paper between fingers using interossei. Ulnar nerve lesion causes inability to hold paper. Correct answer: Card test. Clinical: simple bedside diagnostic tool.
Keyword Definitions
• Radial nerve – Largest branch of brachial plexus, supplies posterior compartment of arm and forearm.
• Spiral groove – Groove on humerus where radial nerve travels, common injury site in fractures.
• Triceps brachii – Muscle with three heads (long, lateral, medial), main extensor of elbow.
• Extensor carpi radialis longus (ECRL) – Radial nerve branch above spiral groove, extends wrist.
• Wrist drop – Inability to extend wrist due to radial nerve lesion.
• Posterior interosseous nerve – Deep radial branch, supplies finger extensors.
• Saturday night palsy – Radial nerve compression in axilla leading to wrist drop.
• Humeral shaft fracture – Common cause of radial nerve injury at spiral groove.
• Supinator canal – Site of posterior interosseous nerve entrapment.
• Clinical localization – Identifying nerve injury by selective motor/sensory loss.
• Dorsal web space – Sensory area supplied by superficial radial nerve.
Chapter: Anatomy / Upper Limb
Topic: Brachial Plexus
Subtopic: Radial Nerve at Spiral Groove
Lead Question – 2013
Which muscle will be paralyzed when radial nerve is injured in just below the spiral groove ?
a) Lateral head of triceps
b) Medial head of triceps
c) Long head of triceps
d) ECRL
Explanation: Radial nerve gives branches to triceps before entering spiral groove, sparing it in distal lesions. ECRL is also supplied above the groove. Injury just below spiral groove paralyzes medial head of triceps. Correct answer: Medial head of triceps. Clinical: elbow extension weak but not absent.
Guessed Questions for NEET PG
1) In a mid-shaft humeral fracture, which motor deficit is most expected?
a) Loss of elbow extension
b) Wrist drop
c) Loss of pronation
d) Finger flexion weakness
Explanation: Radial nerve in spiral groove is injured, sparing triceps but affecting wrist/finger extensors. This causes wrist drop while elbow extension remains intact. Correct answer: Wrist drop. Clinical: selective extensor weakness helps localize lesion.
2) A patient with radial nerve injury in axilla will present with:
a) Wrist drop only
b) Elbow and wrist extension loss
c) Only sensory loss
d) Finger abduction loss
Explanation: Axillary radial nerve lesion affects triceps, wrist extensors, and sensory branches. This produces loss of elbow and wrist extension with sensory loss. Correct answer: Elbow and wrist extension loss. Clinical: classic in crutch palsy.
3) Which muscle is spared in radial nerve injury at spiral groove?
a) Extensor digitorum
b) Extensor carpi radialis longus
c) Extensor pollicis longus
d) Extensor indicis
Explanation: ECRL is supplied before spiral groove. Thus, wrist extension is weak but not lost. Correct answer: Extensor carpi radialis longus. Clinical: partial wrist drop seen instead of complete.
4) Loss of thumb extension is seen in injury to:
a) Median nerve
b) Ulnar nerve
c) Posterior interosseous nerve
d) Musculocutaneous nerve
Explanation: Posterior interosseous nerve supplies extensor pollicis longus and brevis. Its injury causes inability to extend thumb. Correct answer: Posterior interosseous nerve. Clinical: selective thumb drop without wrist involvement.
5) A patient cannot adduct fingers but wrist extension is normal. Likely nerve injured?
a) Radial
b) Median
c) Ulnar
d) Axillary
Explanation: Ulnar nerve supplies interossei for finger adduction. Radial nerve intact preserves wrist extension. Correct answer: Ulnar nerve. Clinical: card test positive.
6) Injury to radial nerve just above wrist affects:
a) Motor only
b) Sensory only
c) Both motor and sensory
d) Neither
Explanation: At wrist, radial nerve is superficial and purely sensory. Injury here causes sensory loss in dorsum of first web space. Correct answer: Sensory only. Clinical: no motor deficit seen.
7) Which nerve is tested by sensation over dorsal first web space?
a) Median
b) Ulnar
c) Radial
d) Musculocutaneous
Explanation: Radial nerve superficial branch supplies skin of first web space dorsally. Correct answer: Radial nerve. Clinical: useful in localizing high radial lesions.
8) Which nerve injury produces “claw hand”?
a) Median
b) Ulnar
c) Radial
d) Axillary
Explanation: Ulnar nerve injury at wrist causes paralysis of medial lumbricals leading to claw hand deformity. Correct answer: Ulnar nerve. Clinical: worsens with distal lesions.
9) Inability to supinate forearm after fracture of proximal radius is due to injury of:
a) Median
b) Radial
c) Musculocutaneous
d) Ulnar
Explanation: Supinator is innervated by posterior interosseous nerve, branch of radial. Injury near proximal radius affects supination. Correct answer: Radial nerve. Clinical: partial supination possible via biceps if intact.
10) A patient develops wrist drop after sleeping with arm compressed over chair. This condition is called:
a) Crutch palsy
b) Saturday night palsy
c) Honeymoon palsy
d) Arcade syndrome
Explanation: Compression of radial nerve in axilla during deep sleep causes Saturday night palsy. Correct answer: Saturday night palsy. Clinical: wrist drop with sensory loss over dorsum hand.
Keyword Definitions
• Radial nerve – Largest branch of brachial plexus, supplies extensors of arm and forearm.
• Posterior interosseous nerve – Deep terminal branch of radial nerve, supplies finger extensors.
• Median nerve – Supplies most anterior forearm muscles and thenar muscles.
• Ulnar nerve – Supplies intrinsic hand muscles and medial forearm muscles.
• Wrist drop – Inability to extend wrist due to radial nerve injury.
• Spiral groove – Location of radial nerve on humerus, commonly injured in fractures.
• Crutch palsy – Radial nerve injury due to axillary compression from crutches.
• Dorsal digital branch – Radial nerve branch supplying skin over first web space.
• Clinical localization – Identifying nerve injuries based on motor and sensory deficits.
• Supinator canal – Site where posterior interosseous nerve may be compressed.
• Extensor compartment – Posterior muscles of forearm responsible for finger/wrist extension.
Chapter: Anatomy / Upper Limb
Topic: Brachial Plexus
Subtopic: Posterior Interosseous Nerve
Lead Question – 2013
A person had injury to right upper limb he is not able to extend fingers but able to extend wrist and elbow. Nerve injured is ?
a) Radial
b) Median
c) Ulnar
d) Posterior interosseous
Explanation: Finger extension is controlled by posterior interosseous nerve, a branch of radial nerve. Wrist and elbow extension are preserved because proximal radial nerve branches are intact. Correct answer: Posterior interosseous nerve. Clinical: injury produces finger drop without wrist drop.
Guessed Questions for NEET PG
1) Which nerve is injured in humeral shaft fracture leading to wrist drop?
a) Median
b) Radial
c) Ulnar
d) Axillary
Explanation: Radial nerve travels in the spiral groove of humerus and is vulnerable in shaft fractures. Injury leads to wrist drop and sensory loss in dorsum of hand. Correct answer: Radial nerve. Clinical: triceps often spared due to proximal innervation.
2) A patient with wrist extension preserved but inability to extend thumb likely has lesion of:
a) Median nerve
b) Radial nerve
c) Posterior interosseous nerve
d) Ulnar nerve
Explanation: Posterior interosseous nerve specifically supplies thumb extensors. Wrist extension is preserved via intact extensor carpi radialis longus. Correct answer: Posterior interosseous nerve. Clinical: selective finger drop is hallmark.
3) Inability to oppose thumb is due to injury of:
a) Ulnar nerve
b) Median nerve
c) Radial nerve
d) Musculocutaneous nerve
Explanation: Median nerve supplies thenar muscles including opponens pollicis. Its injury prevents thumb opposition. Correct answer: Median nerve. Clinical: seen in carpal tunnel syndrome or wrist lacerations.
4) Which nerve is injured in “Saturday night palsy”?
a) Radial
b) Ulnar
c) Median
d) Axillary
Explanation: Prolonged pressure in axilla compresses radial nerve, leading to wrist drop. Correct answer: Radial nerve. Clinical: common in unconscious patients with arm hanging over chair.
5) Loss of sensation in first dorsal web space occurs in:
a) Median nerve lesion
b) Ulnar nerve lesion
c) Radial nerve lesion
d) Axillary nerve lesion
Explanation: The radial nerve supplies skin over dorsum of first web space. Correct answer: Radial nerve. Clinical: this sensory loss confirms radial nerve lesion.
6) Which nerve passes through supinator canal and may be compressed there?
a) Ulnar
b) Median
c) Posterior interosseous
d) Axillary
Explanation: Posterior interosseous nerve passes through supinator canal (Arcade of Frohse) where entrapment can occur. Correct answer: Posterior interosseous nerve. Clinical: presents with finger drop but preserved wrist extension.
7) A patient cannot extend elbow. The nerve involved is:
a) Median
b) Radial
c) Ulnar
d) Musculocutaneous
Explanation: Radial nerve supplies triceps brachii responsible for elbow extension. Injury proximal to triceps branches leads to loss of elbow extension. Correct answer: Radial nerve.
8) Inability to adduct fingers is due to lesion of:
a) Median nerve
b) Ulnar nerve
c) Radial nerve
d) Axillary nerve
Explanation: Ulnar nerve supplies interossei muscles responsible for finger adduction and abduction. Correct answer: Ulnar nerve. Clinical: test by asking patient to hold a card between fingers (card test).
9) Which nerve is commonly injured during axillary lymph node dissection?
a) Long thoracic
b) Radial
c) Axillary
d) Median
Explanation: Long thoracic and thoracodorsal nerves are at risk, but not radial. Correct answer: Long thoracic nerve. Clinical: its injury leads to winging of scapula due to serratus anterior paralysis.
10) A patient with inability to pronate forearm likely has injury of:
a) Radial nerve
b) Median nerve
c) Ulnar nerve
d) Axillary nerve
Explanation: Median nerve supplies pronator teres and pronator quadratus. Injury causes loss of pronation. Correct answer: Median nerve. Clinical: pronation deficit helps localize lesion.
Keyword Definitions
• Brachial plexus – Network of nerves formed by anterior rami of C5–T1 supplying upper limb.
• Radial nerve – Largest branch of brachial plexus, supplies extensor compartment of arm and forearm.
• Ulnar nerve – Arises from medial cord, supplies intrinsic hand muscles and medial forearm.
• Median nerve – Formed by medial and lateral cords, supplies anterior forearm and thenar muscles.
• Axillary nerve – Branch of posterior cord, supplies deltoid and teres minor.
• Posterior cord – Formed by posterior divisions of all trunks of brachial plexus.
• Musculocutaneous nerve – Arises from lateral cord, supplies anterior arm muscles.
• Clinical correlation – Radial nerve injury leads to wrist drop and sensory loss in dorsum of hand.
• Surgical relevance – Axillary dissection may endanger nerves such as thoracodorsal and long thoracic.
• Root value – Spinal segmental origin of a peripheral nerve, important in localization of lesions.
• Extensor compartment – Posterior arm and forearm muscles controlled by radial nerve.
Chapter: Anatomy / Upper Limb
Topic: Brachial Plexus
Subtopic: Radial Nerve
Lead Question – 2013
Largest branch of brachial plexus is
a) Ulnar
b) Median
c) Radial
d) Axillary
Explanation: The radial nerve is the largest branch of the brachial plexus. It arises from the posterior cord (C5–T1) and supplies the extensor compartments of the arm and forearm. Correct answer: Radial nerve. Clinically, its injury causes wrist drop and weak hand grip due to loss of extensors.
Guessed Questions for NEET PG
1) Root value of radial nerve is:
a) C5–C6
b) C5–T1
c) C7–T1
d) C6–C8
Explanation: Radial nerve is derived from the posterior cord of the brachial plexus with root values C5–T1. Correct answer: C5–T1. Clinically, knowledge of root value helps in diagnosing radiculopathies presenting with upper limb weakness.
2) Nerve injured in mid-shaft fracture of humerus:
a) Median
b) Ulnar
c) Radial
d) Axillary
Explanation: Radial nerve runs in the spiral groove of the humerus and is commonly injured in mid-shaft fractures. Correct answer: Radial nerve. This results in wrist drop due to paralysis of wrist extensors.
3) Which nerve supplies triceps brachii?
a) Axillary
b) Radial
c) Median
d) Musculocutaneous
Explanation: Triceps brachii, the main extensor of the elbow, is innervated by the radial nerve. Correct answer: Radial nerve. Clinical: injury above triceps branches causes loss of elbow extension along with wrist drop.
4) Nerve supply of supinator muscle is:
a) Median
b) Radial (deep branch)
c) Ulnar
d) Musculocutaneous
Explanation: The deep branch of the radial nerve, also called the posterior interosseous nerve, innervates the supinator muscle. Correct answer: Radial (deep branch). Clinical: weakness in supination if injured.
5) Wrist drop is due to injury of:
a) Median nerve
b) Ulnar nerve
c) Radial nerve
d) Musculocutaneous nerve
Explanation: Wrist drop occurs due to loss of extensor muscles supplied by the radial nerve. Correct answer: Radial nerve. Clinical: seen in humeral shaft fractures or compressive neuropathy ("Saturday night palsy").
6) Which nerve is closely related to spiral groove of humerus?
a) Axillary
b) Median
c) Radial
d) Ulnar
Explanation: The radial nerve courses through the spiral groove of the humerus. Correct answer: Radial nerve. Clinical: susceptible to injury in humeral shaft fractures leading to sensory and motor deficits.
7) Posterior interosseous nerve is a branch of:
a) Median
b) Ulnar
c) Radial
d) Axillary
Explanation: Posterior interosseous nerve is the terminal deep branch of radial nerve after passing through supinator. Correct answer: Radial nerve. Clinical: supplies most extensor muscles of forearm.
8) Which nerve supplies skin over dorsum of first web space?
a) Median
b) Ulnar
c) Radial
d) Musculocutaneous
Explanation: The dorsal digital branch of radial nerve supplies skin of first web space between thumb and index finger. Correct answer: Radial nerve. Clinical: sensory loss here confirms radial nerve lesion.
9) Nerve injured in improper use of crutches (“crutch palsy”):
a) Median
b) Radial
c) Ulnar
d) Axillary
Explanation: Radial nerve injury occurs in axilla due to compression from crutches or prolonged pressure. Correct answer: Radial nerve. Clinical: causes wrist drop and weakness of grip.
10) Which nerve is tested by extension of wrist against resistance?
a) Median
b) Ulnar
c) Radial
d) Axillary
Explanation: Radial nerve integrity is tested by checking wrist extension against resistance. Correct answer: Radial nerve. Clinical: inability indicates lesion of radial nerve or its branches.
Keyword Definitions
• Thoracodorsal nerve – Branch of posterior cord, supplies latissimus dorsi.
• Root value – Spinal nerves contributing fibers to a peripheral nerve.
• Brachial plexus – Formed by ventral rami of C5–T1, supplies upper limb.
• Latissimus dorsi – Muscle aiding extension, adduction, and medial rotation of humerus.
• Posterior cord – Division of brachial plexus formed by posterior divisions of all trunks.
• Axillary nerve – Terminal branch of posterior cord, innervates deltoid and teres minor.
• Long thoracic nerve – Arises from C5–C7 roots, supplies serratus anterior.
• Suprascapular nerve – Arises from upper trunk, supplies supraspinatus and infraspinatus.
• Clinical correlation – Injury to thoracodorsal nerve impairs arm adduction, weakens shoulder extension.
• Surgical relevance – Preserved during axillary clearance to maintain latissimus dorsi flap viability.
Chapter: Anatomy / Upper Limb
Topic: Brachial Plexus
Subtopic: Thoracodorsal Nerve
Lead Question – 2013
Root value of thoracodorsal nerve?
a) C5, C6, C7
b) C8, T1
c) C6, C7, C8
d) T1, T2
Explanation: Thoracodorsal nerve arises from the posterior cord of brachial plexus with root value C6, C7, and C8. It innervates latissimus dorsi, which is important in climbing and swimming. Correct answer: C6, C7, C8. Clinically preserved during axillary dissections to prevent functional loss of shoulder movements.
Guessed Questions for NEET PG
1) Nerve supply of latissimus dorsi is:
a) Thoracodorsal nerve
b) Dorsal scapular nerve
c) Axillary nerve
d) Long thoracic nerve
Explanation: Latissimus dorsi is supplied by the thoracodorsal nerve (C6–C8). Correct answer: Thoracodorsal nerve. Clinical: important for forceful adduction and extension of the arm, also preserved in reconstructive flap surgeries.
2) Root value of long thoracic nerve is:
a) C5–C7
b) C7–C9
c) C8–T1
d) C5–C6
Explanation: Long thoracic nerve arises from C5, C6, and C7 roots and supplies serratus anterior. Correct answer: C5–C7. Clinical: injury produces winged scapula due to paralysis of serratus anterior.
3) Which nerve is closely related to axillary lymph node dissection?
a) Thoracodorsal nerve
b) Median nerve
c) Ulnar nerve
d) Radial nerve
Explanation: Thoracodorsal nerve lies in the axilla and is at risk during axillary dissection. Correct answer: Thoracodorsal nerve. Clinical: its injury leads to weakness in shoulder extension and loss of latissimus dorsi flap viability.
4) Nerve injured in surgical neck fracture of humerus:
a) Axillary nerve
b) Radial nerve
c) Median nerve
d) Thoracodorsal nerve
Explanation: Axillary nerve winds around the surgical neck of humerus with posterior circumflex humeral artery. Correct answer: Axillary nerve. Clinical: injury causes inability to abduct shoulder beyond 15° and loss of sensation over deltoid patch.
5) Function of latissimus dorsi is:
a) Flexion and lateral rotation
b) Extension, adduction, medial rotation
c) Abduction and supination
d) Flexion and pronation
Explanation: Latissimus dorsi is a powerful extensor, adductor, and medial rotator of the arm. Correct answer: Extension, adduction, medial rotation. Clinical: active in climbing and swimming movements.
6) Root value of axillary nerve:
a) C5–C6
b) C7–C8
c) C8–T1
d) C5–C7
Explanation: Axillary nerve arises from posterior cord with root value C5 and C6. Correct answer: C5–C6. Clinical: injured in humeral fractures causing deltoid paralysis and sensory loss on upper arm.
7) Which nerve supplies serratus anterior?
a) Long thoracic nerve
b) Thoracodorsal nerve
c) Dorsal scapular nerve
d) Axillary nerve
Explanation: Serratus anterior is innervated by the long thoracic nerve (C5–C7). Correct answer: Long thoracic nerve. Clinical: injury produces winged scapula, especially during axillary lymph node dissection.
8) Root value of suprascapular nerve:
a) C5–C6
b) C6–C7
c) C8–T1
d) C5–C7
Explanation: Suprascapular nerve arises from the upper trunk of brachial plexus with root value C5–C6. Correct answer: C5–C6. Clinical: supplies supraspinatus and infraspinatus, important in shoulder abduction and external rotation.
9) Which nerve injury causes wrist drop?
a) Ulnar nerve
b) Median nerve
c) Radial nerve
d) Thoracodorsal nerve
Explanation: Radial nerve injury leads to paralysis of wrist extensors causing wrist drop. Correct answer: Radial nerve. Clinical: commonly seen in mid-shaft humerus fractures.
10) Klumpke’s palsy involves injury to:
a) Upper trunk (C5–C6)
b) Lower trunk (C8–T1)
c) Posterior cord
d) Medial cord
Explanation: Klumpke’s palsy occurs due to lower trunk (C8–T1) injury. Correct answer: Lower trunk (C8–T1). Clinical: causes claw hand deformity and weakness of intrinsic hand muscles.
Keyword Definitions
• Brachial plexus – A network of nerves formed by ventral rami of C5–T1, supplying upper limb.
• Infraclavicular branches – Nerves arising below clavicle from cords of brachial plexus.
• Supraclavicular branches – Nerves arising above clavicle, mainly from roots and trunks.
• Ulnar nerve – Terminal branch of medial cord; motor to intrinsic hand muscles and sensory to medial hand.
• Long thoracic nerve – Supraclavicular branch from roots (C5–C7); supplies serratus anterior.
• Axillary nerve – Terminal branch of posterior cord; supplies deltoid and teres minor.
• Thoracodorsal nerve – Branch of posterior cord; supplies latissimus dorsi.
• Cords of brachial plexus – Named medial, lateral, posterior according to relation with axillary artery.
• Clinical correlation – Injury to long thoracic nerve causes winging of scapula.
• Fascial compartments – Axilla contains cords of plexus, vessels, and lymph nodes surrounded by sheath.
Chapter: Anatomy / Upper Limb
Topic: Brachial Plexus
Subtopic: Infraclavicular vs Supraclavicular Branches
Lead Question – 2013
All are infraclavicular branches of brachial plexus except?
a) Ulnar nerve
b) Long thoracic nerve
c) Axillary nerve
d) Thoracodorsal nerve
Explanation: Infraclavicular branches arise from cords of brachial plexus. Ulnar, axillary, and thoracodorsal nerves are infraclavicular. Long thoracic nerve arises from roots above the clavicle, hence it is supraclavicular. Correct answer: Long thoracic nerve. Clinically, injury to this nerve produces winged scapula due to serratus anterior paralysis.
Guessed Questions for NEET PG
1) Which nerve is injured in winged scapula?
a) Axillary nerve
b) Long thoracic nerve
c) Thoracodorsal nerve
d) Dorsal scapular nerve
Explanation: Winged scapula occurs due to paralysis of serratus anterior muscle from long thoracic nerve injury. Correct answer: Long thoracic nerve. Clinical: commonly injured in axillary dissections or trauma to lateral thoracic wall.
2) Axillary nerve supplies which muscle?
a) Latissimus dorsi
b) Teres major
c) Teres minor
d) Pectoralis minor
Explanation: Axillary nerve innervates deltoid and teres minor. Correct answer: Teres minor. Clinical: Injury causes inability to abduct shoulder beyond 15° and loss of sensation over deltoid patch.
3) Thoracodorsal nerve supplies:
a) Pectoralis major
b) Latissimus dorsi
c) Subscapularis
d) Serratus anterior
Explanation: Thoracodorsal nerve (middle subscapular nerve) arises from posterior cord and supplies latissimus dorsi. Correct answer: Latissimus dorsi. Clinical: important in flap surgeries like latissimus dorsi flap for reconstruction.
4) Ulnar nerve lesion at wrist causes:
a) Wrist drop
b) Claw hand
c) Ape thumb
d) Foot drop
Explanation: Ulnar nerve injury at wrist leads to claw hand due to loss of intrinsic hand muscles. Correct answer: Claw hand. Clinical: common in fractures of hook of hamate or lacerations.
5) Which is a supraclavicular branch of brachial plexus?
a) Musculocutaneous nerve
b) Long thoracic nerve
c) Median nerve
d) Ulnar nerve
Explanation: Long thoracic nerve arises from roots (C5–C7) above the clavicle, making it a supraclavicular branch. Correct answer: Long thoracic nerve. Clinical: vulnerable during axillary lymph node dissection.
6) Which cord gives rise to median nerve?
a) Lateral cord only
b) Medial cord only
c) Both medial and lateral cords
d) Posterior cord
Explanation: Median nerve arises from contributions of both medial and lateral cords. Correct answer: Both medial and lateral cords. Clinical: median nerve lesions cause loss of thumb opposition and ape thumb deformity.
7) Posterior cord of brachial plexus gives rise to:
a) Axillary and radial nerves
b) Ulnar and radial nerves
c) Median and ulnar nerves
d) Musculocutaneous and median nerves
Explanation: Posterior cord terminates as axillary and radial nerves. Correct answer: Axillary and radial nerves. Clinical: injuries affect shoulder abduction and wrist extension respectively.
8) Klumpke’s palsy involves which roots?
a) C5–C6
b) C7
c) C8–T1
d) C5–C7
Explanation: Klumpke’s palsy occurs due to injury to C8–T1 roots, affecting intrinsic hand muscles. Correct answer: C8–T1. Clinical: causes claw hand deformity and sensory loss in medial forearm and hand.
9) Erb’s palsy involves paralysis of:
a) Flexors of forearm
b) Extensors of wrist
c) Abductors and lateral rotators of shoulder
d) Intrinsic muscles of hand
Explanation: Erb’s palsy occurs due to C5–C6 root lesion, affecting deltoid, supraspinatus, infraspinatus, and biceps. Correct answer: Abductors and lateral rotators of shoulder. Clinical: arm hangs medially rotated, extended, pronated (“waiter’s tip”).
10) Which nerve accompanies posterior circumflex humeral artery?
a) Musculocutaneous nerve
b) Radial nerve
c) Axillary nerve
d) Median nerve
Explanation: Axillary nerve travels with posterior circumflex humeral artery through quadrangular space. Correct answer: Axillary nerve. Clinical: injured in surgical neck fractures of humerus.
Keyword Definitions
• Clavipectoral fascia – A strong sheet of connective tissue deep to pectoralis major, enclosing subclavius and pectoralis minor.
• Costocoracoid ligament – Thickened portion of clavipectoral fascia between coracoid process and first rib; origin of fascia.
• Coracoacromial ligament – Connects coracoid process to acromion, prevents superior displacement of humeral head.
• Coracoclavicular ligament – Strong stabilizer between clavicle and coracoid process; prevents clavicle dislocation.
• Costoclavicular ligament – Connects first rib to clavicle, stabilizes sternoclavicular joint.
• Pectoralis minor – Muscle enclosed by clavipectoral fascia, important surgical landmark.
• Subclavius – Small muscle beneath clavicle, enclosed by clavipectoral fascia.
• Axilla – Space under shoulder joint, bounded anteriorly by clavipectoral fascia, important for neurovascular structures.
• Clinical relevance – Thickening or fibrosis of clavipectoral fascia can compress neurovascular bundle in thoracic outlet syndrome.
• Fascia – Connective tissue layers enveloping muscles, vessels, and nerves.
Chapter: Anatomy / Thorax
Topic: Pectoral Region
Subtopic: Clavipectoral Fascia and Ligament Derivation
Lead Question – 2013
Clavipectoral fascia is derived from which ligament?
a) Coracoacromial
b) Coracoclavicular
c) Costoclavicular
d) Costocoracoid
Explanation: Clavipectoral fascia originates from the costocoracoid ligament, extending between the coracoid process and first rib. Correct answer: Costocoracoid ligament. This fascia encloses subclavius and pectoralis minor, forming an important surgical plane. Clinical: In axillary surgeries, the fascia acts as a guide for neurovascular structures, preventing accidental injuries.
Guessed Questions for NEET PG
1) Clavipectoral fascia encloses:
a) Pectoralis major
b) Subclavius and pectoralis minor
c) Serratus anterior
d) Latissimus dorsi
Explanation: The fascia encloses subclavius and pectoralis minor, not pectoralis major. Correct answer: Subclavius and pectoralis minor. Clinical: important in surgical dissection of axilla.
2) Costoclavicular ligament connects:
a) Clavicle to sternum
b) Clavicle to coracoid
c) Clavicle to first rib
d) Coracoid to acromion
Explanation: Costoclavicular ligament stabilizes sternoclavicular joint by attaching clavicle to first rib. Correct answer: Clavicle to first rib. Clinical: damage leads to sternoclavicular instability.
3) Coracoacromial ligament prevents:
a) Inferior displacement of humerus
b) Superior displacement of humeral head
c) Lateral displacement of scapula
d) Medial rotation of clavicle
Explanation: It forms an arch preventing superior displacement of humeral head. Correct answer: Superior displacement of humeral head. Clinical: impingement syndrome occurs beneath this arch.
4) Clavipectoral fascia pierces to transmit:
a) Cephalic vein
b) Axillary vein
c) Subclavian artery
d) Long thoracic nerve
Explanation: Cephalic vein pierces the clavipectoral fascia near deltopectoral groove. Correct answer: Cephalic vein. Clinical: serves as an important venous access site.
5) Axillary sheath is continuous with:
a) Pretracheal fascia
b) Prevertebral fascia
c) Carotid sheath
d) Endothoracic fascia
Explanation: Axillary sheath is extension of prevertebral fascia enclosing axillary vessels and brachial plexus. Correct answer: Prevertebral fascia. Clinical: important in brachial plexus block anesthesia.
6) Subclavius muscle is innervated by:
a) Nerve to subclavius
b) Lateral pectoral nerve
c) Medial pectoral nerve
d) Long thoracic nerve
Explanation: Subclavius is innervated by nerve to subclavius (C5–C6). Correct answer: Nerve to subclavius. Clinical: weakness may cause instability of clavicle in trauma.
7) Pectoralis minor inserts into:
a) Acromion
b) Coracoid process
c) Clavicle
d) Glenoid cavity
Explanation: Pectoralis minor attaches to coracoid process of scapula. Correct answer: Coracoid process. Clinical: hypertrophy or contracture compresses brachial plexus in thoracic outlet syndrome.
8) Which structure lies deep to clavipectoral fascia?
a) Axillary vein
b) Axillary artery
c) Brachial plexus cords
d) All of the above
Explanation: Clavipectoral fascia overlies axillary vessels and brachial plexus cords. Correct answer: All of the above. Clinical: careful dissection required to avoid vascular/nerve injury.
9) Surgical importance of clavipectoral fascia?
a) Defines axillary surgical plane
b) Stabilizes clavicle
c) Prevents scapular rotation
d) Guides deltoid function
Explanation: Clavipectoral fascia acts as an anatomical landmark for axillary surgeries. Correct answer: Defines axillary surgical plane. Clinical: helps surgeons avoid injury to axillary neurovascular structures.
10) Coracoclavicular ligament function is:
a) Stabilize acromioclavicular joint
b) Stabilize sternoclavicular joint
c) Prevent humeral displacement
d) Reinforce shoulder capsule
Explanation: Coracoclavicular ligament is the major stabilizer of acromioclavicular joint. Correct answer: Stabilize acromioclavicular joint. Clinical: rupture causes “shoulder separation” injury.
Chapter: Upper Limb Anatomy
Topic: Bones of the Hand
Subtopic: Ossification of Carpal Bones
Keyword Definitions
Carpal bones: Eight small bones forming the wrist (carpus).
Ossification: The process of bone formation from cartilage or fibrous tissue.
Capitate: One of the carpal bones, centrally located in the distal carpal row.
Lunate: A crescent-shaped carpal bone in the proximal row, beside the scaphoid.
Pisiform: A small, pea-shaped carpal bone on the ulnar side of the wrist.
Trapezium: A carpal bone at the base of the thumb, distal row.
Clinical: Relating to the observation and treatment of patients.
Radiograph: An image produced on sensitive plate or film by X-rays.
Developmental anatomy: Study of the structural changes of organisms as they grow.
Epiphysis: The end part of a long bone, initially growing separately from the shaft.
NEET PG: National Eligibility cum Entrance Test for Postgraduate medical courses in India.
Lead Question – 2013
First carpal bone to appear is?
a) Trapezium
b) Capitate
c) Pisiform
d) Lunate
Explanation: The **capitate** is the first carpal bone to ossify, usually around the 2nd month after birth. This knowledge is essential in pediatric radiology and helps assess bone age.
Guessed MCQs for NEET PG
1. Which carpal bone is last to ossify?
a) Trapezium
b) Capitate
c) Pisiform
d) Lunate
Explanation: Pisiform is the last carpal bone to ossify, appearing around 9–12 years, important for assessing delayed skeletal development in children.
2. Order of appearance of carpal bones in ossification usually is:
a) Capitate, Hamate, Triquetrum
b) Capitate, Trapezium, Pisiform
c) Lunate, Scaphoid, Hamate
d) Scaphoid, Capitate, Pisiform
Explanation: Capitate appears first, followed by Hamate and then Triquetrum, helping in the assessment of normal growth on X-rays.
3. A 3-year-old child’s wrist X-ray lacks ossified pisiform. This is:
a) Normal finding
b) Suggestive of delayed bone age
c) Indicates rickets
d) Indicates infection
Explanation: Absence of pisiform at 3 years is normal; pisiform ossifies around 9–12 years, so this is not a sign of pathology.
4. Which carpal bone lies most medial in anatomical position?
a) Scaphoid
b) Pisiform
c) Hamate
d) Lunate
Explanation: Pisiform is the most medial carpal bone when the palm faces up, a key landmark in wrist anatomy.
5. The bone forming the floor of the anatomical snuffbox is:
a) Lunate
b) Hamate
c) Trapezium
d) Scaphoid
Explanation: The scaphoid forms the floor of the anatomical snuffbox, relevant to wrist trauma assessment.
6. Which carpal bone articulates with the first metacarpal?
a) Capitate
b) Scaphoid
c) Trapezium
d) Trapezoid
Explanation: Trapezium articulates with the first metacarpal and is crucial for thumb movement, especially in clinical settings like Bennett's fracture.
7. A fall on outstretched hand may most likely fracture:
a) Capitate
b) Scaphoid
c) Pisiform
d) Lunate
Explanation: Scaphoid is most commonly fractured due to its anatomical position and vascular supply, important for recognizing complications like avascular necrosis.
8. In carpal tunnel syndrome, which bone forms the floor of the tunnel?
a) Pisiform
b) Hamate
c) Lunate
d) Capitate
Explanation: The floor of the carpal tunnel is formed by the carpal bones, including the **capitate**, important for understanding clinical symptoms and management.
9. An eight-year-old’s wrist X-ray shows ossified capitate and hamate only. Bone age is approximately:
a) 1 year
b) 3 years
c) 5 years
d) 8 years
Explanation: Capitate and hamate ossify by age 1; absence of other carpal bones suggests bone age is around 1 year, valuable for pediatric growth assessment.
10. The capitate bone is located in which row of the carpus?
a) Proximal row
b) Distal row
c) Both rows
d) Middle row
Explanation: The capitate is centrally located in the distal row of carpal bones, a crucial point in wrist biomechanics.
Chapter: Anatomy
Topic: Nose and Paranasal Sinuses
Subtopic: Osteocartilaginous Junction
Keywords:
Osseocartilaginous junction: The anatomical transition zone between bony and cartilaginous parts of the nose.
Nasion: Junction of frontal bone and nasal bones, an external landmark.
Rhinion: Lower end of nasal bones, where bone meets cartilage.
Radix: Root of the nose, depression between forehead and nose.
Columella: Soft tissue between nostrils, separating nasal vestibules.
Clinical relevance: Important in nasal fractures, rhinoplasty, and septal surgeries.
Lead Question - 2012
Osseocartilagenous junction is present at ?
a) Nasion
b) Rhinion
c) Radix
d) Columella
Explanation: The osseocartilaginous junction lies at the rhinion, where the nasal bone meets the upper lateral cartilage. This is clinically important in trauma assessment and nasal surgeries. Correct answer is Rhinion.
Guessed Question 1
Which external landmark corresponds to the junction of frontal bone and nasal bones?
a) Nasion
b) Rhinion
c) Radix
d) Columella
Explanation: The nasion marks the point where the two nasal bones meet the frontal bone. It is used in cephalometry and as a surgical landmark. Correct answer is Nasion.
Guessed Question 2
The radix of the nose is located at?
a) Upper nasal root
b) Mid-nasal dorsum
c) Lower end of septum
d) Alar cartilage junction
Explanation: The radix refers to the root of the nose, situated between the forehead and the dorsum of the nose, often depressed in profile. Correct answer is Upper nasal root.
Guessed Question 3
The soft tissue that separates the nostrils externally is called?
a) Septum
b) Columella
c) Alar base
d) Vestibule
Explanation: The columella is the soft tissue and cartilage that separates the nostrils externally. It is important in cosmetic and reconstructive nasal surgeries. Correct answer is Columella.
Guessed Question 4
Osseocartilaginous junction is a common site for?
a) Septal hematoma
b) Nasal fracture
c) Epistaxis (Little’s area)
d) Foreign body impaction
Explanation: The osseocartilaginous junction at the rhinion is often involved in nasal fractures due to trauma, as it forms a weak point between rigid bone and flexible cartilage. Correct answer is Nasal fracture.
Guessed Question 5
In rhinoplasty, altering the nasal dorsum often involves working at?
a) Nasion
b) Rhinion
c) Columella
d) Alar rim
Explanation: Rhinoplasty commonly involves reshaping the dorsum, where the rhinion marks the osseocartilaginous junction, critical for aesthetic nasal contour. Correct answer is Rhinion.
Guessed Question 6
A blow to the mid-nasal region most likely fractures at?
a) Rhinion
b) Nasion
c) Columella
d) Radix
Explanation: The rhinion is the weakest point of the nasal dorsum where bone meets cartilage, making it the most common fracture site in trauma. Correct answer is Rhinion.
Guessed Question 7
Which landmark is most useful for radiographic assessment of nasal fractures?
a) Columella
b) Rhinion
c) Radix
d) Nasion
Explanation: The nasion is an external landmark used in radiographs for nasal bone fractures. It provides a reliable point for alignment and surgical planning. Correct answer is Nasion.
Guessed Question 8
Epistaxis in Little’s area arises near which junction?
a) Osseocartilaginous junction
b) Septal cartilage only
c) Inferior turbinate junction
d) Maxillary sinus ostium
Explanation: Little’s area (Kiesselbach’s plexus) is located at the anterior septum, close to the osseocartilaginous junction, making it a frequent site of anterior epistaxis. Correct answer is Osseocartilaginous junction.
Guessed Question 9
Basal cell carcinoma of the nose commonly affects?
a) Rhinion
b) Columella
c) Nasal ala
d) Nasion
Explanation: Basal cell carcinoma frequently affects sun-exposed areas such as the nasal ala and dorsum, sparing deeper structures initially. Correct answer is Nasal ala.
Guessed Question 10
Septoplasty commonly corrects deviation at?
a) Columella
b) Rhinion
c) Nasion
d) Posterior choana
Explanation: Septoplasty often addresses deviations near the osseocartilaginous junction (rhinion), as this area frequently causes airway obstruction. Correct answer is Rhinion.
Chapter: Neck Anatomy
Topic: Subclavian Artery & Its Branches
Subtopic: Thyrocervical Trunk — Origin and Branches
Keyword Definitions
Subclavian artery (parts): Divided into three parts in relation to anterior scalene: 1st (medial), 2nd (behind), 3rd (lateral).
Thyrocervical trunk: Short branch arising from the 1st part of subclavian; gives inferior thyroid, ascending cervical, transverse cervical, and suprascapular arteries.
Inferior thyroid artery: Branch supplying lower thyroid, parathyroids, and cervical branches; important in thyroid surgery.
Suprascapular artery: Supplies supraspinatus and infraspinatus regions; runs toward scapular notch.
Transverse cervical artery: Supplies trapezius and posterior neck; has superficial and deep branches.
Ascending cervical artery: Small vertical branch accompanying phrenic or vertebral levels.
Vertebral artery: Another branch of 1st part, ascends through transverse foramina to brainstem.
Clinical relevance: Knowledge of thyrocervical trunk anatomy is vital during central line placement, neck dissection, and thyroid surgery to avoid bleeding.
Lead Question - 2012
The thyrocervical trunk is a branch of which part of subclavian artery?
a) 1st
b) 2nd
c) 3rd
d) None
Explanation: The thyrocervical trunk arises from the first (medial) part of the subclavian artery, proximal to the anterior scalene muscle. It is a short, stout trunk giving inferior thyroid, ascending cervical, transverse cervical and suprascapular arteries. Surgeons must note this relation during neck and thyroid operations. Answer: a) 1st.
1. Which of the following is NOT a usual branch of the thyrocervical trunk?
a) Inferior thyroid artery
b) Transverse cervical artery
c) Suprascapular artery
d) Internal thoracic artery
Explanation: The internal thoracic artery is a branch of the subclavian but arises from the first part directly and not from the thyrocervical trunk. Thyrocervical trunk typically gives inferior thyroid, transverse cervical, suprascapular, and sometimes ascending cervical branches. Answer: d) Internal thoracic artery.
2. The inferior thyroid artery supplies all EXCEPT:
a) Lower pole of thyroid
b) Parathyroid glands
c) Larynx (via branches)
d) Supraorbital region
Explanation: Inferior thyroid artery supplies the lower thyroid, parathyroids and gives laryngeal branches but does not supply the supraorbital region which is served by branches of the ophthalmic artery. It is a branch of the thyrocervical trunk. Answer: d) Supraorbital region.
3. The transverse cervical artery commonly supplies which muscle?
a) Sternocleidomastoid
b) Trapezius (superficial branch)
c) Levator scapulae only
d) Diaphragm
Explanation: The superficial branch of the transverse cervical artery supplies the trapezius muscle and overlying skin. The deep branch (dorsal scapular) may supply levator scapulae and rhomboids. It arises from or near the thyrocervical trunk region. Answer: b) Trapezius (superficial branch).
4. During a thyroid lobectomy, which vessel must be carefully ligated to preserve laryngeal blood supply?
a) Superior thyroid artery
b) Inferior thyroid artery
c) Lingual artery
d) Facial artery
Explanation: Ligation of the inferior thyroid artery risks compromising inferior laryngeal branches; surgeons clip branches close to the thyroid capsule to preserve recurrent laryngeal artery supply. The inferior thyroid commonly arises from the thyrocervical trunk. Answer: b) Inferior thyroid artery.
5. The thyrocervical trunk most often arises medial to which muscle?
a) Anterior scalene
b) Middle scalene
c) Posterior scalene
d) Levator scapulae
Explanation: The first part of the subclavian and its branches, including the thyrocervical trunk and vertebral artery, lie medial to the anterior scalene muscle. This anatomical relation is crucial during central venous access and neck surgery. Answer: a) Anterior scalene.
6. A variant artery arising from thyrocervical trunk ascending along cervical vertebrae is called:
a) Ascending cervical artery
b) Superior thyroid artery
c) Deep cervical artery
d) Occipital artery
Explanation: The ascending cervical artery is a small upward branch often arising from the inferior thyroid or thyrocervical trunk, supplying neck muscles and vertebral bodies; it anastomoses with deep cervical branches. Answer: a) Ascending cervical artery.
7. Injury to the suprascapular artery may compromise blood supply to:
a) Supraspinatus and infraspinatus muscles
b) Levator scapulae primarily
c) Pectoralis major
d) Biceps brachii
Explanation: The suprascapular artery travels to the scapular region and supplies supraspinatus and infraspinatus muscles via scapular anastomoses; it commonly originates from the thyrocervical trunk. Damage affects shoulder girdle perfusion. Answer: a) Supraspinatus and infraspinatus muscles.
8. The vertebral artery arises from which part of the subclavian?
a) 1st part
b) 2nd part
c) 3rd part
d) It varies
Explanation: The vertebral artery classically arises from the first part of the subclavian artery and ascends through transverse foramina to supply posterior brain. Its proximity to thyrocervical trunk branches is important in cervical vascular anatomy. Answer: a) 1st part.
9. In posterior triangle bleeding from thyrocervical branches is best controlled by ligating which artery proximally?
a) Subclavian artery (first part)
b) External carotid artery
c) Vertebral artery
d) Internal thoracic artery
Explanation: Major control of bleeding from thyrocervical branches may require proximal control of the subclavian artery (first part) or selective ligation of offending branch at origin; external carotid ligation will not stop these branches. Answer: a) Subclavian artery (first part).
10. Which imaging modality best delineates small branches of thyrocervical trunk preoperatively?
a) Digital subtraction angiography (DSA)
b) Plain X-ray
c) Chest radiograph
d) EEG
Explanation: DSA provides high-resolution dynamic visualization of arterial branches, ideal for planning embolization or surgery involving the thyrocervical trunk and its branches. CT angiography is an alternative noninvasive option, but DSA remains gold standard for small branch delineation. Answer: a) Digital subtraction angiography (DSA).
Chapter: Lower Limb Anatomy
Topic: Hip Joint & Thigh Muscles
Subtopic: Hip Flexors — Actions and Clinical Relevance
Keyword Definitions
Psoas major — Primary hip flexor arising from lumbar vertebrae; strong hip flexion and trunk flexion contribution.
Iliacus — Joins psoas major to form iliopsoas, major hip flexor inserting on lesser trochanter.
Iliopsoas — Combined tendon of psoas major and iliacus; principal flexor of hip.
Biceps femoris — Hamstring muscle; knee flexor and hip extensor (long head) — not primary hip flexor.
Gluteus maximus — Principal hip extensor and external rotator; active in rising from sitting.
Tensor fasciae latae (TFL) — Assists hip flexion, abduction, and medial rotation; acts via IT band.
Hip flexion — Movement decreasing angle between thigh and trunk; important for walking, climbing stairs.
Femoral nerve — Supplies iliacus and provides motor to quadriceps; psoas major supplied by lumbar plexus.
Hip pathology — Iliopsoas bursitis or tendonitis causes anterior hip pain aggravated by flexion.
Clinical test — Straight leg raise and resisted hip flexion assess iliopsoas function.
Lead Question - 2012
Muscle causing flexion of hip ?
a) Biceps femoris
b) Psoas major
c) Gluteus maximus
d) TFL
Explanation: The psoas major, together with iliacus (forming iliopsoas), is the principal hip flexor producing powerful flexion at the hip and contributing to trunk flexion. Biceps femoris and gluteus maximus are extensors; TFL assists flexion but is not the primary flexor. Answer: b) Psoas major.
1. Primary nerve supply to psoas major is from:
a) Femoral nerve
b) Lumbar plexus (L1–L3)
c) Sciatic nerve
d) Superior gluteal nerve
Explanation: Psoas major receives direct branches from the anterior rami of lumbar spinal nerves (L1–L3) constituting part of the lumbar plexus. Femoral nerve supplies iliacus; sciatic and superior gluteal do not innervate psoas. Clinically lumbar radiculopathy affects hip flexion. Answer: b) Lumbar plexus (L1–L3).
2. Iliopsoas tendonitis typically causes pain at:
a) Posterior thigh
b) Anterior groin/hip
c) Lateral knee
d) Medial ankle
Explanation: Iliopsoas tendonitis presents with anterior groin or hip pain aggravated by hip flexion, resisted straight leg raise, and climbing stairs. It may mimic intra-articular hip pathology; ultrasound or MRI helps confirm tendon inflammation. Answer: b) Anterior groin/hip.
3. Which muscle is a synergist to psoas major in hip flexion?
a) Gluteus maximus
b) Rectus femoris
c) Adductor magnus
d) Piriformis
Explanation: Rectus femoris (part of quadriceps) crosses the hip and assists in hip flexion, acting as a synergist to iliopsoas during activities requiring powerful hip flexion like kicking. Gluteus maximus is an extensor. Answer: b) Rectus femoris.
4. A patient cannot flex hip against resistance but can on gravity-eliminated plane. This suggests:
a) Complete nerve transection
b) Pain inhibition or partial weakness
c) Intact motor function
d) Labral tear
Explanation: Inability to perform resisted hip flexion with preserved movement on gravity-eliminated testing suggests pain inhibition or partial weakness (Grade 3/5), not complete denervation. Further neuro exam and imaging are warranted. Answer: b) Pain inhibition or partial weakness.
5. InTrendelenburg gait is due to weakness of:
a) Hip flexors (psoas)
b) Hip abductors (gluteus medius/minimus)
c) Knee extensors (quadriceps)
d) Ankle dorsiflexors
Explanation: Trendelenburg gait arises from weak hip abductors (gluteus medius/minimus) causing pelvic drop on contralateral side during stance. Psoas weakness causes hip flexion issues but not Trendelenburg. Examination differentiates abductor from flexor pathology. Answer: b) Hip abductors (gluteus medius/minimus).
6. Which motion is produced when iliopsoas acts bilaterally on fixed femur?
a) Trunk extension
b) Trunk flexion (sit-up)
c) Lateral rotation of femur
d) Hip abduction
Explanation: With femur fixed, bilateral contraction of iliopsoas flexes the trunk at the hip (raises the torso), assisting in sit-up movements. It also stabilizes lumbar spine during posture. Dysfunction impairs rising from supine. Answer: b) Trunk flexion (sit-up).
7. Which muscle primarily extends the hip and opposes psoas major?
a) Iliacus
b) Gluteus maximus
c) Sartorius
d) Pectineus
Explanation: Gluteus maximus is the main hip extensor providing power for rising, climbing, and sprinting, functionally opposing the flexion produced by iliopsoas. Injury to gluteus maximus alters gait and seated-to-standing mechanics. Answer: b) Gluteus maximus.
8. Tight psoas major may cause which postural change?
a) Increased lumbar lordosis
b) Flattened lumbar curve
c) Thoracic kyphosis reduction
d) Knee hyperextension
Explanation: A tight psoas pulls lumbar spine into increased lordosis and anterior pelvic tilt, contributing to low back pain and altered gait mechanics. Stretching and posture correction are part of management. Answer: a) Increased lumbar lordosis.
9. Hip flexion power is most compromised by lesion of which structure?
a) Femoral nerve root (L2–L4)
b) Sciatic nerve
c) Obturator nerve
d) Superior gluteal nerve
Explanation: The femoral nerve innervates iliacus and rectus femoris; lesion reduces hip flexion strength. Psoas major is from lumbar plexus; combined lesions of L2–L4 significantly impair hip flexion. Sciatic/obturator/gluteal affect other functions. Answer: a) Femoral nerve root (L2–L4).
10. Clinical test for iliopsoas strength: patient performs?
a) Hip abduction against resistance
b) Resisted hip flexion in sitting or supine
c) Heel raise
d) Knee flexion
Explanation: Resisted hip flexion (patient attempts to lift thigh against resistance) assesses iliopsoas/rectus femoris. Positive pain or weakness suggests tendonitis, nerve lesion, or muscular tear and guides further imaging or EMG. Answer: b) Resisted hip flexion in sitting or supine.
Chapter: Head & Neck Anatomy
Topic: Facial Muscles
Subtopic: Risorius — anatomy, function, and clinical relevance
Keyword Definitions
Risorius — thin superficial facial muscle that retracts the angle of the mouth producing a smile or grimace.
Facial expression muscles — group of muscles innervated by the facial nerve (CN VII) that move the skin of the face.
Mastication muscles — muscles of chewing (masseter, temporalis, pterygoids) innervated by V3, not risorius.
Buccinator — deep cheek muscle that compresses the cheek and assists in mastication; distinct from risorius.
Facial nerve (CN VII) — motor nerve supplying muscles of facial expression including risorius; vulnerable in parotid surgery.
Parotid gland/plexus — risorius lies superficially near parotid region; surgical injury may affect branches of CN VII causing asymmetry.
Botulinum toxin — used cosmetically/therapeutically on facial muscles; improper injection into risorius can alter smile.
Deglutition — swallowing; risorius is not a primary deglutition muscle.
Orbicularis oris — sphincter muscle of the mouth working with risorius for lip movement.
Clinical sign — loss of risorius function causes inability to retract mouth corner, mild asymmetrical smile.
Lead Question - 2012
Risorius is a muscle of ?
a) Mastication
b) Deglutition
c) Facial expression
d) Eye movement
Explanation: Risorius is a superficial muscle of facial expression that retracts the angle of the mouth laterally, contributing to smiling or grimacing. It is innervated by branches of the facial nerve (CN VII) and not involved in mastication, swallowing, or eye movement. Answer: c) Facial expression.
1. Which nerve supplies the risorius muscle?
a) Mandibular division of trigeminal (V3)
b) Facial nerve (CN VII)
c) Hypoglossal nerve (XII)
d) Glossopharyngeal nerve (IX)
Explanation: The risorius receives motor innervation from the facial nerve (CN VII) via its buccal or zygomatic branches. Damage to these branches (eg. parotid surgery) produces weakness of mouth corner retraction and a flattened or asymmetrical smile. Answer: b) Facial nerve (CN VII).
2. Primary action of risorius is to:
a) Elevate mandible
b) Protrude tongue
c) Retract angle of mouth laterally
d) Close eyelids
Explanation: Risorius retracts the angle of the mouth laterally, creating a grin or stretched smile. It acts with levator and depressor muscles to modulate facial expression. It does not elevate the mandible, move the tongue, or close eyelids. Answer: c) Retract angle of mouth laterally.
3. Injury to the facial nerve branch supplying risorius produces which clinical sign?
a) Difficulty in chewing
b) Loss of forehead wrinkling
c) Asymmetrical smile with inability to retract mouth corner
d) Diplopia
Explanation: Paralysis of the branch to risorius causes inability to retract the mouth corner on the affected side, producing an asymmetrical smile. Chewing is mainly V3, forehead wrinkling involves temporal branch, and diplopia relates to ocular muscles. Answer: c) Asymmetrical smile with inability to retract mouth corner.
4. During parotid surgery, which precaution least helps preserve risorius function?
a) Identify and protect facial nerve branches
b) Minimize traction on superficial fascia
c) Avoid deep incisions through masseter
d) Only operate via transoral approach crossing buccinator
Explanation: Preserving facial nerve branches and superficial fascia protects risorius. A transoral approach through buccinator (d) risks injuring branches and the duct, making it least protective. Avoiding deep masseter incisions is sensible. Answer: d) Only operate via transoral approach crossing buccinator.
5. Which muscle lies deep to risorius and assists cheek flattening during mastication?
a) Buccinator
b) Masseter
c) Platysma
d) Levator labii superioris
Explanation: Buccinator lies deep to risorius and compresses the cheek to keep food between teeth, aiding mastication. Masseter is a primary masticator on lateral face. Risorius superficially retracts mouth corner and does not assist primary chewing. Answer: a) Buccinator.
6. Cosmetic injection of botulinum toxin into risorius is used to treat:
a) Jaw claudication
b) Gummy smile due to hyperactive smiling muscles
c) Orbital cellulitis
d) Vocal cord paralysis
Explanation: Botulinum toxin into risorius and adjacent smile muscles can reduce hyperactive lateral mouth retraction contributing to a gummy or asymmetrical smile. Careful dosing prevents excessive weakness and smile distortion. It is not used for orbital cellulitis or vocal cord issues. Answer: b) Gummy smile due to hyperactive smiling muscles.
7. Anatomical variation: risorius commonly inserts into:
a) Skin at angle of mouth
b) Mandibular ramus
c) Zygomatic arch
d) Hyoid bone
Explanation: Risorius inserts into the skin at the angle of mouth (modiolus area), pulling it laterally. It does not attach to bony structures like the mandible, zygoma, or hyoid. Variations in origin exist but insertion is cutaneous. Answer: a) Skin at angle of mouth.
8. Which clinical test assesses risorius function specifically?
a) Puff cheeks against closed lips
b) Ask patient to show teeth and retract mouth corners laterally
c) Test tongue protrusion
d) Assess shoulder shrug
Explanation: Asking the patient to smile broadly and retract mouth corners tests risorius among other smile muscles; inability to retract the affected corner suggests weakness. Puffing cheeks tests buccinator and orbicularis, tongue relates to XII, shoulder shrug to accessory nerve. Answer: b) Ask patient to show teeth and retract mouth corners laterally.
9. In facial nerve palsy localized to the buccal branches, which action is most compromised?
a) Eye closure
b) Nasolabial fold flattening and mouth corner retraction
c) Head rotation
d) Tongue movement
Explanation: Buccal branches supply muscles of the midface including risorius, buccinator, and orbicularis oris; palsy flattens nasolabial fold and impairs mouth corner retraction and labial competence. Eye closure is temporal/zygomatic, head rotation accessory. Answer: b) Nasolabial fold flattening and mouth corner retraction.
10. Which statement about risorius is FALSE?
a) It is a muscle of facial expression
b) It aids chewing by elevating mandible
c) It is innervated by CN VII
d) It inserts into skin at mouth angle
Explanation: Risorius does not aid mastication or elevate the mandible; that is the role of masseter and temporalis. Risorius retracts mouth angle, is innervated by facial nerve, and inserts into the skin at the angle of the mouth. Answer: b) It aids chewing by elevating mandible (FALSE).
Chapter: Larynx
Topic: Cartilages of Larynx
Subtopic: Types of Laryngeal Cartilages
Keyword Definitions
Hyaline cartilage: Firm, translucent cartilage found in laryngeal cartilages like thyroid, cricoid, arytenoid. Can ossify with age.
Elastic cartilage: Flexible cartilage that retains shape, seen in epiglottis, corniculate, cuneiform.
Cricoid cartilage: Only complete ring of cartilage in airway, hyaline in nature.
Epiglottis: Leaf-shaped, elastic cartilage preventing aspiration.
Arytenoid cartilage: Paired hyaline cartilages with muscular and vocal processes.
Corniculate & cuneiform cartilages: Accessory elastic cartilages for laryngeal support.
Ossification of laryngeal cartilages: Hyaline cartilages ossify with age; elastic do not.
Lead Question – 2012
Which of the following laryngeal cartilage is hyaline?
a) Epiglottis
b) Corniculate
c) Cricoid
d) Cuneiform
Explanation: Cricoid cartilage is a hyaline cartilage forming the only complete ring of the airway. Unlike elastic cartilages such as epiglottis, corniculate, and cuneiform, hyaline cartilages like cricoid and thyroid tend to calcify with age. Correct answer is c) Cricoid.
Guessed Questions (NEET PG Style)
All of the following laryngeal cartilages are hyaline except
a) Thyroid
b) Arytenoid
c) Corniculate
d) Cricoid
Explanation: Except corniculate, all listed are hyaline cartilages. Corniculate is elastic cartilage, paired and small. Correct answer is c) Corniculate.
Which laryngeal cartilage ossifies earliest with age?
a) Epiglottis
b) Cricoid
c) Corniculate
d) Cuneiform
Explanation: Cricoid is the earliest to ossify among hyaline cartilages. Elastic cartilages such as epiglottis and corniculate remain flexible lifelong. Correct answer is b) Cricoid.
A patient with hoarseness due to arthritis of laryngeal joint most likely has involvement of
a) Cricoarytenoid joint
b) Cricothyroid joint
c) Atlanto-occipital joint
d) Temporomandibular joint
Explanation: Hoarseness occurs when cricoarytenoid joints (between arytenoid and cricoid, both hyaline) are involved in rheumatoid arthritis. Correct answer is a) Cricoarytenoid joint.
Which cartilage provides attachment for vocal cords?
a) Arytenoid
b) Corniculate
c) Epiglottis
d) Cuneiform
Explanation: Arytenoid cartilages (hyaline) provide vocal process where vocal cords attach. They are essential for phonation. Correct answer is a) Arytenoid.
Which laryngeal cartilage is leaf-shaped and elastic?
a) Cricoid
b) Epiglottis
c) Arytenoid
d) Thyroid
Explanation: Epiglottis is a flexible, elastic cartilage preventing aspiration during swallowing. Correct answer is b) Epiglottis.
All laryngeal cartilages ossify with age except
a) Thyroid
b) Cricoid
c) Arytenoid
d) Epiglottis
Explanation: Elastic cartilages like epiglottis, corniculate, and cuneiform do not ossify. Hyaline cartilages ossify with age. Correct answer is d) Epiglottis.
The only complete ring of cartilage in the airway is
a) Thyroid
b) Epiglottis
c) Cricoid
d) Arytenoid
Explanation: Cricoid cartilage forms a complete ring, unlike thyroid (open posteriorly). It supports airway and marks level of C6 vertebra. Correct answer is c) Cricoid.
In intubation, which cartilage is pressed for cricoid pressure (Sellick’s maneuver)?
a) Arytenoid
b) Thyroid
c) Cricoid
d) Epiglottis
Explanation: Cricoid cartilage is pressed to prevent aspiration by occluding esophagus during intubation. Correct answer is c) Cricoid.
Which muscle attaches to the muscular process of arytenoid cartilage?
a) Lateral cricoarytenoid
b) Cricothyroid
c) Posterior cricoarytenoid
d) Both a and c
Explanation: Muscular process of arytenoid gives attachment to posterior and lateral cricoarytenoid muscles controlling vocal cord movements. Correct answer is d) Both a and c.
Elastic cartilage of larynx includes all except
a) Epiglottis
b) Corniculate
c) Cuneiform
d) Arytenoid
Explanation: Arytenoid is hyaline cartilage, whereas epiglottis, corniculate, and cuneiform are elastic. Correct answer is d) Arytenoid.
During swallowing, which laryngeal cartilage protects airway by covering inlet?
a) Epiglottis
b) Corniculate
c) Cuneiform
d) Arytenoid
Explanation: Epiglottis (elastic cartilage) folds backward during swallowing, preventing aspiration. Correct answer is a) Epiglottis.
Chapter: Pharynx
Topic: Pharyngeal Muscles
Subtopic: Passavant's Ridge
Keyword Definitions:
Passavant's Ridge: A mucosal ridge on the posterior pharyngeal wall during swallowing.
Superior Constrictor: Muscle forming posterior pharyngeal wall, contributes to ridge formation.
Palatopharyngeus: Elevates pharynx and contributes fibers to ridge.
Palatoglossus: Muscle connecting tongue to palate, not involved in ridge.
Inferior Constrictor: Pharyngeal constrictor muscle, not part of ridge formation.
Soft Palate: Muscular fold that closes nasopharynx during swallowing.
Lead Question – 2012
Passavant ridge ?
a) Superior constrictor and palatopharyngeus
b) Inferior constrictor and palatopharyngeus
c) Superior constrictor and palatoglossus
d) Inferior constrictor and palatoglossus
Explanation: Passavant’s ridge is a mucosal prominence formed by contraction of the superior constrictor and palatopharyngeus during swallowing. It helps close the nasopharyngeal isthmus against the soft palate, preventing nasal regurgitation. Correct answer: a) Superior constrictor and palatopharyngeus.
Question 2. A patient presents with nasal regurgitation of liquids. Dysfunction of which structure is most likely?
a) Palatoglossus
b) Passavant’s ridge
c) Cricopharyngeus
d) Stylopharyngeus
Explanation: Failure of Passavant’s ridge to form properly prevents closure of nasopharynx, causing nasal regurgitation. This occurs with superior constrictor or palatopharyngeus weakness. Correct answer: b) Passavant’s ridge.
Question 3. Which muscle is primarily responsible for preventing food from entering the nasopharynx?
a) Tensor veli palatini
b) Levator veli palatini
c) Palatopharyngeus
d) Stylopharyngeus
Explanation: Levator veli palatini elevates the soft palate, sealing the nasopharynx against Passavant’s ridge. This prevents regurgitation during swallowing. Correct answer: b) Levator veli palatini.
Question 4. A lesion of glossopharyngeal nerve will most likely cause?
a) Absent gag reflex
b) Nasal regurgitation
c) Hoarseness
d) Shoulder droop
Explanation: Glossopharyngeal nerve supplies sensory input for gag reflex. Its lesion abolishes gag reflex while motor defects are due to vagus/accessory nerve. Correct answer: a) Absent gag reflex.
Question 5. Inadequate closure of the nasopharynx during swallowing is due to paralysis of?
a) Palatopharyngeus
b) Stylopharyngeus
c) Masseter
d) Cricopharyngeus
Explanation: Palatopharyngeus elevates pharynx and forms Passavant’s ridge. Its dysfunction leads to failure of nasopharyngeal closure. Correct answer: a) Palatopharyngeus.
Question 6. A child with cleft palate develops hypernasal speech. The main cause is?
a) Weak palatal closure
b) Weak tongue movement
c) Laryngeal stenosis
d) Weak jaw closure
Explanation: Hypernasality results from failure of palatal muscles and Passavant’s ridge to close the nasopharyngeal opening during speech. Correct answer: a) Weak palatal closure.
Question 7. Which nerve mediates motor supply to palatal muscles except tensor veli palatini?
a) Glossopharyngeal
b) Vagus via cranial accessory
c) Trigeminal mandibular
d) Hypoglossal
Explanation: All palatal muscles except tensor veli palatini are supplied by vagus through cranial part of accessory nerve. Correct answer: b) Vagus via cranial accessory.
Question 8. Which muscle is supplied by mandibular nerve among palatal muscles?
a) Levator veli palatini
b) Tensor veli palatini
c) Palatopharyngeus
d) Palatoglossus
Explanation: Tensor veli palatini is the only palatal muscle supplied by mandibular division of trigeminal nerve. Correct answer: b) Tensor veli palatini.
Question 9. A 40-year-old develops difficulty swallowing with nasal escape of food. Which nerve is most likely affected?
a) Vagus
b) Hypoglossal
c) Facial
d) Glossopharyngeal
Explanation: Vagus nerve supplies motor fibers to palatal muscles (via cranial accessory). Its lesion causes palatal paralysis and nasal regurgitation. Correct answer: a) Vagus.
Question 10. Which muscle contributes to Passavant’s ridge along with superior constrictor?
a) Palatopharyngeus
b) Palatoglossus
c) Stylopharyngeus
d) Tensor veli palatini
Explanation: Passavant’s ridge is formed by palatopharyngeus fibers joining superior constrictor contraction. Correct answer: a) Palatopharyngeus.
Question 11. A surgeon notes poor closure of nasopharynx after adenoidectomy. Which structure is compromised?
a) Passavant’s ridge
b) Cricopharyngeus
c) Stylopharyngeus
d) Tensor veli tympani
Explanation: If Passavant’s ridge or its contributing muscles are injured, nasopharyngeal closure fails, leading to regurgitation or hypernasal speech. Correct answer: a) Passavant’s ridge.
Chapter: Head & Neck
Topic: Pharynx
Subtopic: Killian’s dehiscence and constrictor muscles
Keyword definitions
Killian’s dehiscence — a weak area between thyropharyngeus and cricopharyngeus parts of the inferior constrictor, site of Zenker’s diverticulum.
Zenker’s diverticulum — pharyngoesophageal pouch protruding through Killian’s dehiscence causing dysphagia and regurgitation.
Inferior constrictor — lowest pharyngeal constrictor, with thyropharyngeus and cricopharyngeus parts.
Superior constrictor — pharyngeal muscle that aids velopharyngeal closure.
Middle constrictor — constrictor muscle arising from hyoid bone.
Pharyngeal plexus — motor supply from vagus nerve, sensory from glossopharyngeal.
Retropharyngeal space — potential space behind pharynx extending to mediastinum.
Esophageal constrictions — physiological narrowings: cricopharyngeal, aortic arch, left bronchus, diaphragm.
Cricopharyngeus — part of inferior constrictor, forms upper esophageal sphincter.
Thyropharyngeus — oblique part of inferior constrictor arising from thyroid cartilage.
Lead Question - 2012
Killian dehiscence is in ?
a) Superior constrictor
b) Inferior constrictor
c) Middle constrictor
d) None
Explanation (50 words): Killian’s dehiscence is a weak triangular gap between thyropharyngeus and cricopharyngeus fibers of the inferior constrictor muscle. It is the anatomical site where Zenker’s diverticulum develops, causing progressive dysphagia and aspiration risk. Answer: b) Inferior constrictor. This area is clinically important in esophageal surgery and ENT practice.
1. Zenker’s diverticulum arises from?
a) Between thyropharyngeus and cricopharyngeus
b) Between superior and middle constrictor
c) At cricoid cartilage
d) At hyoid bone
Explanation (50 words): Zenker’s diverticulum is a pulsion diverticulum occurring through Killian’s dehiscence, a weak area between thyropharyngeus and cricopharyngeus parts of the inferior constrictor. It leads to regurgitation of undigested food, chronic cough, and aspiration. Answer: a) Between thyropharyngeus and cricopharyngeus.
2. Cricopharyngeus muscle acts as?
a) Lower esophageal sphincter
b) Upper esophageal sphincter
c) Middle constrictor
d) Pharyngeal raphe
Explanation (50 words): Cricopharyngeus, the horizontal part of the inferior constrictor, functions as the upper esophageal sphincter. It relaxes during swallowing and contracts to prevent regurgitation of esophageal contents. Dysfunction causes dysphagia and predisposes to pharyngoesophageal diverticula. Answer: b) Upper esophageal sphincter.
3. Clinical feature of Zenker’s diverticulum?
a) Dysphagia
b) Hematemesis
c) Hemoptysis
d) Otalgia
Explanation (50 words): Zenker’s diverticulum presents with progressive dysphagia, regurgitation of undigested food, halitosis, nocturnal coughing, and aspiration pneumonia. It occurs through Killian’s dehiscence at the inferior constrictor. Answer: a) Dysphagia. It may mimic esophageal strictures clinically, but diagnosis is confirmed radiologically by barium swallow.
4. Nerve supply of inferior constrictor?
a) Glossopharyngeal
b) Vagus (via pharyngeal plexus & external/recurrent laryngeal nerves)
c) Hypoglossal
d) Accessory nerve
Explanation (50 words): Inferior constrictor is innervated by the vagus nerve via the pharyngeal plexus and additionally by external and recurrent laryngeal branches. This coordinated supply aids swallowing and sphincter function. Answer: b) Vagus (via pharyngeal plexus & external/recurrent laryngeal nerves).
5. Retropharyngeal abscess spreads into?
a) Mediastinum
b) Orbit
c) Mastoid air cells
d) Anterior triangle
Explanation (50 words): The retropharyngeal space extends from the base of skull to posterior mediastinum. Infections in this space, especially in children, can spread inferiorly and cause mediastinitis, a life-threatening condition. Answer: a) Mediastinum. Clinically, presents with fever, neck stiffness, dysphagia, and respiratory distress.
6. First esophageal constriction is at?
a) Cricopharyngeus
b) Aortic arch
c) Diaphragm
d) Left bronchus
Explanation (50 words): The esophagus has four constrictions: upper (cricopharyngeus at C6), aortic arch, left bronchus, and diaphragmatic hiatus. The first is at the cricopharyngeus, site of Killian’s dehiscence and common foreign body impaction. Answer: a) Cricopharyngeus.
7. Killian-Jamieson diverticulum arises from?
a) Lateral wall below cricopharyngeus
b) Posterior midline
c) Between middle and inferior constrictor
d) At aortic arch
Explanation (50 words): Killian-Jamieson diverticulum arises from the lateral wall of the cervical esophagus, just below the cricopharyngeus, unlike Zenker’s which is posterior. It is rare but can mimic Zenker’s clinically. Answer: a) Lateral wall below cricopharyngeus.
8. Best investigation for Zenker’s diverticulum?
a) Barium swallow
b) Endoscopy
c) CT neck
d) Ultrasound
Explanation (50 words): The diagnostic investigation of choice for Zenker’s diverticulum is barium swallow, which outlines the posterior pharyngoesophageal pouch. Endoscopy is avoided initially due to risk of perforation. Answer: a) Barium swallow.
9. Killian’s triangle is located?
a) Between thyropharyngeus & cricopharyngeus
b) Between superior & middle constrictor
c) Between middle & inferior constrictor
d) Between cricoid and trachea
Explanation (50 words): Killian’s triangle (dehiscence) is a weak posterior gap between oblique fibers of thyropharyngeus and horizontal fibers of cricopharyngeus, both parts of the inferior constrictor. Answer: a) Between thyropharyngeus & cricopharyngeus.
10. Common complication of untreated Zenker’s diverticulum?
a) Aspiration pneumonia
b) Pulmonary embolism
c) Otitis media
d) Carotid dissection
Explanation (50 words): Aspiration of regurgitated food from Zenker’s diverticulum leads to recurrent chest infections and aspiration pneumonia, the most common complication. Answer: a) Aspiration pneumonia. Chronic untreated cases may also cause malnutrition, halitosis, and rarely squamous carcinoma.
Chapter: Head & Neck
Topic: Pharynx
Subtopic: Retropharyngeal space & related anatomy
Keyword definitions
Retropharyngeal space — potential space between buccopharyngeal fascia and alar fascia; pathway for infection spread.
Buccopharyngeal fascia — fascia covering outer surface of pharyngeal constrictors.
Alar fascia — layer separating retropharyngeal space from danger space posteriorly.
Danger space — space between alar and prevertebral fascia; infection can spread to mediastinum.
C7 vertebra — landmark near transition of cervical esophagus.
Bifurcation of trachea — occurs around T4–T5 level, carina of trachea.
Esophageal constrictions — natural narrowings: cricopharyngeal, aortic arch, left bronchus, diaphragmatic.
Prevertebral fascia — deep cervical fascia covering vertebral column and muscles.
Pharyngeal abscess — pus collection in retropharyngeal space, potentially life-threatening airway obstruction.
Mediastinitis — severe infection spreading into mediastinum, often from danger space.
Lead Question - 2012
Lower limit of retropharyngeal space is at ?
a) C 7
b) Bifurcation of trachea
c) 4 th esophageal constriction
d) None
Explanation (50 words): The retropharyngeal space extends from the base of the skull down to the level of T2 vertebra (around bifurcation of trachea). Below this, it continues as the danger space reaching the diaphragm. Answer: b) Bifurcation of trachea. Infections here may descend into mediastinum rapidly.
1. Which fascia forms the posterior boundary of the retropharyngeal space?
a) Buccopharyngeal fascia
b) Alar fascia
c) Prevertebral fascia
d) Investing fascia
Explanation: The retropharyngeal space is bounded anteriorly by buccopharyngeal fascia and posteriorly by alar fascia. Beyond alar fascia lies danger space and prevertebral fascia. Answer: b) Alar fascia. Clinical infections can breach this and spread rapidly to mediastinum causing acute mediastinitis, a life-threatening complication in deep neck infections.
2. Retropharyngeal abscess commonly arises from?
a) Dental infections
b) Middle ear infection
c) Upper respiratory tract infection
d) Cervical spine tuberculosis
Explanation: Retropharyngeal abscess usually follows upper respiratory tract infections in children due to suppuration of retropharyngeal lymph nodes, which regress after age 5. In adults, trauma or tuberculosis of cervical spine can cause it. Answer: c) Upper respiratory tract infection. Presents with fever, dysphagia, stridor, and bulging posterior pharyngeal wall.
3. Clinical danger of retropharyngeal space infection is?
a) Spread to mediastinum
b) Spread to parotid
c) Spread to orbit
d) Spread to mastoid
Explanation: Retropharyngeal infections may pass through alar fascia into danger space, which extends from skull base to diaphragm. This leads to mediastinitis with high mortality. Answer: a) Spread to mediastinum. Early recognition and surgical drainage are essential to prevent systemic sepsis and airway compromise in these cases.
4. The retropharyngeal space is directly anterior to:
a) Larynx
b) Trachea
c) Prevertebral fascia
d) Carotid sheath
Explanation: Retropharyngeal space lies between buccopharyngeal fascia covering pharynx and alar fascia, anterior to prevertebral fascia. Answer: c) Prevertebral fascia. Infections here may erode vertebral bodies or spread posteriorly to spinal epidural space. Imaging (CT/MRI) shows widening of this space in suspected deep neck infections in clinical practice.
5. In children, retropharyngeal abscess most often occurs due to suppuration of?
a) Pharyngeal tonsil
b) Retropharyngeal lymph nodes
c) Jugulodigastric nodes
d) Submandibular nodes
Explanation: Retropharyngeal lymph nodes drain nasopharynx, adenoids, posterior pharynx and are present until age 4–5. Infection of these nodes causes retropharyngeal abscess in children. Answer: b) Retropharyngeal lymph nodes. Presents with fever, torticollis, stridor. After age 5, such abscesses become uncommon because lymph nodes regress significantly.
6. Radiological sign of retropharyngeal abscess on lateral neck X-ray?
a) Widened prevertebral space
b) Air-fluid level
c) Straightened cervical lordosis
d) Calcification
Explanation: A widened prevertebral soft tissue shadow (>7 mm at C2 or >14 mm at C6 in children) suggests retropharyngeal abscess. Answer: a) Widened prevertebral space. Additional CT confirms pus collection. Clinically, respiratory distress with neck swelling mandates urgent ENT referral and surgical drainage along with antibiotics.
7. Which clinical symptom strongly suggests retropharyngeal abscess in a child?
a) Persistent cough
b) Trismus
c) Neck stiffness with noisy breathing
d) Ear discharge
Explanation: Child with fever, sore throat, neck stiffness, drooling, and inspiratory stridor strongly suggests retropharyngeal abscess. Noisy breathing results from airway narrowing. Answer: c) Neck stiffness with noisy breathing. Immediate airway assessment and imaging are life-saving. Delay can result in aspiration, septicemia, and airway collapse in severe cases.
8. Which deep neck space communicates with retropharyngeal space?
a) Parapharyngeal space
b) Carotid sheath
c) Submandibular space
d) Sublingual space
Explanation: Retropharyngeal space communicates laterally with parapharyngeal space, allowing infection spread between them. Answer: a) Parapharyngeal space. Clinically, this explains rapid progression of neck abscesses and potential involvement of carotid sheath structures when infections cross compartments. Imaging studies help map spread before surgical drainage intervention in hospital practice.
9. Which cervical level marks transition of retropharyngeal space into danger space?
a) C2
b) C6
c) T2
d) T4
Explanation: Retropharyngeal space ends at T2 level. Below this, it continues as danger space which extends to diaphragm. Answer: c) T2. This anatomical division is critical, since infections crossing this level rapidly descend into mediastinum, requiring aggressive management. Surgeons note this level during cervical deep space drainage procedures carefully.
10. Which structure lies anterior to retropharyngeal space?
a) Buccopharyngeal fascia and pharynx
b) Carotid sheath
c) Laryngeal cartilages
d) Trachea
Explanation: The retropharyngeal space is posterior to the pharynx, which is covered externally by buccopharyngeal fascia. Thus anterior boundary is formed by pharyngeal wall with buccopharyngeal fascia. Answer: a) Buccopharyngeal fascia and pharynx. Infection presents as posterior pharyngeal wall bulge clinically, visible on oral examination.
Chapter: Head & Neck
Topic: Pharynx
Subtopic: Passavant's ridge & palatal/pharyngeal muscles
Keyword definitions
Passavant's ridge — transient mucosal bulge on posterior pharyngeal wall during velopharyngeal closure.
Palatoglossus — muscle forming anterior faucial pillar; elevates posterior tongue and narrows oropharyngeal isthmus.
Superior constrictor — upper pharyngeal constrictor muscle assisting in pharyngeal peristalsis and velopharyngeal seal.
Salpingopharyngeus — elevates lateral pharyngeal wall and helps open Eustachian tube.
Palatopharyngeus — forms posterior pillar; elevates pharynx and helps close oropharyngeal isthmus.
Palatal aponeurosis — fibrous central sheet of soft palate formed by tensor veli palatini tendon.
Tensor veli palatini — tenses soft palate and opens auditory tube; innervated by V3.
Levator veli palatini — elevates soft palate; important for velopharyngeal closure.
Pharyngeal raphe — midline fibrous seam where constrictor muscles attach.
Pharyngeal plexus — nerve network providing motor (CN X) and sensory (CN IX) supply.
Lead Question - 2012
Passavants ridge is formed by ?
a) Palatoglossus
b) Superior constrictor
c) Salpingopharyngeus
d) Palatopharyngeus
Explanation (50 words): Passavant's ridge is a transient mucosal bulge on the posterior pharyngeal wall during swallowing and speech. It results primarily from contraction of the superior pharyngeal constrictor muscle, aiding velopharyngeal closure. Answer: b) Superior constrictor. This prevents nasal regurgitation and assists speech resonance during swallowing and phonation.
1. Which muscle forms the palatopharyngeal arch (posterior pillar)?
a) Palatoglossus
b) Palatopharyngeus
c) Levator veli palatini
d) Tensor veli palatini
Explanation: The palatopharyngeal arch is formed by palatopharyngeus covered with mucosa. It elevates the laryngopharynx and closes the oropharyngeal isthmus. Answer: b) Palatopharyngeus.
2. The motor supply of the superior pharyngeal constrictor is by?
a) Pharyngeal branch of vagus (pharyngeal plexus)
b) Glossopharyngeal nerve
c) Hypoglossal nerve
d) Facial nerve
Explanation: Superior constrictor receives motor innervation via the pharyngeal plexus from vagus (CN X). Glossopharyngeal provides sensory. Answer: a) Pharyngeal branch of vagus.
3. Which muscle elevates and widens the pharynx during swallowing?
a) Stylopharyngeus
b) Palatoglossus
c) Superior constrictor
d) Salpingopharyngeus
Explanation: Stylopharyngeus (supplied by CN IX) elevates and widens pharynx, aiding bolus passage. Answer: a) Stylopharyngeus.
4. Which muscle tenses the soft palate?
a) Tensor veli palatini
b) Levator veli palatini
c) Palatoglossus
d) Palatopharyngeus
Explanation: Tensor veli palatini tenses the soft palate and opens auditory tube, innervated by V3. Answer: a) Tensor veli palatini.
5. Which muscle primarily opens the pharyngeal orifice of the auditory tube?
a) Tensor veli palatini
b) Levator veli palatini
c) Salpingopharyngeus
d) Palatoglossus
Explanation: Tensor veli palatini actively opens the auditory tube during swallowing, equilibrating pressure. Answer: a) Tensor veli palatini.
6. Primary action of palatoglossus is to:
a) Elevate posterior tongue
b) Depress larynx
c) Tense soft palate
d) Open auditory tube
Explanation: Palatoglossus elevates posterior tongue and narrows oropharyngeal isthmus, supplied by pharyngeal plexus (CN X). Answer: a) Elevate posterior tongue.
7. The pharyngeal raphe is best described as:
a) Median fibrous seam where constrictors attach
b) Palatal aponeurosis continuation
c) Pharyngobasilar fascia thickening
d) Attachment of levator veli palatini
Explanation: The pharyngeal raphe is a midline fibrous seam where constrictors insert posteriorly. Answer: a) Median fibrous seam.
8. Which muscle originates from the palatal aponeurosis?
a) Tensor veli palatini
b) Levator veli palatini
c) Palatoglossus
d) Stylopharyngeus
Explanation: Palatoglossus originates from the palatal aponeurosis and descends to the tongue. Answer: c) Palatoglossus.
9. Palatoglossus receives its motor supply from:
a) Hypoglossal nerve
b) Vagus nerve via pharyngeal plexus
c) Glossopharyngeal nerve
d) Trigeminal nerve
Explanation: Unlike other tongue muscles, palatoglossus is supplied by vagus (CN X) via pharyngeal plexus. Answer: b) Vagus nerve.
10. Salpingopharyngeus primarily:
a) Elevates lateral pharyngeal wall and opens auditory tube
b) Depresses hyoid bone
c) Tenses the soft palate
d) Closes nasal cavity
Explanation: Salpingopharyngeus elevates lateral wall and assists in opening auditory tube, helping pressure balance. Answer: a) Elevates lateral pharyngeal wall and opens auditory tube.
Chapter: Head & Neck Anatomy | Topic: Nasal Cavity & Paranasal Sinuses | Subtopic: Lymphatic Drainage of Nose
Keyword Definitions
Lateral wall of nose: Side wall bearing turbinates and meati.
Submandibular nodes: Level Ib nodes draining anterior nasal cavity/vestibule.
Retropharyngeal nodes: Nodes behind pharynx draining posterior nasal cavity.
Deep cervical nodes: Jugular chain receiving lymph from head & neck.
Anterior nasal cavity: Area near vestibule, drains anteroinferiorly.
Posterior nasal cavity: Choanal region, drains to retropharyngeal/deep cervical.
Kiesselbach’s area: Vascular plexus on anterior septum.
Waldeyer’s ring: Lymphoid ring around naso-oropharynx.
Level II nodes: Upper jugular nodes under sternomastoid.
Sentinel node: First draining node from a primary site.
Lead Question – 2012
Lymphatic drainage of lateral wall of nose
a) Submandibular nodes
b) Retropharyngeal nodes
c) Deep cervical nodes
d) All of the above
Explanation (Answer: d)
Anterior parts of the lateral nasal wall drain to submandibular nodes, while posterior parts drain to retropharyngeal nodes; both channels ultimately reach the deep cervical chain. Hence all listed groups participate in drainage depending on subsite, making “All of the above” the correct option for comprehensive lateral wall drainage.
2) Posterior lateral nasal wall carcinoma most classically first drains to:
a) Submental nodes
b) Retropharyngeal nodes
c) Supraclavicular nodes
d) Occipital nodes
Explanation (Answer: b)
Posterior nasal cavity, including posterior lateral wall near choanae, frequently drains to retropharyngeal nodes before reaching the upper deep cervical chain. Submental drainage is for lower lip/anterior floor of mouth, supraclavicular for lower neck catchment, and occipital for posterior scalp.
3) Which statement about lymphatics of the nasal cavity is MOST accurate?
a) All nasal subsites drain only to submental nodes
b) Anterior subsites favor submandibular drainage
c) Posterior subsites drain only to Level IV
d) Lymph bypasses deep cervical nodes
Explanation (Answer: b)
Anterior nasal cavity, including vestibule and inferior meatus region, commonly drains to submandibular nodes. Posterior subsites use retropharyngeal and then deep cervical chains. Deep cervical involvement is common; exclusive Level IV or submental drainage is incorrect. Thus, anterior-to-submandibular is the most accurate.
4) A clinician suspects retropharyngeal nodal disease in a posterior choanal mass. The next echelon typically involved is:
a) Level II deep cervical nodes
b) Pretracheal nodes
c) Submental nodes
d) Parotid nodes
Explanation (Answer: a)
Posterior nasal cavity drains to retropharyngeal nodes, then commonly to upper deep cervical (Level II) nodes along the internal jugular chain. Pretracheal nodes are for lower airway/thyroid regions, submental for lower lip/anterior floor, and parotid for lateral face/scalp anterior to ear.
5) In epistaxis originating from anterior lateral wall near the vestibule, which nodal basin most likely shows reactive enlargement?
a) Level II nodes
b) Submandibular nodes
c) Level IV nodes
d) Retropharyngeal nodes
Explanation (Answer: b)
Inflammation or infection in anterior lateral nasal wall/vestibule commonly drains to submandibular nodes, which may become reactive. Retropharyngeal nodes are more posterior. Level II may be secondary, and Level IV is a lower jugular station not typically first involved in anterior nasal pathology.
6) A 42-year-old with posterior lateral nasal wall tumor has occult nodal spread. Which imaging-detected node best supports expected drainage?
a) Retropharyngeal node behind pharyngeal wall
b) Submental node below chin
c) Level V posterior triangle node
d) Supraclavicular node
Explanation (Answer: a)
Posterior lateral nasal wall drains to retropharyngeal nodes first. Submental, Level V, and supraclavicular nodes are not typical initial stations for nasal cavity primaries. Finding a retropharyngeal metastasis aligns with the anatomical lymphatic pathways of posterior nasal subsites.
7) Which subsite pairing is CORRECT regarding primary lymphatic drainage?
a) Anterior lateral wall → Submandibular nodes
b) Posterior lateral wall → Submental nodes
c) Anterior lateral wall → Retropharyngeal nodes
d) Posterior lateral wall → Occipital nodes
Explanation (Answer: a)
Anterior lateral wall and vestibule drain to submandibular nodes. Posterior lateral wall prefers retropharyngeal and then deep cervical nodes. Submental and occipital nodal groups are not the primary drainage for these nasal subsites, making option a the accurate pairing.
8) A child with posterior nasal infection develops torticollis and fever. Which node is classically implicated?
a) Retropharyngeal node (suppurative adenitis)
b) Submandibular node
c) Preauricular node
d) Level IV node
Explanation (Answer: a)
Retropharyngeal nodes drain the posterior nasal cavity and nasopharynx; suppurative adenitis can present with neck stiffness/torticollis and fever in children. Submandibular nodes typically reflect anterior oral/nasal infections. Preauricular and Level IV nodes are not the classical primary sites for posterior nasal infections.
9) For surgical planning in anterior lateral nasal wall cancer, which selective neck dissection levels are MOST relevant initially?
a) Levels I–III (emphasis on Level Ib)
b) Levels IV–V only
c) Level VI central compartment
d) Parotidectomy with Level V only
Explanation (Answer: a)
Anterior lateral wall tends to involve submandibular (Level Ib) first, with potential spread to Levels II–III. Levels IV–V are lower/posterior and less commonly initial. Level VI pertains to central compartment organs. Parotidectomy targets preauricular/parotid nodes, not primary anterior nasal drainage.
10) Which statement about deep cervical nodes in nasal drainage is TRUE?
a) They are never involved in nasal malignancies
b) They receive efferents from both submandibular and retropharyngeal nodes
c) They drain only the scalp and ear
d) They are equivalent to supraclavicular nodes exclusively
Explanation (Answer: b)
Deep cervical nodes (jugular chain) are final common pathways for multiple head-neck subsites. They receive efferents from submandibular and retropharyngeal nodes and are often involved in nasal malignancies. They do not exclusively serve scalp/ear, nor are they limited to supraclavicular territory.
11) A 60-year-old with recurrent posterior epistaxis and fullness behind the soft palate shows a node of Rouvière on MRI. Primary site drainage suggests:
a) Anterior vestibular lesion
b) Posterior lateral nasal wall involvement
c) External nasal skin carcinoma
d) Floor of mouth lesion
Explanation (Answer: b)
Node of Rouvière is the lateral retropharyngeal node, commonly receiving lymph from nasopharynx and posterior nasal cavity, including posterior lateral wall. Anterior vestibule and external nasal skin drain anteriorly to submandibular/preauricular nodes, while floor of mouth drains to submental/submandibular, not retropharyngeal.
Chapter: Head and Neck Anatomy
Topic: Pterygopalatine Fossa
Subtopic: Contents and Relations
Keyword Definitions
Pterygopalatine fossa: A small inverted pyramidal space deep in the face, between maxilla and sphenoid bone, transmitting nerves and vessels.
Pterygopalatine ganglion: A parasympathetic ganglion in the fossa associated with lacrimal and nasal secretions.
Maxillary artery: One of the terminal branches of the external carotid artery, supplying deep facial structures.
Maxillary nerve: Second division of the trigeminal nerve (CN V2), carrying sensory fibers from midface.
Greater petrosal nerve: Branch of facial nerve carrying parasympathetic fibers to the pterygopalatine ganglion.
Lead Question – 2012
All of the following are structures associated with pterygopalatine fossa, EXCEPT:
a) Pterygopalatine ganglion
b) Mid third of maxillary artery
c) Maxillary nerve
d) Greater petrosal nerve
Explanation: The pterygopalatine fossa contains the pterygopalatine ganglion, terminal part of maxillary artery, and maxillary nerve. The mid-third of the maxillary artery is not directly associated, as only its terminal part enters the fossa. Correct answer: b) Mid third of maxillary artery.
Guessed Questions for NEET PG
Q1. Which foramen communicates the pterygopalatine fossa with the orbit?
a) Foramen rotundum
b) Inferior orbital fissure
c) Sphenopalatine foramen
d) Pterygoid canal
Explanation: The inferior orbital fissure communicates the pterygopalatine fossa with the orbit, allowing passage of infraorbital vessels and nerves. This is clinically relevant in orbital infections. Correct answer: b) Inferior orbital fissure.
Q2. Through which foramen does the maxillary nerve enter the pterygopalatine fossa?
a) Foramen ovale
b) Foramen rotundum
c) Pterygoid canal
d) Greater palatine foramen
Explanation: The maxillary nerve enters the pterygopalatine fossa via the foramen rotundum. This pathway is important in maxillary nerve block anesthesia. Correct answer: b) Foramen rotundum.
Q3. Which artery passes through the sphenopalatine foramen into the nasal cavity?
a) Facial artery
b) Sphenopalatine artery
c) Infraorbital artery
d) Middle meningeal artery
Explanation: The sphenopalatine artery, a terminal branch of the maxillary artery, passes through the sphenopalatine foramen to supply the nasal cavity. Correct answer: b) Sphenopalatine artery.
Q4. The greater palatine nerve arises from which structure?
a) Pterygopalatine ganglion
b) Otic ganglion
c) Geniculate ganglion
d) Trigeminal ganglion
Explanation: The greater palatine nerve arises from the pterygopalatine ganglion and supplies the hard palate mucosa. Correct answer: a) Pterygopalatine ganglion.
Q5. Which clinical block involves deposition of anesthetic into the pterygopalatine fossa?
a) Inferior alveolar nerve block
b) Maxillary nerve block
c) Lingual nerve block
d) Mandibular nerve block
Explanation: The maxillary nerve block is performed by injecting anesthetic into the pterygopalatine fossa, providing anesthesia to the midface. Correct answer: b) Maxillary nerve block.
Q6. The Vidian nerve is formed by union of greater petrosal nerve and which other nerve?
a) Chorda tympani
b) Deep petrosal nerve
c) Lesser petrosal nerve
d) Auriculotemporal nerve
Explanation: The Vidian nerve (nerve of pterygoid canal) is formed by the union of greater petrosal and deep petrosal nerves. Correct answer: b) Deep petrosal nerve.
Q7. The pterygopalatine ganglion is functionally associated with which cranial nerve?
a) Trigeminal nerve
b) Facial nerve
c) Glossopharyngeal nerve
d) Vagus nerve
Explanation: Though anatomically linked to the maxillary nerve, the pterygopalatine ganglion is functionally associated with the facial nerve via parasympathetic fibers. Correct answer: b) Facial nerve.
Q8. Which structure passes through the pterygoid canal to reach the pterygopalatine fossa?
a) Greater palatine nerve
b) Nerve of pterygoid canal
c) Chorda tympani
d) Auriculotemporal nerve
Explanation: The nerve of pterygoid canal (Vidian nerve) passes through the pterygoid canal to enter the pterygopalatine fossa. Correct answer: b) Nerve of pterygoid canal.
Q9. A tumor in the pterygopalatine fossa would most likely compress which of the following, leading to facial pain?
a) Optic nerve
b) Maxillary nerve
c) Hypoglossal nerve
d) Vestibulocochlear nerve
Explanation: A lesion in the pterygopalatine fossa can compress the maxillary nerve, resulting in trigeminal neuralgia affecting the midface. Correct answer: b) Maxillary nerve.
Q10. Which venous connection makes the pterygopalatine fossa a potential route for spread of infection to the cavernous sinus?
a) Retromandibular vein
b) Pterygoid venous plexus
c) Facial vein
d) Superior ophthalmic vein
Explanation: The pterygoid venous plexus communicates with the cavernous sinus and connects to the pterygopalatine fossa, serving as a dangerous route for infection spread. Correct answer: b) Pterygoid venous plexus.
Chapter: Lower Limb Anatomy
Topic: Femur
Sub-topic: Blood Supply of Femur
Nutrient artery: A major artery entering the shaft of a long bone to supply the inner two-thirds of the cortex and marrow.
Profunda femoris artery: Deep artery of the thigh; gives branches to thigh muscles and nutrient artery to femur.
Femoral artery: Main artery supplying the lower limb, continuation of external iliac artery.
Popliteal artery: Continuation of femoral artery behind the knee, supplies knee joint and leg muscles.
Middle circumflex femoral artery: Branch of profunda femoris artery supplying the head and neck of femur.
Medial circumflex femoral artery: Branch of profunda femoris artery supplying femoral head and neck.
Femoral triangle: Anatomical space in upper thigh containing femoral nerve, artery, vein, and lymphatics.
Osteomyelitis: Infection of bone, commonly via bloodstream.
1. The nutrient artery to the femur is? 2012 Lead Question
a) Profunda femoris artery
b) Femoral artery
c) Popliteal artery
d) Middle circumflex femoral artery
Correct answer: a) Profunda femoris artery. The nutrient artery to the femur mainly arises from the profunda femoris artery. It enters the nutrient foramen on the femur's posterior surface and supplies the inner two-thirds of the cortical bone and marrow, essential for bone nourishment and repair.
2. Which branch of the profunda femoris artery supplies the head and neck of the femur?
a) Lateral circumflex femoral artery
b) Medial circumflex femoral artery
c) Nutrient artery
d) Descending genicular artery
Correct answer: b) Medial circumflex femoral artery. This artery arises from the profunda femoris and mainly supplies the head and neck of the femur, which is critical for femoral head viability, especially after fractures.
3. Injury to which artery is most likely to cause avascular necrosis of the femoral head in femoral neck fractures?
a) Lateral circumflex femoral artery
b) Medial circumflex femoral artery
c) Profunda femoris artery
d) Popliteal artery
Correct answer: b) Medial circumflex femoral artery. This artery is the primary blood supply to the femoral head. Damage during femoral neck fractures can cause avascular necrosis leading to joint dysfunction.
4. The profunda femoris artery typically arises from the femoral artery at which location?
a) At the inguinal ligament
b) Just below the inguinal ligament
c) At the adductor hiatus
d) At the popliteal fossa
Correct answer: b) Just below the inguinal ligament. The profunda femoris artery arises about 3-4 cm below the inguinal ligament and passes posteriorly to supply deep thigh structures.
5. Which artery provides important collateral circulation around the thigh in cases of femoral artery occlusion?
a) Profunda femoris artery
b) Popliteal artery
c) Middle circumflex femoral artery
d) Dorsalis pedis artery
Correct answer: a) Profunda femoris artery. Its perforating branches form collateral pathways that maintain blood flow when the femoral artery is occluded.
6. Damage to the nutrient artery during femoral shaft fracture may result in:
a) Muscle ischemia
b) Delayed bone healing
c) Joint dislocation
d) Nerve injury
Correct answer: b) Delayed bone healing. The nutrient artery supplies the inner cortical bone and marrow; damage reduces blood supply, impeding bone repair and potentially causing nonunion.
7. In the femoral triangle, the femoral artery lies _______ to the femoral vein.
a) Medial
b) Lateral
c) Posterior
d) Anterior
Correct answer: b) Lateral. The femoral artery is lateral to the femoral vein in the femoral triangle, an important consideration during vascular access.
8. The nutrient artery primarily supplies which part of the femur?
a) Outer cortex
b) Inner two-thirds of cortex and marrow
c) Joint capsule
d) Surrounding muscles
Correct answer: b) Inner two-thirds of cortex and marrow. The nutrient artery enters via the nutrient foramen and nourishes the bone marrow and inner cortex essential for bone vitality.
9. Hematogenous osteomyelitis of the femur commonly involves spread via which artery?
a) Popliteal artery
b) Medial circumflex femoral artery
c) Nutrient artery
d) Lateral circumflex femoral artery
Correct answer: c) Nutrient artery. Bacteria enter via the nutrient artery to infect the marrow cavity, causing osteomyelitis.
10. During femoral artery cannulation, the puncture site is ideally located:
a) 1 cm below the inguinal ligament
b) 2-3 cm below the inguinal ligament
c) At the midpoint of femoral triangle
d) At the adductor hiatus
Correct answer: a) 1 cm below the inguinal ligament. This site allows safe access to the femoral artery while avoiding the femoral vein and minimizing complications.
Chapter: Lower Limb Anatomy
Topic: Femoral Triangle & Related Structures
Sub-topic: Structures Passing Behind Inguinal Ligament
Keyword Definitions:
Inguinal Ligament: A fibrous band from the anterior superior iliac spine to the pubic tubercle, forming the base of the inguinal canal.
Femoral Vein: A major deep vein of the thigh, located medial to the femoral artery in the femoral triangle.
Psoas Major: A large muscle of the posterior abdominal wall, important for hip flexion.
Genitofemoral Nerve: A mixed nerve of lumbar origin, with genital and femoral branches.
Femoral Triangle: An anatomical space in the upper thigh containing femoral nerve, artery, vein, and lymphatics.
Retroinguinal Space: Area beneath the inguinal ligament containing vessels, nerves, and muscles passing into the thigh.
Iliacus Muscle: A hip flexor muscle passing beneath the inguinal ligament along with the psoas major.
Lymphatics of Lower Limb: Vessels and nodes draining lower extremities, passing through femoral canal.
Lead Question – NEET PG 2012:
Which structure(s) passes behind the inguinal ligament?
a) Femoral branch of genitofemoral nerve
b) Femoral vein
c) Psoas major
d) All
Explanation: The inguinal ligament forms the lower boundary of the abdomen. Structures passing beneath it include muscles (psoas major, iliacus), vessels (femoral artery, femoral vein), nerves (femoral nerve, femoral branch of genitofemoral nerve, lateral femoral cutaneous nerve), and lymphatics. Since all listed options pass behind it, the correct answer is d) All.
Q2. Which nerve passes lateral to the femoral artery beneath the inguinal ligament?
a) Obturator nerve
b) Femoral nerve
c) Sciatic nerve
d) Pudendal nerve
Explanation: The femoral nerve lies lateral to the femoral artery beneath the inguinal ligament within the muscular compartment of the retroinguinal space. This anatomical relationship is key in femoral catheterization and nerve blocks. The correct answer is b) Femoral nerve.
Q3. The femoral vein lies in relation to the femoral artery as:
a) Lateral
b) Medial
c) Posterior
d) Anterior
Explanation: In the femoral triangle, the femoral vein lies medial to the femoral artery. Clinically, this helps in identifying vascular structures during catheterization. Remember the order from lateral to medial: Nerve – Artery – Vein – Lymphatics (NAVL). The correct answer is b) Medial.
Q4. Which structure occupies the muscular lacuna beneath the inguinal ligament?
a) Femoral artery
b) Femoral vein
c) Psoas major
d) Femoral canal
Explanation: The muscular lacuna is the lateral compartment beneath the inguinal ligament, containing the psoas major, iliacus, and femoral nerve. Vessels pass through the vascular lacuna. The correct answer is c) Psoas major.
Q5. In femoral hernia, the hernial sac passes through:
a) Muscular lacuna
b) Femoral canal
c) Obturator canal
d) Inguinal canal
Explanation: Femoral hernia occurs when abdominal contents protrude through the femoral canal, which lies medial to the femoral vein beneath the inguinal ligament. It is more common in females due to a wider pelvis. The correct answer is b) Femoral canal.
Q6. The lateral femoral cutaneous nerve passes beneath the inguinal ligament:
a) Medial to ASIS
b) Lateral to ASIS
c) Through femoral canal
d) Through obturator canal
Explanation: The lateral femoral cutaneous nerve passes beneath the inguinal ligament about 1 cm medial to the anterior superior iliac spine (ASIS) to supply sensation to the lateral thigh. Compression here may cause meralgia paresthetica. The correct answer is a) Medial to ASIS.
Q7. Which vessel lies directly medial to the femoral artery beneath the inguinal ligament?
a) Femoral vein
b) Femoral nerve
c) Deep femoral artery
d) Great saphenous vein
Explanation: The femoral vein lies immediately medial to the femoral artery in the femoral sheath beneath the inguinal ligament. This arrangement is vital for femoral venous access. The correct answer is a) Femoral vein.
Q8. In retroperitoneal hemorrhage, which muscle passing under the inguinal ligament may become tender?
a) Rectus femoris
b) Psoas major
c) Sartorius
d) Tensor fasciae latae
Explanation: Psoas major lies retroperitoneally and passes under the inguinal ligament into the thigh. Retroperitoneal bleeding can cause irritation and tenderness in the psoas muscle, leading to difficulty in hip flexion. The correct answer is b) Psoas major.
Q9. Which structure does NOT pass beneath the inguinal ligament?
a) Femoral artery
b) Femoral vein
c) Obturator nerve
d) Psoas major
Explanation: The obturator nerve passes through the obturator canal in the pelvis and does not cross beneath the inguinal ligament. The others listed are retroinguinal structures. The correct answer is c) Obturator nerve.
Q10. Which artery lies lateral to the femoral vein beneath the inguinal ligament?
a) Femoral artery
b) Deep femoral artery
c) Popliteal artery
d) Superficial epigastric artery
Explanation: The femoral artery lies lateral to the femoral vein beneath the inguinal ligament. This relationship is consistent in anatomical positioning and is clinically relevant for surgical approaches. The correct answer is a) Femoral artery.
Chapter: Anatomy / Topic: Upper Limb / Subtopic: 1st Carpometacarpal (CMC) Joint — Thumb
1st carpometacarpal (CMC) joint — the articulation between the trapezium (carpal bone) and the base of the first metacarpal; crucial for thumb opposition and wide range of motion.
Saddle joint — a type of synovial joint where opposing surfaces are reciprocally concavo-convex, allowing flexion/extension, abduction/adduction and limited rotation (circumduction and opposition).
Opposition — complex thumb movement combining flexion, abduction and medial rotation at the CMC and interphalangeal/metacarpophalangeal joints permitting the tip-to-tip pinch.
Basal joint osteoarthritis — degenerative arthritis of the 1st CMC commonly in postmenopausal women, producing pain at the base of thumb, reduced grip, positive grind test.
Stabilizing ligaments — include the anterior oblique (beak) ligament, ulnar collateral ligament, dorsal radial ligament and intermetacarpal ligament; their integrity is essential for joint stability.
Lead Question - 2012: 1st carpometacarpal joint is?
a) Pivot
b) Hinge
c) Ball and Socket
d) Saddle
Explanation (answer included): The 1st carpometacarpal joint of the thumb is classically described as a saddle-type synovial joint (sellar joint). The opposing surfaces of the trapezium and the base of the first metacarpal are reciprocally convex and concave in perpendicular planes, allowing flexion-extension, abduction-adduction, opposition and limited rotation. This unique saddle configuration enables the thumb's wide range of motion; therefore the correct answer is (d) Saddle. Biomechanically this joint is essential for precision grip and pinch, and its degeneration leads to basal joint osteoarthritis.
Q2. Which ligament is considered the most important stabilizer against dorsal subluxation of the 1st CMC joint?
a) Radial collateral ligament
b) Anterior oblique (beak) ligament
c) Ulnar collateral ligament of the thumb MCP
d) Transverse carpal ligament
Explanation (answer included): The anterior oblique ligament (also called the beak ligament) originates from the palmar tubercle of the trapezium and inserts on the palmar base of the first metacarpal; it restrains dorsal subluxation and excessive radial translation of the metacarpal. Laxity or rupture of this ligament is strongly implicated in basal joint instability and osteoarthritis. Therefore the most important stabilizer against dorsal subluxation at the 1st CMC is the (b) Anterior oblique (beak) ligament. Surgical reconstructions often recreate its function.
Q3. A 60-year-old woman complains of pain at the base of her thumb and difficulty pinching. Which clinical sign is most specific for 1st CMC osteoarthritis?
a) Tinel's sign
b) Finkelstein's test
c) Positive grind test at the CMC joint
d) Phalen's test
Explanation (answer included): The CMC grind test involves axial compression and rotation of the first metacarpal against the trapezium and reproduces pain and crepitus when osteoarthritis is present at the basal joint. It is a sensitive and relatively specific clinical maneuver for basal joint arthritis and helps distinguish it from de Quervain tenosynovitis (Finkelstein) or carpal tunnel signs (Tinel/Phalen). Thus the expected finding is a (c) Positive grind test at the CMC joint, correlating with symptoms of pain during pinch and reduced grip strength.
Q4. Which movements are permitted by the saddle joint configuration of the 1st CMC? (choose best)
a) Flexion–extension and pronation only
b) Abduction–adduction and supination only
c) Flexion–extension, abduction–adduction and limited rotation (opposition)
d) Pure hinge motion only
Explanation (answer included): The saddle (sellar) morphology of the 1st CMC allows movements in two planes — flexion–extension and abduction–adduction — and, importantly, permits limited axial rotation that enables opposition (a composite motion combining flexion, abduction and medial rotation). This multi-planar capability underlies precision grip and tip-to-tip pinch function. Therefore the best description is (c) Flexion–extension, abduction–adduction and limited rotation (opposition). Loss of any component reduces functional hand use and is targeted in reconstructive surgery.
Q5. The articular surfaces of the 1st CMC joint are lined by which type of cartilage?
a) Fibrocartilage only
b) Hyaline (articular) cartilage
c) Elastic cartilage
d) No cartilage — bone to bone
Explanation (answer included): The articular surfaces of the trapezium and the base of the first metacarpal are covered by hyaline (articular) cartilage typical of synovial joints. This cartilage allows smooth gliding movements and distributes load. Degeneration of this hyaline cartilage is central to osteoarthritis of the basal joint. Therefore the correct answer is (b) Hyaline (articular) cartilage. Histologically, loss of this cartilage leads to joint space narrowing and subchondral changes on imaging.
Q6. In advanced symptomatic 1st CMC osteoarthritis refractory to conservative care, which surgical procedure is commonly performed?
a) Trapeziectomy with ligament reconstruction and tendon interposition (LRTI)
b) Carpal tunnel release
c) Ulnar shortening osteotomy
d) External fixation of the wrist
Explanation (answer included): Trapeziectomy combined with ligament reconstruction and tendon interposition (LRTI) is a widely used definitive surgical treatment for advanced basal joint osteoarthritis of the thumb. The procedure removes the arthritic trapezium, reconstructs stabilizing ligaments (often using a slip of the flexor carpi radialis tendon) and places tendon tissue to maintain spacing and cushion the first metacarpal. This operation relieves pain and preserves thumb function. Therefore the correct option is (a) Trapeziectomy with ligament reconstruction and tendon interposition (LRTI).
Q7. The muscle primarily responsible for thumb opposition is:
a) Abductor pollicis longus
b) Flexor pollicis longus
c) Opponens pollicis (thenar muscle)
d) Adductor pollicis
Explanation (answer included): Opponens pollicis, one of the thenar muscles innervated by the recurrent branch of the median nerve, acts at the first metacarpal to produce medial rotation and flexion, enabling opposition of the thumb toward the fingers. While abductor pollicis longus and flexor pollicis longus assist specific movements, opposition as a complex composite movement is driven mainly by opponens pollicis. Therefore the correct answer is (c) Opponens pollicis (thenar muscle). Loss of opponens function markedly impairs fine prehension.
Q8. Radiographic signs of 1st CMC osteoarthritis include all EXCEPT:
a) Joint space narrowing and subchondral sclerosis
b) Osteophyte formation at the trapezio-metacarpal articulation
c) Periarticular erosions typical of rheumatoid arthritis only
d) Subluxation or collapse of the first metacarpal base
Explanation (answer included): Typical radiographic features of osteoarthritis at the 1st CMC joint are joint space narrowing, subchondral sclerosis, osteophyte formation and sometimes subluxation of the metacarpal base. Periarticular erosions are more characteristic of inflammatory arthritides such as rheumatoid arthritis rather than degenerative basal joint disease. Therefore the statement that is NOT a typical sign of primary CMC osteoarthritis is (c) Periarticular erosions typical of rheumatoid arthritis only. Radiographs guide staging and treatment decisions.
Q9. A fall on an outstretched hand with force through the thumb can cause which injury at the base of the first metacarpal?
a) Bennett fracture (intra-articular fracture dislocation)
b) Scaphoid waist fracture only
c) Distal radius Colles fracture exclusively
d) Ulnar styloid avulsion only
Explanation (answer included): A Bennett fracture is an oblique intra-articular fracture at the base of the first metacarpal that extends into the CMC joint and is commonly caused by axial loading of a partially flexed thumb during a fall or punch. It typically produces subluxation due to pull of abductor pollicis longus. Thus the injury most associated with force transmitted through the thumb base is the (a) Bennett fracture (intra-articular fracture dislocation). Prompt recognition and reduction are important to restore CMC congruity and function.
Q10. Conservative management of early symptomatic 1st CMC arthritis commonly includes all EXCEPT:
a) Thumb splintage and activity modification
b) Analgesics and NSAIDs
c) Corticosteroid injection into the CMC joint
d) Immediate trapeziectomy without trial of conservative care
Explanation (answer included): Early management of symptomatic basal joint OA emphasizes non-operative measures: thumb splints to rest the joint, analgesics/NSAIDs for pain control, occupational therapy and intra-articular corticosteroid injections for selected patients. Immediate trapeziectomy is a definitive surgical option reserved for refractory, advanced disease after conservative therapy has failed. Therefore the statement that is NOT part of conservative care is (d) Immediate trapeziectomy without trial of conservative care. A staged approach optimizes outcomes and avoids unnecessary surgery.
Q11. Neurovascular supply: the main sensory innervation of the palmar aspect of the thumb (including CMC region) is by which nerve?
a) Ulnar nerve
b) Radial nerve superficial branch
c) Median nerve (digital branches)
d) Posterior interosseous nerve
Explanation (answer included): The palmar digital sensation of the thumb, including the region around the first CMC joint and the palmar surface, is primarily supplied by the median nerve via its proper palmar digital branches. The radial superficial branch supplies radial dorsal aspects and the ulnar nerve supplies the ulnar digits. Posterior interosseous is motor to extensors. Thus the correct answer is (c) Median nerve (digital branches). Sensory testing helps differentiate causes of thumb pain and guides regional anesthesia for procedures.
End of set. Each explanation contains the correct choice and is ≥50 words. Use this HTML in Blogger's HTML editor — it preserves the light red lead question block, light yellow explanation blocks, and 16px dark black body text for SEO and readability.
Topic: Axilla and Brachial Region
Subtopic: Spaces of the Scapular Region
Keywords & Definitions:
Upper Triangular Space: Anatomical space bordered by teres minor, teres major, and long head of triceps; transmits circumflex scapular artery.
Quadrangular Space: Space transmitting axillary nerve and posterior circumflex humeral artery.
Profunda Brachii Artery: Main branch of brachial artery supplying posterior arm.
Axillary Nerve: Supplies deltoid and teres minor, passes through quadrangular space.
Circumflex Scapular Artery: Branch of subscapular artery passing through upper triangular space.
Anterior Circumflex Humeral Artery: Supplies humeral head, passes anterior to surgical neck.
Posterior Circumflex Humeral Artery: Supplies shoulder joint and deltoid, passes through quadrangular space.
Teres Minor: Rotator cuff muscle, external rotation of arm.
Teres Major: Adduction and medial rotation of arm.
Long Head of Triceps: Extends and adducts arm, forms boundary of scapular spaces.
Lead Question (NEET PG 2012):
Structure passes through upper triangular space:
a) Profunda brachii
b) Anterior circumflex humeral artery
c) Posterior circumflex humeral artery
d) Circumflex scapular artery
Explanation: The upper triangular space (also called medial triangular space) is bordered superiorly by teres minor, inferiorly by teres major, and laterally by the long head of triceps brachii. It transmits the circumflex scapular artery, a branch of the subscapular artery. Other arteries like posterior circumflex humeral and profunda brachii pass through different spaces, not this one. Answer: d) Circumflex scapular artery
Q2. Which structure passes through the quadrangular space?
a) Radial nerve
b) Axillary nerve and posterior circumflex humeral artery
c) Circumflex scapular artery
d) Profunda brachii artery
The quadrangular space contains the axillary nerve and posterior circumflex humeral artery. It is bordered by teres minor (superior), teres major (inferior), long head of triceps (medial), and surgical neck of humerus (lateral). Answer: b) Axillary nerve and posterior circumflex humeral artery
Q3. The lower triangular space transmits:
a) Profunda brachii artery and radial nerve
b) Axillary nerve
c) Circumflex scapular artery
d) Anterior circumflex humeral artery
The lower triangular space, bordered by teres major (superior), long head of triceps (medial), and shaft of humerus (lateral), transmits the profunda brachii artery and radial nerve into the radial groove. Answer: a) Profunda brachii artery and radial nerve
Q4. A fracture of the surgical neck of the humerus may injure which space's contents?
a) Upper triangular space
b) Quadrangular space
c) Lower triangular space
d) Deltopectoral groove
Surgical neck fractures endanger the contents of the quadrangular space — the axillary nerve and posterior circumflex humeral artery — leading to deltoid weakness and sensory loss over the regimental badge area. Answer: b) Quadrangular space
Q5. Which artery is a branch of the subscapular artery?
a) Posterior circumflex humeral artery
b) Circumflex scapular artery
c) Profunda brachii artery
d) Anterior circumflex humeral artery
The circumflex scapular artery is a branch of the subscapular artery, itself a branch of the third part of the axillary artery. It passes through the upper triangular space to supply scapular anastomosis. Answer: b) Circumflex scapular artery
Q6. In axillary artery ligation proximal to subscapular artery, blood supply to upper limb is maintained by:
a) Vertebral artery
b) Scapular anastomosis
c) Deep cervical artery
d) Internal thoracic artery
Scapular anastomosis involves circumflex scapular, suprascapular, and dorsal scapular arteries, maintaining upper limb perfusion when axillary artery is ligated proximal to subscapular artery. Answer: b) Scapular anastomosis
Q7. A penetrating injury in the medial border of scapula damaging circumflex scapular artery will affect blood flow mainly to:
a) Deltoid
b) Trapezius
c) Infraspinatus and teres minor region
d) Biceps brachii
The circumflex scapular artery participates in scapular anastomosis, supplying infraspinatus, teres minor, and surrounding areas. Injury compromises posterior scapular region perfusion. Answer: c) Infraspinatus and teres minor region
Q8. Which nerve is closely related to the circumflex scapular artery in the triangular space?
a) Radial nerve
b) Axillary nerve
c) No major nerve
d) Suprascapular nerve
The upper triangular space transmits only the circumflex scapular artery and has no major nerve passing through it. Answer: c) No major nerve
Q9. Which muscle forms the superior boundary of the upper triangular space?
a) Teres major
b) Teres minor
c) Subscapularis
d) Latissimus dorsi
Teres minor, a rotator cuff muscle, forms the superior boundary of the upper triangular space, assisting in external rotation of the humerus. Answer: b) Teres minor
Q10. In shoulder surgery, the upper triangular space is important because:
a) It contains axillary nerve
b) It contains circumflex scapular artery
c) It contains suprascapular artery
d) It contains profunda brachii artery
Surgeons must avoid damaging the circumflex scapular artery during posterior shoulder approaches. This artery passes through the upper triangular space and is vital for scapular anastomosis. Answer: b) It contains circumflex scapular artery
Keyword Definitions
Ulnar nerve: A major nerve of the upper limb supplying intrinsic hand muscles and sensation to the medial hand.
Deep branch of ulnar nerve: Motor branch supplying most intrinsic hand muscles except thenar group and lateral lumbricals.
Hypothenar eminence: The fleshy mound on the medial side of the palm formed by hypothenar muscles.
Froment's sign: A clinical sign of ulnar nerve palsy, tested by asking patient to hold paper between thumb and index finger.
Lumbrical muscles: Intrinsic muscles that flex MCP joints and extend IP joints of fingers.
Median nerve: Supplies thenar muscles, lateral lumbricals, and sensation to lateral palm.
Radial nerve: Supplies posterior arm and forearm muscles and sensation to dorsum of hand.
Intrinsic hand muscles: Small muscles located entirely within the hand, controlling fine movements.
Adductor pollicis: Muscle that adducts the thumb, innervated by deep branch of ulnar nerve.
Claw hand: Deformity due to ulnar nerve injury affecting lumbricals and interossei.
Lead Question – NEET PG 2012:
Following a deep cut overlying the hypothenar eminence, it is observed that the patient cannot hold a sheet of paper between the 2nd and 3rd digits. Which of the following nerves is most likely damaged?
Deep branch of ulnar nerve
Deep branch of the radial nerve
Superficial branch of ulnar nerve
Median nerve
Explanation: The inability to hold paper between the fingers indicates weakness of palmar interossei, which adduct the fingers, and are supplied by the deep branch of the ulnar nerve. A cut at the hypothenar region can damage this motor branch, sparing sensation. Radial and median nerves do not supply these muscles.
Q2. Which muscle is tested by Froment's sign?
Adductor pollicis
Flexor pollicis longus
Abductor pollicis brevis
First dorsal interosseous
Explanation: Froment's sign tests adductor pollicis, innervated by the deep branch of the ulnar nerve. In palsy, the thumb flexes at the IP joint via flexor pollicis longus (median nerve) as compensation. This sign differentiates ulnar nerve injury from other hand muscle pathologies.
Q3. A patient has numbness over the medial 1½ fingers and weakness in finger adduction. The lesion is most likely at:
Wrist
Elbow
Brachial plexus
Axilla
Explanation: Medial finger numbness with adduction weakness indicates ulnar nerve involvement. A lesion at the wrist affects both sensory and motor fibers of the hand while sparing forearm muscles. Higher lesions affect more proximal muscles as well.
Q4. Which part of the ulnar nerve carries sensation from the medial hand?
Superficial branch
Deep branch
Posterior branch
Lateral branch
Explanation: The superficial branch of the ulnar nerve is sensory, supplying the palmar aspect of the medial 1½ fingers. The deep branch is purely motor, controlling most intrinsic hand muscles. Understanding this distinction is crucial in diagnosing nerve lesions.
Q5. Injury to the deep branch of the ulnar nerve causes paralysis of all except:
Adductor pollicis
Palmar interossei
Thenar muscles
Dorsal interossei
Explanation: The thenar muscles are supplied primarily by the median nerve, except adductor pollicis which is ulnar-innervated. Thus, in deep branch ulnar injury, thenar muscles remain functional except for adductor pollicis.
Q6. Which deformity occurs in distal ulnar nerve injury?
Wrist drop
Claw hand
Ape thumb
Benediction sign
Explanation: Distal ulnar nerve injury causes claw hand due to paralysis of lumbricals to ring and little fingers, leading to MCP hyperextension and IP joint flexion. Wrist drop is radial nerve injury, ape thumb is median nerve injury.
Q7. Which test best evaluates palmar interossei function?
Finger abduction
Paper-holding test
Thumb opposition
Wrist flexion
Explanation: The paper-holding test assesses palmar interossei, which adduct fingers. In ulnar nerve palsy, the patient cannot maintain grip on the paper due to muscle weakness.
Q8. Damage to which nerve branch causes isolated motor loss without sensory deficit in the hand?
Deep branch of ulnar nerve
Median nerve at wrist
Superficial branch of ulnar nerve
Posterior interosseous nerve
Explanation: The deep branch of the ulnar nerve is purely motor, supplying intrinsic hand muscles. Lesions here result in motor loss without sensory impairment, unlike superficial branch injury.
Q9. In a low ulnar nerve injury, which lumbricals are affected?
First and second
Third and fourth
All four
None
Explanation: The third and fourth lumbricals, supplied by the ulnar nerve, are paralyzed in low ulnar injuries. First and second lumbricals are median nerve-innervated and remain functional.
Q10. Which muscle group is spared in an isolated deep branch ulnar injury?
Hypothenar muscles
Dorsal interossei
Thenar muscles
Palmar interossei
Explanation: Thenar muscles are mainly median nerve-innervated (except adductor pollicis). Hence, deep branch ulnar injury spares most thenar muscles but affects hypothenar, interossei, and adductor pollicis.
Keyword Definitions:
Clavicle: The collarbone connecting the sternum to the scapula.
Shoulder Girdle: The bony ring formed by the clavicle and scapula.
Subclavius Muscle: Small muscle beneath the clavicle that stabilizes it.
Pectoralis Major: Large chest muscle involved in arm flexion and adduction.
Latissimus Dorsi: Large back muscle aiding in arm extension and rotation.
Serratus Anterior: Muscle on the side of the thorax, important in scapular protraction.
SC Joint: Sternoclavicular joint connecting the clavicle to the sternum.
Acromioclavicular Joint: Joint between clavicle and scapula's acromion process.
Shoulder Movement: Includes abduction, adduction, flexion, extension, rotation, and circumduction.
Lead Question - 2012:
Which muscle steadies the clavicle during movement of shoulder?
a) Pectoralis major
b) Latissimus dorsi
c) Subclavius
d) Serratus anterior
Explanation:
Correct Answer: c) Subclavius
The subclavius muscle, located between the first rib and clavicle, acts to stabilize the clavicle during movements of the shoulder girdle. It protects the subclavian vessels and brachial plexus by cushioning against clavicular displacement. Without its stabilizing role, excessive clavicular movement could impair arm mechanics and risk injury to underlying neurovascular structures.
1) Which joint connects the clavicle to the sternum?
a) Acromioclavicular joint
b) Glenohumeral joint
c) Sternoclavicular joint
d) Costoclavicular joint
Explanation:
Answer: c) Sternoclavicular joint. This is a synovial saddle joint that anchors the clavicle to the sternum. It allows limited but essential movements of the clavicle, facilitating full range of shoulder motion while maintaining stability for upper limb function.
2) A fracture at the midshaft of the clavicle may endanger which underlying structure?
a) Axillary artery
b) Subclavian vessels
c) Cephalic vein
d) Thoracodorsal nerve
Explanation:
Answer: b) Subclavian vessels. These lie posterior and inferior to the clavicle. Midshaft fractures may cause displacement that risks vessel injury. The subclavius muscle provides partial protection, but severe trauma can still compromise blood flow.
3) Which nerve supplies the subclavius muscle?
a) Suprascapular nerve
b) Nerve to subclavius
c) Long thoracic nerve
d) Thoracodorsal nerve
Explanation:
Answer: b) Nerve to subclavius. This small branch arises from the upper trunk of the brachial plexus (C5-C6). It innervates the subclavius muscle, enabling it to stabilize the clavicle during pectoral girdle movements.
4) In an overhead throw, which muscle prevents excessive superior displacement of the clavicle?
a) Subclavius
b) Deltoid
c) Trapezius
d) Rhomboid major
Explanation:
Answer: a) Subclavius. The subclavius contracts to stabilize and slightly depress the clavicle, preventing it from riding upward and disrupting the sternoclavicular joint during forceful upper limb actions.
5) Which movement would be most affected by loss of subclavius muscle function?
a) Elbow extension
b) Shoulder abduction
c) Scapular retraction
d) Wrist flexion
Explanation:
Answer: b) Shoulder abduction. Without clavicular stability, abduction beyond 90° becomes inefficient, as the clavicle must rotate posteriorly to allow full range of shoulder movement.
6) The costoclavicular ligament is located between the clavicle and which structure?
a) First rib
b) Second rib
c) Acromion
d) Coracoid process
Explanation:
Answer: a) First rib. It anchors the clavicle to the first rib, limiting excessive elevation and assisting in stabilization during shoulder girdle motion.
7) Which muscle is an antagonist to the subclavius in clavicular elevation?
a) Upper trapezius
b) Latissimus dorsi
c) Pectoralis major
d) Rhomboid minor
Explanation:
Answer: a) Upper trapezius. It elevates the clavicle at the sternoclavicular joint, opposing the depressing action of the subclavius muscle.
8) A patient with brachial plexus injury at C5-C6 may lose function of which muscle stabilizing the clavicle?
a) Subclavius
b) Deltoid
c) Trapezius
d) Serratus anterior
Explanation:
Answer: a) Subclavius. C5-C6 roots contribute to the nerve to subclavius. Injury here can weaken clavicular stability during shoulder movement, impairing upper limb function.
9) Which muscle originates from the first rib and inserts into the clavicle?
a) Subclavius
b) Pectoralis minor
c) Serratus anterior
d) Levator scapulae
Explanation:
Answer: a) Subclavius. Its origin from the first rib and insertion into the inferior clavicle allows it to serve as a dynamic stabilizer during shoulder girdle movement.
10) Injury to the subclavius may indirectly affect which joint most?
a) Glenohumeral joint
b) Sternoclavicular joint
c) Humeroulnar joint
d) Radiocarpal joint
Explanation:
Answer: b) Sternoclavicular joint. Without subclavius support, the SC joint becomes more vulnerable to instability and dislocation during powerful arm movements.